SIM Ophthalmology

December 8, 2017 | Author: Cristian Louie Bagang | Category: Cornea, Human Eye, Retina, Optics, Lens (Optics)
Share Embed Donate


Short Description

ophthalmology...

Description

Self-Instructional Materials in Ophthalmology Edited By

Marissa N. Valbuena M.D., MHPEd Associate Professor Department of Ophthalmology and Visual Science College of Medicine University of the Philippines Manila

July 2005

i

Contents Authors Preface

iv vi

1. Anatomy of the Eye Marissa N. Valbuena M.D., MHPEd

1

2. Physiology of the Eye Richard C. Kho, MD

18

3. Ocular Symptomatology Marissa N. Valbuena M.D., MHPEd & Arnold T. Salud M.D.

33

4. Eye Examination Teresita R. Castillo, MD, MHPEd

38

5. Disturbance in Vision 5.1 Disorders of the Cornea Ruben LimBonSiong, MD

53

5.2 Cataract Leonardo R. Mangubat, MD

67

5.3 Disorders of the Retina, Choroid and Vitreous Pearl T. Villalon, MD

73

5.4 Glaucoma Norman M. Aquino, MD & Marissa N. Valbuena M.D., MHPEd

88

5.5 Disorders of the Optic Nerve Raul D. Cruz, MD

98

5.6. Errors of Refraction Juan Ma. Pablo R. Nañagas, MD, MPH, MSNA

107

6. Red Eye , Tearing and Discharge 6.1 The Red Eye Leo D. P. Cubillan, MD, MS

115

6.2 Uveitis and Scleritis Teresita R. Castillo, MD, MHPEd

120

6.2 Tearing Alexander D. Tan, MD

146

ii

7. Deviation and Displacement of the Eye 7.1 Strabismus Marissa N. Valbuena M.D., MHPEd

153

7.2 Proptosis Prospero Ma. C. Tuaño, MD

167

8. Special Topics 8.1 Retinoblastoma Rolando Enrique D. Domingo, MD

179

8.2 Ocular Manifestations of Systemic Diseases Romulo N. Aguilar, MD, PhD & Teresita R. Castillo, MD, MHPEd

187

8.3 Eyelid Malposisitons Franklin P. Kleiner, M.D.

206

8.4 Ocular Trauma and Emergencies Ma. Margarita L. Luna, MD, Marissa N. Valbuena M.D., MHPEd & Paulo Ma. N. Pagkatipunan, MD, MHA

221

8.5 Ocular Pharmacology Rosie R. Noche, MD

235

Authors

iii

Romulo N. Aguilar, MD, PhD Clinical Associate Professor Department of Ophthalmology and Visual Science College of Medicine University of the Philippines Manila Ocular Manifestations of Systemic Diseases

Richard C. Kho, MD Clinical Associate Professor Department of Ophthalmology and Visual Science College of Medicine University of the Philippines Manila Physiology of the Eye

Norman M. Aquino, MD Clinical Associate Professor Department of Ophthalmology and Visual Science College of Medicine University of the Philippines Manila Glaaucoma

Franklin P. Kleiner, MD Clinical Associate Professor Department of Ophthalmology and Visual Science College of Medicine University of the Philippines Manila Eyelid Malpositions

Teresita R. Castillo, MD, MHPEd Clinical Associate Professor Department of Ophthalmology and Visual Science College of Medicine University of the Philippines Manila Eye Examination ; Uveitis and Scleritis ; Ocular Manifestations of Systemic Diseases

Ruben LimBonSiong, MD Clinical Associate Professor Department of Ophthalmology and Visual Science College of Medicine University of the Philippines Manila Disorders of the Cornea

Leo D. P. Cubillan, MD, MS Clinical Associate Professor Department of Ophthalmology and Visual Science College of Medicine University of the Philippines Manila Red Eye

Ma. Margarita L. Luna, MD Clinical Associate Professor Department of Ophthalmology and Visual Science College of Medicine University of the Philippines Manila Ocular Trauma and Emergencies

Raul D. Cruz, MD Clinical Associate Professor Department of Ophthalmology and Visual Science College of Medicine University of the Philippines Manila Disorders of the Optic Nerve

Leonardo R. Mangubat, MD Associate Professor Department of Ophthalmology and Visual Science College of Medicine University of the Philippines Manila Cataract

Rolando Enrique D. Domingo, MD Clinical Associate Professor Department of Ophthalmology and Visual Science College of Medicine University of the Philippines Manila Retinoblastoma

Juan Ma. Pablo R. Nañagas, MD, MPH, MSNA Professor Department of Ophthalmology and Visual Science College of Medicine University of the Philippines Manila Errors of Refraction

iv

Paulo Ma. N. Pagkatipunan, MD Clinical Associate Professor Department of Ophthalmology and Visual Science College of Medicine University of the Philippines Manila Ocular Trauma and Emergencies

Prospero Ma. C. Tuaño, MD Professor Department of Ophthalmology and Visual Science College of Medicine University of the Philippines Manila Proptosisi

Rosie R. Noche, MD Clinical Associate Professor Department of Ophthalmology and Visual Science College of Medicine University of the Philippines Manila Ocular Pharmacology

Marissa N. Valbuena, MD, MHPEd Associate Professor Department of Ophthalmology and Visual Science College of Medicine University of the Philippines Manila Anatomy of the Eye ; Ocular Symptomatology ; Glaucoma ; Strabismus ; Ocular Trauma and Emergencies

Arnold T. Salud, MD Clinical Associate Professor Department of Ophthalmology and Visual Science College of Medicine University of the Philippines Manila Ocular Symptomatology

Pearl T. Villalon, MD Associate Professor Department of Ophthalmology and Visual Science College of Medicine University of the Philippines Manila Disorders of the Retina, Choroid and Vitreous

Alexander D. Tan, MD Clinical Associate Professor Department of Ophthalmology and Visual Science College of Medicine University of the Philippines Manila Tearing

v

Preface In the Organ System Integration Curriculum of the UP College of Medicine the medical student will have their first exposure to the field of Ophthalmology at Year Level IV. The Sensory Organs – Eye Module is a 4-day rotation consisting of didactic lectures, small group discussions and practicum of skills in history taking and ocular examination. Aside from the introductory lectures in Anatomy and Physiology of the Eye and Ocular History and Eye Examinations, the rest of the module will be problem based, covering the different eye problems that patients may present in the clinic. This series of self-instructional materials is organized in the same manner, with additional topics of Ocular Manifestations of Systemic Diseases, Ocular Trauma and Emergencies and Ocular Pharmacology at the end of the series. These study materials will supplement the lectures the medical students will receive and will also help them in preparing for the small group discussions. Marissa N. Valbuena MD, MHPEd July 2005

vi

ANATOMY OF THE EYE /1

ANATOMY OF THE EYE Marissa N. Valbuena M.D., MHPEd INTRODUCTION An understanding of the anatomy of the eye, orbit, visual pathway and the central control of ocular movements is essential in understanding the eye diseases and other diseases which have ocular manifestations. This module is an overview of the anatomy of the eye and the student is advised to read the references listed at the end of the module for more details.

OBJECTIVES After the completion of this instructional material, the student is expected to 1. Describe the different parts of the eye and adnexae. 2. Describe the functions of the parts of the eye and adnexae.

PREREQUISITE KNOWLEDGE AND PREPARATION The materials discussed in this module is the prerequisite of all the subsequent modules.

INTENDED USERS This module was developed to provide the medical student with the background knowledge of the anatomy of the eye and adnexae. Together with the module on “Physiology of the Eye”, this module will help the student understand how the eye functions, how patients can be evaluated and examined and how the different eye disorders manifest in patients..

CONTENT Outline : A. Orbit B. Eyeball 1. Conjunctiva 2. Tenon’s capsule 3. Sclera and episclera 4. Cornea 5. Uveal tract – iris, ciliary body, choroid 6. Lens 7. Aqueous 8. Anterior chamber angle 9. Retina 10. Vitreous C. Extraocular muscles D. Ocular adnexae 1. Eyebrows 2. Eyelids 3. Orbital septum 1

ANATOMY OF THE EYE /2

4. Lid retractors 5. Lacrimal Comples E. Optic nerve ORBIT The orbit is a pear shaped structure with the optic nerve as its stem. It is 30 cc in volume in adults and the eye occupies 20 % of the space and the muscles and fat accounts for the rest. The orbit is limited anteriorly by the orbital septum, which serves as a barrier between the eyelid and the orbit. It is also related to the frontal sinus above, maxillary sinus below and the ethmoid and sphenoid sinuses medially. Orbital Walls 1. Roof : frontal bone, sphenoid bone 2. Lateral wall : sphenoid bone, zygomatic bone 3. Floor : maxillary bone, zygomatic bone 4. Medial wall : ethmoid, lacrimal bone, frontal bone, maxillary bone

Fig 1. Orbital walls

Orbital Apex The orbital apex is the entry site of all the nerves and blood vessels to the eye and all the extraocular muscles except the inferior oblique

Fig 2. Orbital apex

2

ANATOMY OF THE EYE /3

Blood Supply A. Arterial Supply : Ophthalmic Artery (branch of internal carotid artery) 1. Central retinal artery 2. Lacrimal artery – supplies lacrimal gland and upper eyelid 3. Muscular branches to the muscles – continue to form the anterior ciliary arteries and supply the sclera, episclera, limbus and conjunctiva and contribute to the major arterial circle of the iris. 4. Long posterior ciliary arteries – supplies the ciliary body. The 2 long posterior ciliary arteries anastomose with each other and with the anterior ciliary arteries to form the major arterial circle of the iris. 5. Short posterior ciliary arteries – supply choroid and part of the optic nerve 6. Medial palpebral arteries to both eyelids B. Venous Drainage : Superior and inferior ophthalmic veins, into which drains the vortex veins, anterior ciliary veins and the central retinal vein. The ophthalmic veins communicate with the cavernous sinus. The skin of the periorbital region drain to the angular vein, and to the supraorbital and supratrochlear vein branches of the superior ophthalmic vein. This provides a direct communication between the skin of the face and the cavernous sinus. EYEBALL 1. CONJUNCTIVA The conjunctiva is a thin transparent mucous membrane consisting of 2 parts 1. Palpebral conjunctiva – lines the posterior surface of the eyelid and is adherent to the tarsus. 2. Bulbar conjuctiva – is loosely attached to the orbital septum in the fornices and is folded many times. This allows the eye to move and enlarge the secretory conjunctival surface. The semilunar fold is a thickened fold of bulbar conjunctival at the inner canthus and corresponds to the nictitating membrane of lower animals. The conjunctiva has the following layers: 1. Conjunctival epithelium – consists of 2-5 layers of stratified columnar epethelial cells. The superficial epithelial cells consists of mucous secreting goblet cells. The basal epithelial cells are deeper and may contain pigments near the limbus. 2. Conjuctival stroma has an adenoid (superficial) layer and a fibrous (deep) layer. The adenoid layer contains lymphoid tissue and ‘follicle-like” structures without germinal centers. and develops after the 2nd or 3rd month of life. The fibrous layer is composed of connective tissue that attaches to the tarsus and is loosely arranged over the globe. The accessory lacrimal glands (glands of Krause and Wolfring) located in the stroma resemble the lacrimal gland in structure and function. The conjunctival arteries are derived from the anterior ciliary and palpebral arteries and anastomose freely. Conjuctival veins follow the arterial pattern. The conjuctival lymphatics with the lymphatics of the eyelids form a rich lymphatic plexus. The conjunctiva is innervated by the ophthalmic (first) division of the trigeminal nerve. 2. TENON’S CAPSULE The Tenon’s capsule is a fibrous membrane covering the globe from the limbus to the optic nerve At the limbus, the conjuctiva, Tenon’s capsule and the episclera are fused together. Posteriorly the inner surface of the Tenon’s capsule lies against the sclera and the outer aspect lies in contact with the orbital fat and structures within the extraocular muscle cone. At the point where Tenon’s capsule is pierced by the tendons

3

ANATOMY OF THE EYE /4

of the extraocular muscles, it sends out tubular reflections around each of the muscles. These fascial reflections become continuous with the fascia of the muscles and the fused fascia send out expansions to the surrounding structures and to the orbital bones called check ligaments. Inferiorly, the Tenon’s capsule fuse with the fascia of the inferior rectus and inferior oblique to form the suspensory ligament of Lockwood, upon which the globe rests. 3. SCLERA AND EPISCLERA The sclera is the fibrous outer layer of the eye consisting mainly of collagen. It is dense and white and continuous with the cornea anteriorly and the optic nerve dural sheath posteriorly. It is thinnest at the insertion of the recti mucles (0.3 mm); elsewhere it is 0.6 mm thick. The outer layer of the anterior sclera is covered with a thin layer of fine elastic tissue, the episclera, which contains blood vessels that nourish the sclera.

Fig 3. Cross section of the eye

4. CORNEA The cornea is a transparent tissue inserted to the sclera at the limbus. It is thicker at the periphery (0.65 mm) than at the center (0.52 mm). Its horizontal diameter (11.75 mm) is slightly bigger than its vertical diameter (10.6 mm) There are 5 layers of the cornea : 1. Epithelium : 5-6 layers of cells, continuous with the epithelium of the bulbar conjunctiva 2. Bowman’s membrane : clear acellular layer, a modified portion of the stroma. 3. Stroma : 90 % of corneal thickness; composed of intertwining lamellae of collagen fibrils that run parallel to the surface of the cornea and because of their size and proximity are optically clear. The lamellae run within the ground substance of hydarated polyglycans in association with the keratocytes that produce the collagen and ground substance. 4. Descemet’s membrane : basal lamina of corneal endothelium 5. Endothelium : single layer of cells ; responsible for maintaining the deturgescence of the cornea and failure of function leads to corneal edema. Cell loss occurs with age and injury. Endothelial repair occurs with cell enlargement and sliding of existing cells with minimal capacity for cell division.

Fig 4. Cross section of the cornea

4

ANATOMY OF THE EYE /5

The cornea gets its nutrition from the vessels of the limbus, the aqueous and the tears. The superficial cornea gets most of its oxygen from the tears. The sensory nerves of the cornea is from the ophthalmic division of the trigeminal nerve. The transparency of the cornea is due to its uniform structure, avascularity, and deturgescence. 6. UVEAL TRACT The uveal tract is composed of the iris , the ciliary body and the choroid. It is the middle vascular layer of the eye and contributes to the blood supply of the retina. A. IRIS Is a flat surface with a central opening, the pupil. The iris lies in contiguity with the anterior surface of the lens, dividing the anterior chamber from the posterior chamber, both of which contains aqueous humor. Within the stroma of the iris are the sphincter and dilator muscles. The 2 pigmented posterior layers of the iris represent anterior extensions of the neuroretina and the retinal pigment epithelium (RPE). The blood supply of the iris is from the major circle of the iris. The iris capillaries are non fenestrated. Sensory supply is from fibers of the ciliary nerve. The pupil controls the light entering the eye. The papillary size is determined by the balance between constriction due to parasympathetic activity via the oculomotor nerve and dilation due to sympathetic activity. B. CILIARY BODY The ciliary body consists of 2 zones 1. Pars plicata : 2 mm wide; ciliary processes arise from this zone. The ciliary processes are composed mainly of large fenestrated capillaries and veins that drain to the vortex veins. The 2 layers of the ciliary epithelium are the internal non pigmented layer (representing the anterior extension of the neuroretina) and the external pigmented layer (representing the RPE). The ciliary processes produce the aqueous. 2. Pars plana – 4 mm ; flattened posterior zone The ciliary muscle is composed of longitudinal, circular and radial fibers. 1. Circular fibers: contraction and relaxation of the zonular fibers alters the capsule of the lens thus giving variable focus for far and near objects of fixation. 2. Longitudinal fibers : insert to the trabecular meshwork, influencing its pore size 3. Radial fibers The blood supply of the ciliary body is from the major circle of the iris and the nerve supply is from the ciliary nerves.

5

ANATOMY OF THE EYE /6

Fig 5. Vascu;lar supply of the eye

C. CHOROID The choroid is the posterior portion of the uveal tract, located between the retina and the sclera. The internal portion of the choroidal vessels is called the choriocapillaries. Blood from the choroidal vessels drain via 4 vortex veins, one in each posterior quadrant. The choroid nourishes the outer portion of the retina.

Fig 6. Cross section of the choroid

7. LENS The lens is a biconvex, avascular clear structure, 4 mm thick and 9 mm in diameter. It is suspended behind the iris by the zonules which connects it with the ciliary body. Anterior to the lens is the aqueous and posterior to it is the vitreous. The lens capsule is a semi-permeable membrane (to water and electrolytes). A subcapsular epithelium is present anteriorly. The lens nucleus is harder than the cortex. With age, the subepithelial lamellar fibers are continuously produced, gradually making the lens larger and less elastic.

6

ANATOMY OF THE EYE /7

The lens consists of 65 % water and 35% protein and minerals. There are no blood vessels, pain fibers of nerves in the lens.

Fig. 7. Magnified view of a section of the lens showing lens capsule and epithelium

8. AQUEOUS The aqueous is a clear fluid that fills the anterior and posterior chambers of the eye. Its volume is about 230 µL and its rate of production which is subject to diurnal variation is 2.5 µL/ min. Its composition is similar to plasma except for higher concentration of ascorbate, pyruvate and lactate and lower concentrations of protein, urea and glucose. Aqueous is produced by the ciliary epithelium. From the posterior chamber, the aqueous pass through the pupil to go to the anterior chamber and then to the trabecular meshwork, to the Schelemm’s canal and into the venous system. Some aqueous passes between the bundles of the ciliary body and through the sclera (uveoscleral pathway). 9. ANTERIOR CHAMBER ANGLE The anterior chamber angle lies at the junction of the periphearal cornea and the root of the iris. Its main anatomic features are Schwalbe’s line, trabecular meshwork ( which overlies the Schlemms’s canal) and the scleral spur. The Schwalbe’s line corresponds to the termination of the corneal endothelium. The trabecular meshwork is triangular in cross section with the base directed to the ciliary body. I is composed of perforated sheets of collagen and elastic tissue with decreasing pore size as the canal of Schlemm is approached. The longitudinal muscles of the ciliary body insert into the trabecular meshwork. The scleral spur is an inward extension of the sclera between the ciliary body and the Schlemm’s canal, to which the ciliary body and the iris are attached.

7

ANATOMY OF THE EYE /8

Fig 8. Anterior chamber angle

10. RETINA The retina is a thin, semi-transparent, multilayered sheet of neural tissue that lines the inner aspect of the posterior 2/3 of the wall of the eye. It extends anteriorly as the ora serrata. The outer surface of the retina is apposed to the retinal pigment epithelium (RPE). Except at the disc and the ora serrata, the retina and RPE are easily separated to form a subretinal space, such as occurs in retinal detachment. The inner layer of the retina is apposed to the vitreous The 10 layers of the retina, from the inner aspect are the following: 1. internal limiting membrane 2. nerve fiber layer – ganglion cell axons passing to the optic nerve 3. ganglion cell layer 4. inner plexiform layer – connections of the ganglion cells with the amacrine and bipolar cells 5. inner nuclear layer – cell bodies of the bipolar, amacrine and horizontal cells 6. outer plexiform layer – connections of the bipolar and horizontal cells with the photoreceptors 7. outer nuclear layer – cell nuclei of photoreceptors 8. external limiting membrane 9. phototreceptor layer – rod and cones inner and outer segments 10. retinal pigment epithelium (RPE) – The inner layer of the Bruch’s membranes is actually the basement membrane of the RPE The retina is 0.1 mm thick at the ora serrata and 0.56 mm thick at the posterior pole. In the center of the posterior retina is the macula. It is clinically seen as a 3 mm area of yellowish pigmentation (due to xanthophylls pigments) and bounded by the temporal vascular arcades. In the center of the macula is the fovea, clinically seen as a depression and corresponds to the “foveal reflex”. It corresponds to the retinal avascular zone of fluorescein angiography. Histologically, the fovea is characterized by thinning of the outer nuclear layer and the absence of the other parenchymal layers. The foveola is the most central portion of the fovea, in which the photoreceptors are all cones, and the thinnest part of the retina. All these histologic features provide for fine visual discrimination. The normally empty extracellular space of the retina is potentially greatest at the macula, and diseases that can lead to accumulation of fluid causes thickening of this area.

8

ANATOMY OF THE EYE /9

Fig 9. Layers of the retina

The retina receives its blood supply from 1. choriocapillaries – supply outer third of retina, from outer plexiform layer to RPE 2. central retinal artery – supply the inner 2/3 of the retina The fovea is supplied entirely by the choriocapillaries and is susceptible to irreparable damage when the macula is detached. The retinal blood vessels have a nonfenestrated endothelium, which forms the inner blood-retinal barrier. The endothelium of the choroidal vessels is fenestrated. The outer blood-retinal barrier lies at the level of the RPE.

Fig 10.

Macula

9

ANATOMY OF THE EYE /10

Fig 11. Histophotograph of the retina at the area of the macula

Fig 12. Diagram of the layers of the retina in the area of the macula

11. VITREOUS The vitreous is a clear, avascular body, comprising 2/3 of the volume and weight of the eye. It fills the space bounded by the lens, retina and optic disc. The hyaloid membrane, the outer surface of the vitreous is in contact with the posterior lens capsule, zonules, pars plana epithelium, retina and optic nerve head. The base of the vitreous maintains a firm attachment through out life with the pars plana epithelium and the retina immediately behing the ora serrata. The attachment to the lens capsule and the optic nerve head is firm early in life but soon disappears. The vitreous is 99% water. Collagen and hyaluronic acid makes the vitreous gel like because of their ability to bind large amounts of water. EXTRAOCULAR MUSCLES The 4 recti muscles originate from the annulus of Zinn at the apex of the orbit and are named after their insertion at the sclera on the medial, lateral, superior and inferior aspect of the eye. The superior oblique is the longest and thinnest of the extraocular muscles. The inferior oblique originates from the nasal side of the orbital wall and is the only extraocular muscle that does not originate from the apex of the orbit. Table 1 below summarizes the origin, insertion, action and innervation of the extraocular muscles.

10

ANATOMY OF THE EYE /11

Muscle

Origin

Insertion

Medial rectus (MR) Lateral rectus (LR) Superior rectus (SR)

Annulus of Zinn

Inferior rectus (IR)

Annulus of Zinn

6.5 mm from inferior limbus

23°

Superior oblique (SO)

Orbit apex above Annulus of Zinn (functional origin at trochlea) Behind lacrimal fossa

Posterior equator at superotemporal quadrant

51°

Posterior to the equator in inferotemporal quadrant

51°

Inferior oblique (IO)

Annulus of Zinn Annulus of Zinn

5.5 mm from medial limbus 6.9 mm from lateral limbus 7.7 mm from superior limbus

Direction of pull 90 °

Action from Primary Position Adduction

Innervation Cranial Nerve III

90°

Abduction

VI

23°

Elevation Intorsion Adduction Depression Extorsion Adduction Intorsion Depression Abduction

III

Extorsion Elevation Abduction

III

III IV

Table 1. Extraocular Muscles

Fig 13. Spiral of Tillaux, showing the insertion of the recti muscles to the sclera

The blood supply to the extraocular muscles is from the musclular branchs of the ophthalmic artery. The lateral retus and inferior obliques are also supplied by the branches from the lacrimal artery and infraorbital artery respectively. OCULAR ADNEXA 1. EYEBROWS The eyebrows are folds of thickened skin covered with hair. The glabella is the hairless prominence in between the eyebrows.

11

ANATOMY OF THE EYE /12

2. EYELIDS The upper and lower lids (palpebrae) are folds of skin that can close to protect the anterior portion of the eye. Blinking helps spread the tear film, keeping the cornea and conjunctiva wet. Layers of the eyelids 1. Skin – thin, loose, elastic, few hair follicles and no subcutaneous fat. 2. Orbicularis oculi muscle – Circular muscle fibers surround the palpebral fissure which functions to close the eyelids. It is innervated by the facial nerve. 3. Areolar tissue – under the orbicularis oculi, communicates with the subaponeurotic layer of the scalp. 4. Tarsal plates – dense fibrous tissue layer ; main support of the eyelids 5. Palpebral conjunctiva – adheres firmly to tarsal plate Lid Margin – free lid margin is 25-30 mm long and 2 mm wide. It is divided by the gray line (mucocutaneous junction) into anterior and posterior margin. 1. Anterior margin a. Eyelashes b. Glands of Zeis – modified sebaceous glands ; open onto hair follicles at the base of eyelashes c. Glands of Moll – modifies sweat glands ; open in a row near the base of the eyelashes 2. Posterior margin – in close contact with the globe ; along margins are the small orifices of the meobomian glands (modified sebaceous glands) 3. Lacrimal punctum – at the medial end of posterior margin of the lid ; small elevation with a central opening ; carry tears through the canaliculus to the lacrimal sac.

Fig 14. Lid margin, medial portion of the eyelids

12

ANATOMY OF THE EYE /13

Fig 15. Cross-section of the eyelids

4. ORBITAL SEPTUM The orbital septum is the fascia behind the portion of the orbicularis muscle that lies between the orbital rim and the tarsus. It serves as a barrier between the lid and the orbit 5. LID RETRACTORS The lid retractors are responsible for opening the eyelids; have striated and smooth muscle components A. Upper lid 1. Levator palpebrae superioris 2. Muller’s muscle (superior tarsal muscle) B. Lower lid 1. Inferior rectus muscle 2. Inferior tarsal muscle 6. LACRIMAL COMPLEX A. Lacrimal gland ; has orbital portion and palpebral portion B. Accessory lacrimal glands of Krause and Wolfring – located in the sustantia propria of palpebral conjunctiva C. Canaliculi D. Lacrimal sac E. Nasolacrimal duct- drains out to the nasal cavity Blood supply of the lacrimal gland is from the lacrimal artery and venous blood drain to ophthalmic vein. Lymphatics drain into preauricular lymph nodes. Nerve supply to the lacrimal gland is by a. lacrimal nerve (sensory), a branch of the trigeminal first division b. great superficial petrosal nerve (secretory) c. sympathetic nerves

13

ANATOMY OF THE EYE /14

Fig .16. Lacrimal drainage system

OPTIC NERVE The trunk of the optic nerve consists of about 1 million axons arising from the ganglion cells of the retina a. intraocular portion – optic nerve head ; 1.5 mm in diameter b. orbital portion – 3 mm in diameter, 25-30 mm long, located within the muscle cone c. intracanalicular portion – 4-9 mm long d. intracranial portion- 10 mm long, and with the opposite optic nerve joins to from optic chiasm

Fig 17. Optic nerve

Fibers of the optic nerve consist of a. visual fibers – 80%, synapse in the lateral geniculate body on neurons whose axons terminate in the visual cortex of the occipital lobe b. pupillary fibers – 20% , bypass the geniculate body en route to the pretectal area. The ganglion cells of the retina and their axons are part of the central nervous system and as such, do not regenerate if severed. The optic nerve sheath is continuous with the meninges.

14

ANATOMY OF THE EYE /15

The surface layer of the optic disc receives blood from the branches of the retinal arterioles. The rest of the nerve in front of the lamina cribrosa is from the peripaillary choroidal vessels. At the region of the lamina cribrosa, the blood supply is from the short posterior ciliary arteries. Retrolaminar nerve receive blood from branches of the central retinal artery. The rest of the introrbital portion, intracnalicular and intracranial portions are supplied by pial vessels from branches of ophthalmic artery and other branches of the internal carotid artery.

Fig 18. Cross-section of the optic nerve

Fig. 19 Blood supply of the optic nerve

15

ANATOMY OF THE EYE /16

SUMMARY An understanding of the anatomy of the eye, ocular adnexae, orbit, visual pathways and the cranial nerves is important in the proper diagnosis of ocular diseases and other disorders with ocular manifestations. REFERENCES 1. Duane, Thomas and Jaeger, Edward . Clinical Ophthalmology, Philadelphia : Harper and Row , latest edition 2. Riordan-Eva, Whitcher, John. Vaughn and Ashbury’s General Ophthalmology , 16th Edition, New York: Lange Medical Books/ McGraw Hill, 2004 3. Scheie, Harold, Albert, Daniel. Textbook of Ophthalmology, Philadelphia : W.B. Saunders Co, latest edition 4. Selected images from the lecture of Leonardo Mangubat, Anatomy of the Eye and Adnexae, SELF-TEST 1. Decrease in aqueous production can best be achieved by destruction of which part of the eye? A. Pars plicata B. Choroid C. Iris D. Pars plana 2. Which one of the following rectus muscle tendons inserts on the sclera farthest from the corneal limbus? A. superior rectus B. inferior rectus C. medial rectus D. lateral rectus 3. The levator palpebrae is innervated by what nerve A. III B. IV C. V D. VII 4. The following structures are part of the medial orbital wall, EXCEPT A. ethmoid bone B. lacrimal bone C. maxillary bone D. sphenoid bone 5.What layer of the retina does the the choriocapillary supply with oxygen? A. ganglion cell layer B. nerve fiber layer C. photoreceptors D. inner nuclear layer

16

ANATOMY OF THE EYE /17

6.Which of the following statements regarding the cornea is FALSE ? A. The corneal endothelium is important in maintaining corneal dehydration. B. The water content of the cornea is less than that of the sclera. C. Normal central corneal thickness is 1.00 mm D. Corneal diameter is greater horizontally than vertically. 7. Which is not a layer of the eyelid ? A. Skin B. Conjunctiva C. Tenon’s capsule D. Orbicularis muscle E. Tarsus 8.The following structures must maintain their clarity in order good vision EXCEPT A. Cornea B. aqueous C. lens D. vitreous E. choroid 9. The optic nerve consists of axons from what cells in the retina? A. amacrine cells B. bipolar cells C. ganglion cells D. photoreceptor cells 10. Which muscle is an adductor? A. medial rectus B. lateral rectus C. superior oblique D. inferior oblique ANSWERS TO SELF-TEST 1. A 2. A 3. A 4. D 5. C 6. C 7. C 8. E 9. C 10. A .

17

PHYSIOLOGY OF THE EYE /18

PHYSIOLOGY OF THE EYE Richard C. Kho, M.D. INTRODUCTION This self-instructional material (SIM) is designed to help the medical student acquire an overview of the biophysical elements at work within (and outside) the human eye, for the latter to function as a sense organ subserving vision. Understanding basic concepts of light energy, its “transformation” in the human eye, its conversion to nerve impulses and eventual visual perception, is a pre-requisite to effective diagnosis and subsequent management of eye diseases.

OBJECTIVES Upon completion of this SIM, the student should be able to discuss the following :: 1. The physical properties of light 2. The processes involved as soon as light strikes the human eye 3. The internal bending of light as it focuses on the retina, i.e., optics and refraction in the human eye 4. Retinal processes which transform light energy that result in visual perception 5. Basic neuro-anatomic architecture of the visual pathway, as well as topographical localization of lesions

PREREQUISITE KNOWLEDGE AND PREPARATION Students should have a working knowledge of the basic anatomy of the human eye. A general knowledge of the neuro-anatomy of the afferent visual pathways would likewise be useful.

INTENDED USERS This SIM was prepared for the medical student just embarking on the study of the anatomy and physiology of the human eye. It does not aim to supplant ophthalmology textbooks which provide a more detailed discussion of advanced concepts in optics and refraction, retinal physiology, and neuro-ophthalmology.

CONTENT This module is divided into two parts: PART I: The Eye as an Optical Instrument A. Physical Optics -The physical properties of light B. Geometric Optics -The process in which external light energy is focused on the retina PART II: The Eye as a Sense Organ C. Physiologic Optics -The biochemical and functional processes that occur in the retina to produce visual energy

18

PHYSIOLOGY OF THE EYE/ 19

D. Psychologic Optics or Neuro-Ophthalmologic Optics -The conduction of visual energy to the occipital visual center PART I: The Eye as an Optical Instrument A. Physical Optics

Light is the basic stimulus for vision. This comprises only a small portion of the electromagnetic spectrum of energy:

Fig 1. The Electromagnetic Spectrum

This small portion, called the visible spectrum, is the ONLY portion of the spectrum that can stimulate the photoreceptors of the human retina. It extends from 380 micra (3800 angstrom units) to 760 micra (7600 angstrom units). Right after the UV spectrum (violet), the wavelength of each color increases as it moves toward the direction of infrared rays (red). There are 3 Important Characteristics of Light: 1) Velocity or Speed -3 X 1010 cm/sec in vacuum; slower in clear air and in denser media. 2) Wavelength -size determines the color; with violet (380µ ) the shortest, and red (760µ ) the longest.

Fig 2. Wavelength

3) Frequency -number of complete cycles moving past a specific point over a given period of time. *Velocity = Wavelength X Frequency

19

PHYSIOLOGY OF THE EYE/ 20 PART I: The Eye as an Optical Instrument B. Geometric Optics This process, in-between physical optics and physiologic optics, comprise of events that occur from the moment light strikes the eye, and eventually gets focused on the retina. Its principal basis is the transmission and bending of the direction of travel of light rays, i.e., REFRACTION. Refraction of Light As light passes through a transparent solid or liquid media, it slows down depending on the density of the media. The relative unit of measurement of this capacity is called the index of refraction. The Refractive Index (n) is a constant depending on the material; it determines the angle of deviation. air = 1.0 water = 1.33 glass > 1.40 It is simply a relative unit compared to air. As light passes from one medium to another of a different index of refraction and at a certain angle, there is bending of light, i.e. light is Refracted.

Fig 3. Refraction of Light

Prism Any media whose 2 sides are not parallel will refract light rays ---- light is deviated towards the base of the prism. apex

light source

base Fig.4 Prismatic Effect on Light

20

PHYSIOLOGY OF THE EYE/ 21 Basis of Lenses Lenses can be viewed as a certain arrangement of prisms (remember that light is deflected towards the base of the prism). A converging lens (positive lens) can be thought of as two prisms joined at the base, while a diverging lens (negative lens) can be thought of as two prisms joined at the apex.

converging

diverging

Fig 5. Converging and Diverging Lenses

Power of the Lens A Diopter is a unit of measurement of lens power. It is a measure of convergence or divergence, and a reciprocal of focal distance. The power of the lens depends on its curvature and the difference in refractive indices. The Eye Can be thought of as a series of lenses whose main goal is to focus light rays from the external world unto the retina: – cornea – aqueous – lens – vitreous The average human eye has a total converging power of about 60 diopters. The main refractive components are as follows: Cornea ~ +40 Diopters Lens ~ +20 Diopters

Emmetropia is a condition wherein parallel light rays fall into a pinpoint focus on the retina.

Fig 6. Emmetropia: Light is focused ON the retina 21

PHYSIOLOGY OF THE EYE/ 22

Ammetropia is a condition wherein parallel light rays DO NOT fall into a pinpoint focus on the retina:

•Myopia •Hyperopia •Astigmatism Myopia, commonly known as “nearsightedness”, is a condition wherein parallel light rays focus at a point in front of the retina. It can be axial (eyeball longer than average) or refractive (corneal curvature steeper than average).

Fig 7. Myopia: Light is focused IN FRONT OF the retina

To Correct Myopia, one would need a divergent lens (“negative” or biconcave lens to neutralize the convergent effect of the myopic eye) in order to focus light rays on the retina.

Fig 8. A Negative Lens “pushes back” the image unto the retina Hyperopia, commonly known as “farsightedness”, is a condition wherein parallel light rays focus at a point behind the retina. It can be axial (eyeball shorter than average) or refractive (corneal curvature flatter than average).

Fig 9. Hyperopia: Light is focused BEHIND the retina

22

PHYSIOLOGY OF THE EYE/ 23 To Correct Hyperopia, one would need a convergent lens (“positive” or biconvex lens) in order to focus light rays on the retina.

Fig 10. A Positive Lens “pulls frontward” the image unto the retina

Astigmatism is a condition wherein the curvature of the cornea or of the lens is not the same in different meridians. Here, parallel light rays focus on 2 separate lines or planes. One can imagine that the curvature of the eye in astigmatism resembles one side of a football, instead of a basketball (in eyes without astigmatism). To correct astigmatism, one would need cylindrical lenses (lenses each with power in two different meridians/axes)

spherical

astigmatic

Fig 11.The front curvature of two different balls illustrate the difference in the curvature of spherical corneas (basketball) vs. astigmatic corneas (football).

Types of Astigmatism: 1. Simple Myopic - one image on the retina, one image in front of the retina 2. Simple Hyperopic - one image on the retina, one image behind the retina 3. Compound Myopic - both images in front of the retina 4. Compound Hyperopic - both images at the back of the retina 5. Mixed Astigmatism - one image in front of the retina, one image at the back of the retina Correction of Ammetropia: 1. Spectacles 2. Contact lenses • soft, rigid gas permeable, hard, etc. • multifocal 3. Refractive Surgery • PRK (photorefractive keratectomy) • RK (radial keratotomy) • LASIK (laser-assisted in situ keratomilieusis) Principle of Accommodation To focus on a nearby object, the brain sends out signals to contract the smooth muscles of the ciliary body; this enables the zonules to loosen up, which in turn increases the lens curvature (lens thickens), and thereby increasing its converging power. Presbyopia With aging (around 40 years old), there is loss of focusing or accommodative power of the human eye. One would need “plus lenses” (presbyopic glasses/reading adds) to make up for the lost automatic focusing power of the lens.

23

PHYSIOLOGY OF THE EYE/ 24 PART II: The Eye as a Sense Organ C. Physiologic Optics The Human Retina is a thin, semi-transparent, multilayered sheet of neural tissue that lines the inner aspect of the posterior 2/3 of the wall of the globe. The young, adult retina contains approximately 120 million rods, and about 6million cones.

Fig 12. Layers of the Human Retina

The human retina is capable of perceiving the following visual senses:

•Light sense •Form sense •Color sense Light Sense: The Role of Visual Pigments For the eye to perceive light, the latter has to be converted into the biochemical energy of the visual nerve impulse. First, it must be absorbed by the visual pigments located at the outer segments of the rods and cones. These visual pigments (rhodopsin, Iodopsin, etc.) are lipid-protein complexes of a fat-soluble aldehyde of Vitamin A, plus a protein called opsin. Vitamin A occurs only in animal tissue. A molecule of its precursor (beta-carotene) derived from plants, is split into two to form molecules of Vitamin A in the form of an alcohol. Vitamin A occurs in two forms (isomers), a cis-retinal and a trans-retinal structure. Only the cis-retinal isomer combines with opsin to form rhodopsin.

Photochemistry of Vision When light strikes rhodopsin, it is split into cis-retinal (cis-retinene) and opsin after passing through a series of orange intermediate compounds (lumirhodopsin, metarhodopsin, etc). Two major events occur with the split of rhodopsin: 1) A sudden reduction of sodium influx through the photoreceptor plasma membrane together with increased permeability of the membrane to calcium ions result in a relative hyperpolarization of the plasma membrane and initiates an electrical/nerve impulse. 24

PHYSIOLOGY OF THE EYE/ 25 2) The transformation of cis-retinene to trans-retinene releases energy. Trans-retinal is reconverted to cis-retinal by the action of the retinene isomeraze enzyme with energy provided by the DPNH2-DPN dehydrogenase system. Cis-retinal, as soon as it is formed combines with opsin to form the stable product rhodopsin. This combination also releases energy which is utilized in the oxidation of retinol (Vit A-alcohol) to retinal (Vit A-aldehyde or retinene).

Fig 13. The Photochemistry of Vision

Form Sense: Visual Acuity Form sense discriminates between stimuli, i.e., to see two stimuli separately as two instead of fusing them into one. It determines the acuity of vision. Simply put, it is the minimum amount of separation between two light sources at a given distance from the eye so that they can still be seen as two. These two lights subserve an angle at the nodal point of the eye called the minimum visual angle.

Minimum Visual Angle

Experimentally, the smallest detectable line subtends one minute of arc.

Fig 14. Minimum Visual Angle •

The big “E” on the Snellen Chart subtends an angle of 5 minutes

Fig 15. The Snellen E and its corresponding visual angles.

25

PHYSIOLOGY OF THE EYE/ 26 Testing Visual Acuity using the Snellen Chart

•Letters are constructed so that they subtend the same visual angle when viewed at distances of up to 200ft

Fig 16. Construction of the Snellen Chart for consistency

One usually measures visual acuity at 20ft (6m) and is recorded as two numbers: The numerator represents the distance between chart and patient, while the denominator represents the smallest row of letters that the patient’s eye can read. For example, a visual acuity of 20/40 simply means that the patient’s eye can only read from 20 ft, what a normal (emmetropic) eye can read at 40ft.

Feet 20/200 20/100 20/70 20/50 20/40 20/30 20/25 20/20

E FP TOZ LPED PECFD EDFCZP

Meters 6/60 6/30 6/21 6/15 6/12 6/9 6/7.5 6/6

FELOPZD DEFPOTEC

Fig 17 and Table 1. Recording Visual Acuity Using the Snellen Chart

Color Sense: A Function of the Cone Photoreceptors White light or sunlight is a composite of different colors corresponding to each wavelength in the visible spectrum.

Fig 18.The Color Spectrum

26

PHYSIOLOGY OF THE EYE/ 27 Color Blindness “Color blindness” occurs in about 10% of all males and about 1% of all females. It has a sex-linked, recessive pattern of inheritance. True color blindness (total absence of one type of photo pigment or color-sensitive cone) is rare. Most of the time, all photo pigments are present except for a relative deficiency of one color---an “anomaly”. • In Trichromats, all 3 colors are present but has a relative deficiency in one. –Deuteranomalous (green anomaly) –Protanomalous (red anomaly) –Trianomalous (blue anomaly) •

In Dichromats, there is total loss of one color pigment –Deuteranopes (no green) –Protanopes (no red) –Trianopes (no blue)

• •

Monochromats or Cone Monochromats (atypical) have only one color pigment Achromats or Rod Monochromat (typical) are totally color blind.

PART II: The Eye as a Sense Organ D. Neuro-ophthalmic Optics Basic Concepts Monocular vision, seen in lower vertebrates, is a less-advanced form of visual function wherein visual impressions from one side cross-over to the contralateral cerebral cortex completely (there is complete decussation).

Fig 19. Visual Pathway in Monocular Vision

In Binocular vision, there is nasal (partial) decussation of fibers from the two sides. As a result, both retinas send the same visual impressions to the visual cortex.

27

PHYSIOLOGY OF THE EYE/ 28

Fig 20. Visual Pathway in Binocular Vision

Stereopsis or depth perception is possible only with binocular vision Neuro-anatomic Pathways These are structures which perceive, relay, and process visual information. From the external world, all the way to its end terminal, the following are its components:



Eye (retina)



Optic Nerve (CN II)



Chiasm



Optic Tract



LateralGeniculate Nucleus (LGN)



Optic Radiation



Striate Cortex

Fig 21. The Afferent Visual Pathway

Note that the visual field and the retina are optically inverted, i.e., the right visual fields (both the right field of right eye and the right field of left eye) are projected to the left hemi-retina of both eyes and, retrochiasmally, the left visual pathway until its termination in the left occipital lobe. Vertically, visual field and retinal projections follow a similar pattern of optical inversion. In addition, there is direct one-to-one correspondence between visual direction in space and retinal location. This retino-topic organization is preserved throughout the entire visual pathway, and this logical architecture is the basis for localization of lesions in the visual pathway via visual field testing (perimetry).

28

PHYSIOLOGY OF THE EYE/ 29 Understanding the neuro-anatomy of the visual pathway is the key to effective evaluation, localization, and eventual diagnosis of many intracranial lesions.

A. B. C. D. E. F. G.

Fig 22. Location of lesion with corresponding visual field defects left optic nerve – central scotoma/generalized depression of the left eye optic chiasm- bitemporal hemianopia left optic chiasm (lateral aspect)- left nasal hemianopia left optic tract- Right Homonymous Hemianopia left optic radiation (temporal lobe)- Right Superior Homonymous Quadrantanopia (“pie in the sky”) left optic radiation (parietal lobe)- Right Inferior Homonymous Quadrantanopia (“pie on the floor’) left occipital lobe (visual/striate cortex)- Right Homonymous Hemianopia SUMMARY

•PART I: The Eye as an Optical Instrument A. Physical Optics • 3 properties of light: 1) velocity 2) wavelength 3) frequency

B. Geometric Optics • refractive index (n) • prisms • lenses (converging and diverging) • emmetropia • ammetropia 29

PHYSIOLOGY OF THE EYE/ 30 myopia hyperopia astigmatism ƒ simple myopic ƒ simple hyperopic ƒ compound myopic ƒ compound hyperopic ƒ mixed astigmatism correction of ammetropia o spectacles o contact lenses o refractive surgery o o o



•PART II: The Eye as a Sense Organ

C. Physiologic Optics • light sense: role of visual pigments o photochemistry of vision • form sense: visual acuity o minimum visual angle o testing visual acuity with the Snellen Chart • color sense: a function of photoreceptors o color blindness ƒ Trichromat ƒ Dichromat ƒ Monochromat ƒ Achromat D. Psychologic Optics or Neuro-Ophthalmologic Optics • monocular vision • binocular vision • neuroanatomy of the afferent visual pathway o lesions and corresponding visual field defects REFERENCES

1) Espiritu RB. Ophthalmologic Optics. Manila: Department of Ophthalmology and Visual SciencesUP-PGH Medical Center; 2001. 2) Vaughan D, Asbury T, Riordan-Eva P, editors. General ophthalmology. 13th ed. Appleton and Lange; 1992. 3) Spalton DJ, Hitchings RA, Hunter PA, editors. Atlas of Clinical Ophthalmology. 2nd ed. London: Wolfe Publishing; 1994. 4) Goldberg S. Clinical Neuroanatomy Made Ridiculously Simple. Miami: Medmaster Inc; 1979. 5) DeMyer W. Technique Of The Neurologic Examination: A Programmed Text. 3rd ed. McGraw-Hill Book Company; 1980. 6) Selected Images from http://www.eyecenter.com.ph

30

PHYSIOLOGY OF THE EYE/ 31 SELF TEST 1. In the visible spectrum, which of the following colors has the shortest wavelength? A. blue B. green C. orange D. red 2. Which of the following happens to the velocity of light as it passes from a medium of higher refractive index, to one of lower refractive index? A. slows down B. speeds up C. stays the same D. is dissipated 3. True or False? A diverging (negative) lens can be thought of as 2 prisms with the bases adjacent. 4. Which of the following structures accounts for the highest refractive omponent of the human eye? A. lens B. cornea C. vitreous D. aqueous 5. Match the following refractive states in reference to the location of the image relative to the retina. ___A.. myopia 1. on the retina ___B.. hyperopia 2. in front of the retina ___C.. emmetropia 3. behind the retina 6. To correct astigmatism, one would need which type of lens? A. diverging (negative) lens B. cylindrical lens C. converging (positive) lens D. prisms 7. To correct hyperopia, one would need which lens? A. diverging (negative) lens B. cylindrical lens C. converging (positive) lens D. prisms 8. Which type of astigmatism has both images focused at the back of the retina? A. simple myopic B. simple hyperopic C. compound myopic D. compound hyperopic E. mixed astigmatism

31

PHYSIOLOGY OF THE EYE/ 32 9. In presbyopia, one would need which type of lens to make up for the lost converging power of the human lens? A. diverging (negative) lens B. cylindrical lens C. converging (positive) lens D. prisms 10. Rhodopsin is formed by the combination of which molecules? A. opsin and beta-carotene B. beta-carotene and Vit A alcohol C. cis-retinal isomer and opsin D. lumirhodopsin and metarhodopsin 11. Each arm (1/5 its total height) of the big “E” in the Snellen Chart subtends how much angle? A. 5 min B. 1 min C. 10 min D. 20 min 12. True or False? In recording visual acuity, the numerator represents the distance between the chart and the patient. 13. Which condition is described as having total loss of red color? A. protanope B. protanomaly C. deuteranope D. deeuteranomaly 14. A lesion of the optic chiasm would most likely present with which kind of visual field defects? A. binasal hemianopia B. left homonymous hemianopia C. bitemporal hemianopia D. left superior homonymous quadrantanopia 15. Which of the following lesions would most likely give rise to a left superior homonymous quadrantanopia? A. left temporal lobe B. right temporal lobe C. left parietal lobe D. right parietal lobe ANSWERS TO SELF-TEST 1) 2) 3) 4) 5)

A B F B A-2, B-3, C-1

6) 7) 8) 9) 10)

B C D C C

11) 12) 13) 14) 15)

32

B T A C B

OCULAR SYMPTOMATOLOGY / 33

OCULAR SYMPTOMATOLOGY Marissa N. Valbuena M.D., MHPEd Arnold T. Salud M.D. INTRODUCTION One should have a good understanding of ocular symptomatology to be able to perform a complete ophthalmic evaluation/ examination, which in turn is necessary to come up with accurate diagnoses.

OBJECTIVES Upon completion of this unit of instruction, the student should be able to 1. discuss the different ocular symptoms. 2. to be able to perform a good ocular history.

PREREQUISITE KNOWLEDGE AND PREPARATION The student should have basic knowledge of the anatomy and physiology of the eye and adnexae. Skills in interviewing a patient will also be helpful.

INTENDED USERS Although this material was developed to provide the medical students with knowledge on ocular symptomatology, this should be supplemented by small group learning directed to developing their skills in taking ocular history.

CONTENT Ocular symptoms can be classified into three general types: 1. abnormalities of vision 2. abnormalities of ocular appearance 3. abnormalities of ocular sensation – pain and discomfort These symptoms should always be described according to 1. onset – gradual, rapid or asymptomatic Example of asymptomatic onset is that the blurring of vision was discovered only when patient inadvertently covered one eye. 2. 3. 4. 5.

duration – brief, chronic frequency – continuous, intermittent degree – mild, moderate or severe location – focal or diffuse, unilateral or bilateral

Determine if forms of treatment have already been initiated/tried. If so, to what extent have they helped to relieve the symptoms? Are there circumstances that provoke or worsen the condition? Is this the first time these symptoms are experienced? No associated other signs/symptoms? 33

OCULAR SYMPTOMATOLOGY / 34

1. ABNORMALITIES OF VISION A. Visual Loss Patients can describe visual loss as “nanlalabo”, “maulap ang panningin”, “nawawala ang paningin” or “nabulag” When a patient reports impairment of vision, the examiner should determine when it occurred, whether onset was sudden or gradual, whether one or both eyes were affected. If both eyes are involved, which is worse, which failed first and how much time has elapsed between the two. Actual onset of visual impairment may not coincide with the time given by the patient. Vision in one eye may have been deteriorating over the years, becoming noticeable when the patient accidentally covered one eye. One should distinguish between decreased central acuity and peripheral vision. Disturbances in peripheral vision may be focal such as scotoma, or may involve a bigger area as in hemianopsia. A scotoma is a blind or partially blind area in the visual field while hemianopsia is blindness in one-half of the visual field. Abnormalities in the central nervous visual pathway disturb the visual field more than the central visual acuity. Is the visual loss transient or permanent? Transient loss of vision may be to vascular disorders anywhere from the retina to the occipital cortex. Is the patient’s vision worse or better refraction may have better vision when will read better if they position their Patients with central focal cataracts may

in some circumstances ? Patients with error of they squint their eyes. Patients with presbyopia reading material further away from their eyes. have worse vision in bright sunlight.

Decline in visual acuity may be due to abnormalities anywhere along the optical and neurologic pathway. Consider the following as possible causes: 1. refractive error 2. ptosis 3. ocular media disturbance (corneal edema, hemorrhage) 4. retinal abnormalities 5. optic nerve diseases 6. intracranial visual pathway abnormalities B. Visual Aberrations 1. GLARE, PHOTOPHOBIA Patients may describe this as “silaw” or “nasisilaw”

34

hyphema,

cataract,

vitreous

OCULAR SYMPTOMATOLOGY / 35

Irritative disease of the conjunctiva or cornea specially foreign bodies of the cornea may induce photophobia. Acute inflammation of the iris may likewise make the eye sensitive to ordinary light. Glare may also result from uncorrected EOR, scratches on spectacle lenses, excessive pupillary dilatation, hazy ocular media 2. VISUAL DISTORTION Manifests as irregular patterns of dimness, wavy or jagged lines, image magnification/ minification. May be caused by migraine, optical distortion from strong corrective lenses, lesions involving the macula and optic nerve. 3.FLASHING/FLICKERING LIGHTS Patients may describe this as “may parang kidlat”, “biglang may maliwanag” May indicate retinal traction, or migrainous scintillations. 4.FLOATING SPOTS “May lumulutang sa harap ng mata” May represent normal vitreous strands due to “normal” vitreous changes.Or may be secondary to pathologic presence of pigments, blood, or inflammatory cells. 5.OSCILLOPSIA “Gumagalaw o lumilikot and paningin” Shaking field of vision may be due to harmless lid twitching (myokymia), or to certain forms of nystagmus 6. DOUBLE VISION “Nagdadalawa ang paningin” “doble ang paningin”, naduduling” Monocular diplopia manifests as a split shadow or ghost image. Causes include uncorrected error of refraction, media abnormalities such as cataract, corneal irregularities Binocular diplopia disappears when one eye is covered may be vertical, horizontal, diagonal or torsional. The diplopia may be more severe ( 2 images more widely separated) in certain gazes or head position.

35

OCULAR SYMPTOMATOLOGY / 36

2 ABNORMALITIES OF APPEARANCE A. Red Eye Must differentiate between redness of the lids and periocular area (ocular adnexa) from that of the globe. Preseptal cellutitis Orbital cellulits “Namamaga ang mata”

VS

Conjunctivitis “namumula ang mata”, “sore eyes” Subconjunctival hemorrhage “dumugo ang mata” Scleritis Iritis Acute glaucoma Pterygium etc

Color abnormalities other than redness 1. jaundice 2. hyperpigmented spots (on the iris/ocular surface) – examples are Nevus of Ota , subepithelial melanosis B. C. D. E.

Ptosis – drooping of the eyelids, “Napipikit”, “kirat ang mata” Focal growths – in the eyelids or eye surface , “bukol”, “maga” Exopthalmos – protrusion of the eyeball, “lumuluwa ang mata” Ocular deviation or misalignlent – “duling”, “banlag” ; esodeviation (inward turning of the eye), exodeviation (outward turning of the eye), hypertropia (upward turning of the eye) or hypotropia (downward turning of the eye)

3. ABNORMALITIES OF OCULAR SENSATION A. Eye Pain “Masakit”, “makirot”, “mahapdi” Must be characterized in terms of location: 1. 2. 3. 4.

periocular (may be tenderness of the lid, tear sac, sinuses or temporal artery) retrobulbar (due to orbital inflammation, orbital myositis, optic neuritis) ocular (may be due to corneal abrasion, corneal foreign body, glaucoma, endophthalmitis) non-specific (fatigue from ocular accommodation, binocular fusion, or referred discomfort from non-ocular tension or fatigue)

Deep seated aching, boring or throbbing pain may be may be due to inflammation of the iris and ciliary body. Orbital infection can give rise to severe pain. Herpez zoster may give pain in the eye before any visible involvement of the eye and may persist after the disease has resolved. Tenderness, soreness or pain on pressure may be due to inflammation of the lids, corneal foreign body or any anterior segment inflammation. B. Eye Irritation Superficial discomfort is usually caused by ocular surface abnormalities 1. Itching – Often a sign of allergic sensitivity, “makati” 36

OCULAR SYMPTOMATOLOGY / 37

2. Dryness – Burning, gritty, mild foreign body sensation. Can occur with dry eyes or other types of mild corneal irritation, “may buhangin”, “maaligasgas” 3. Tearing – may be due to irritation of the ocular surface; or may be a sign of abnormal lacrimal drainage , “nagluluha” 4. Ocular Secretions – “nagmumuta”, Characretize discharge as to color, consistency, amount a Mucoid discharge – allergic b Mucopurulent – bacterial/viral conjunctivitis c Dried matter/crusts on lashes – Blepharitis C. Headache Uncorrected errors of refraction and presbyopia frequently cause headache referred to the eyes or brow and comes with reading and computer work. Migraine headaches and sinusitis are frequent causes of headache. Headaches may not come from the eye. High and low blood pressure may also give rise to headaches around the eyes. Headache from rise in intracranial pressure is usually severe and associated with nausea and vomiting. SUMMARY Ocular symptoms consist of abnormalities in vision, appearance and sensation. The student should ask clarifying questions in order to get sufficient detail to pinpoint the etiology of the ocular disorder. REFERENCE 1. Riordan-Eva, Whitcher, John. Vaughn and Ashbury’s General Ophthalmology , 16th Edition, New York: Lange Medical Books/ McGraw Hill, 2004 2. Scheie, Harold, Albert, Daniel. Textbook of Ophthalmology. Philadelpia : W.B Saunders LEARNING ACTIVITY Students should pair and role play. One will be the doctor and the other the patient. The doctor should take the history of the patient with any of the following chief complaint : 1. “Malabo ang mata” 2. “may sore eyes” 3. “mahapdi ang mata” 4. “banlag” The “doctor” will write the patient’s history and the partner will comment on the completeness and accuracy of the history.

37

BASIC EYE EXAMINATION /38

BASIC EYE EXAMINATION Teresita R. Castillo, MD, MHPEd INTRODUCTION This self-instructional material is designed to help the student learn important concepts on proper eye examination. It will explain how to examine the eye and basic visual function. The proper method of basic eye examination in an individual is an important skill that every physician should possess. Performing a systematic eye examination will enable the physician to evaluate ocular complaints and subsequently provide immediate emergency care whenever the need arises. Furthermore, this will enable the physician to recognize ocular conditions that may require further referral to an ophthalmologist for definitive management. An eye examination may also provide the physician with information on the status or condition of certain systemic diseases such as diabetes, hypertension and thyroid diseases.

OBJECTIVES Upon completion of this unit of instruction, the student should be able to discuss the principles of performing the basic eye examination. Specifically, he/she should be able to 1. discuss the value and rationale of the various parts of the basic eye examination 2. determine a patient’s visual acuity 3. assess the pupillary reflexes 4. evaluate ocular motility 5. determine intraocular pressure 6. perform direct ophthalmoscopy for a systematic fundus examination 7. record the results of the eye examination properly and accurately

PREREQUISITE KNOWLEDGE AND PREPARATION Students should have a working knowledge of the basic anatomy of the eye and its adnexa. It is advised that this written material be completed first prior to performance of the eye examination on an actual patient. The students should likewise familiarize themselves with the basic eye instruments utilized in examining the eye. These include distance and near vision eye charts, penlight, tonometer and the direct ophthalmoscope.

INTENDED USERS Although this material was developed to provide the medical student with the principles of each area of the eye examination, this should be supplemented by small group sessions directed at actual performance of these skills.

CONTENT All patients should have an eye examination as part of a general physical examination. Visual acuity, gross examination of the eye and its adnexae, extraocular muscle

38

BASIC EYE EXAMINATION / 39

movements, intraocular pressure determination and fundus examination using the direct ophthalmoscope constitute the basic eye examination. These will be discussed individually. VISUAL ACUITY TESTING Measurement of the visual acuity provides clinicians with a standard tool for reporting and recording a patient’s vision. Standard notations used for recording of visual acuity are shown in Table 1. Table 1. Notations used in Recording Visual Acuity VA OD OS OU sc cc ph NV

visual acuity (oculus dexter) Right eye (oculus sinister) Left eye (oculus uterique) Both eyes without correction with correction pinhole near vision

DISTANCE VISUAL ACUITY Distance visual acuity measurement should be performed in all patients, including children because of the importance of early detection of amblyopia. Determination of visual acuity is done prior to any manipulation of the eye to avoid any medico-legal issues that may arise in the future. Distance visual acuity is recorded as a ratio or fraction which compares the performance of the patient with an agreed upon standard. VA =

distance from the patient to the chart __ distance at which normal eye can read the given line Example: VA = 20/40 indicates that the patient can recognize at 20 ft, a symbol that can be recognized by a person with normal visual acuity at 40 ft.

Visual acuity of 20/20 represents normal vision. Alternative notations are shown in Table 2.

Fig. 1 Snellen Letter

Fig. 2 Pocket Near Vision Chart

39

BASIC EYE EXAMINATION / 40

Table 2. Alternative Notations for Recording Visual Acuity English System Notation Metric System Notation Decimal Notation 20/200 6/60 0.1 20/100 6/30 0.2 20/70 6/21 0.3 20/50 6/15 0.4 20/40 6/12 0.5 20/30 6/9 0.7 20/25 6/7.5 0.8 20/20 6/6 1.0 Visual Acuity is generally tested using the Snellen Chart (Figure 1) - letter, number, illiterate E, picture charts;) at a distance of 6 meters or 20 feet (which is equated with optical infinity). The general steps are as follows: 1. Place patient at the designated distance of 20 feet or 6 meters from a well illuminated Snellen Chart. If the patient has corrective lenses, ask the patient to wear them during the test. 2. By convention, the right eye is tested and recorded first. Have the patient occlude his left eye using an opaque occluder. The palm of the patient’s hand may also be used to occlude the vision in the right eye. 3. Ask the patient to read the chart starting at the 20/200 line proceeding to the smallest line which he/she can distinguish more than half of the letters. 4. Record the acuity measurement by jotting down the numeric designation of the smallest line that the patient was able to read. 5. Occlude the patient’s right eye and repeat steps 3 and 4. 6. If the patient’s visual acuity is less than 20/20 in one or both eyes, repeat the test with the patient viewing the test chart through a pinhole occluder and record these results. If the patient cannot see the largest Snellen letter, proceed as follows: 1. Reduce the distance between the patient and the chart until he/she is able to read the 20/200 line. Record this new distance as the numerator of the acuity designation while retaining the denominator. For example, if the patient is able to read the 6/60 (20/200) line at a distance of four meters, the vision is recorded as 4/60. 2. If the patient is unable to see the largest Snellen letter even at a distance of one meter or 3 feet, hold up one hand and ask the patient to count the number of extended fingers. Record the distance at which counting fingers is done accurately. For example, if a patient can count fingers at a distance of ½ meter, visual acuity is recorded as CF at ½ meter. 3. If the patient cannot count fingers, determine whether or not he/she can detect the movement of your hand. Record a positive response as hand motion designated as HM. 4. If the patient can detect hand motion, use a penlight to determine if the direction of the source of the light can be correctly detected by the patient. Shine the light on four quadrants. Record your findings as follows: good LPj

able to identify light source in all four quadrants

40

BASIC EYE EXAMINATION / 41

fair LPj poor LPj

able to identify light source from 2-3 quadrants able to identify light source only from one quadrant

5. If the patient is unable to correctly identify the direction of the source of the light but is able to detect its presence, record the patient’s response as light perception (LP). If the presence of light can not be detected by the patient, this is recorded as No Light Perception (NLP) Differences in acuity can often be due to refractive error. Improvement of the patient’s vision using a pinhole would imply that the probably has an error of refraction. In some instances, as in infants or toddlers, one will not be able to utilize these standard charts in determining visual acuity. In such instances, the examiner should be alert to other signs. Withdrawal or a change in facial expression in response to light or sudden movement indicates the presence of vision. NEAR VISUAL ACUITY Near visual acuity testing is routinely done for patients over 35 years of age. Otherwise, testing “at near” is done if the patient complaints about their near vision. It is also sometimes done for instances when distance testing is difficult or impossible (at patient’s bedside). Unlike distance vision testing, near vision is tested with both eyes open. The standard near vision chart (Figure 2) is held at a distance of 14 inches or 35 cm. If the patient normally wears glasses for reading, he should wear them during testing. Since letter size designations and test distances vary, both size and distance should be recorded (ex. J5 at 14 inches, 6 pt at 35 cm). If a standard near vision card is not available, any printed material such as a telephone book or a newspaper may be substituted. Both the approximate type size read and the distance at which the material was held are recorded. GROSS EXAMINATION OF THE EYE AND ADNEXAE EXTERNAL INSPECTION With adequate illumination, the examiner can inspect the lids, surrounding tissues and palpebral fissure. The presence of redness or any mass should be noted and recorded. Palpation of the orbital rim and lids may also be indicated based on the patient’s history and symptoms. The position of the eyes should be examined from in front, from above, (looking down over the patients brow while seated), and from the side. These views would highlight any possible protrusion of the eye ball. Observe the eyes from the front, look at the position of the eyelids relative to the pupil. If the white of the sclera can be seen all around the iris then this could be due to

41

BASIC EYE EXAMINATION / 42

exophthalmos or lid retraction. On the other hand if the lids are encroaching on the iris and pupil, then this could be due to enophthalmos or ptosis. With the aid of a penlight, inspection of the conjunctiva and sclera is done to note the presence of any abnormality. Instruct the patient to look up while retracting the lower lid or look down while raising the upper lid to note the presence of redness, discharge or any other abnormalities. The penlight also aids in the inspection of both the cornea and the iris. PUPILLARY REACTION TESTING This includes testing the patient’s direct and consensual pupillary reactions to light. Direct Pupillary Reaction. To test the pupillary reaction to light, first direct the penlight at the patient’s right eye and note if it constricts. Constriction is a normal reaction. Repeat the procedure on the left eye to test the left pupil. Consensual Pupillary Reaction. To test the consensual pupillary reaction to light, direct the penlight at the right eye and watch the left pupil to see if it constricts along with the right pupil. The presence of constriction is the normal consensual response. Repeat the procedure for the left pupil, watching the right pupil for the response. Frequently, pupillary inspection reveals active or prior eye disease with alterations in pupillary shape or size due to damage to the pupillary sphincter or adhesions between the iris and the lens. OCULAR MOTILITY TESTING Examining the eye movements begins by observing the eyes in the primary position (i.e. looking straight ahead). Shine a penlight at the eyes and ask the patient to look at it. Observe whether the reflection of light is centered in the middle of the pupils. With both eyes open, instruct the patient to follow your finger or a small target through the six cardinal positions of gaze (Figure 3). Ask the patient to tell you if double vision is noted at any point. Also observe the eye if there is any limitation of movement or nystagmus. When the EOM movement is tested with both eyes open, this is referred to as Version Test. Repeat the same examination one eye at a time, Duction Test. Use your finger and move it SLOWLY through the different cardinal positions shown below, keeping it roughly 30 cm from the patient. This will enable the examiner to systematically test each muscle in the primary field of action. Thus, a possible isolated weakness or paralysis of a muscle can be detected. A nerve palsy or muscle weakness can alter the movements of the eyes when going through the six cardinal positions. There may well be limitations in certain directions, and double vision associated with looking in that direction.

Fig. 3 Six Cardinal Directions of Gaze

42

BASIC EYE EXAMINATION / 43

Table 3. Cardinal Fields of Gaze Right and Up Left and Up Right superior rectus Left superior rectus Left inferior oblique Right inferior oblique Right Left Right lateral rectus Left lateral rectus Left medial rectus Right medial rectus Right and Down Left and Down Right inferior rectus Left inferior rectus Left superior oblique Right superior oblique Common abnormalities in the alignment of the eyes are shown in the figure below.

NORMAL LEFT ESOTROPIA LEFT EXOTROPIA

LEFT HYPERTROPIA LEFT HYPOTROPIA Fig. 4 Common abnormalities in the alignment of the eyes

Findings are recorded as follows:

SR

SR

IO MR

LR

LR

SO

IR OD

OS

IR

Fig. 5 Recording of results of Ocular Motility Testing SR – superior rectus, LR – lateral rectus, IR – inferior rectus, IO – inferior oblique, MR – medial rectus, SO – superior oblique, OD – right eye, OS – left eye

INTRAOCULAR PRESSURE DETERMINATION This is the pressure that is created within the closed environment of the eye. The pressure is governed by a balance between the production of aqueous humor and its drainage. The normal flow of aqueous is illustrated in Fig. 6 .

43

BASIC EYE EXAMINATION / 44 Ciliary body

Iris POSTERIOR CHAMBER ANTERIOR CHAMBER

Lens

Fig. 6 Normal Aqueous Flow

Intraocular pressures (IOP) vary from individual to individual and fluctuate within a normal range. A rise in this pressure or a person sensitive to certain pressures can be transmitted to the back of the eye and damage the optic nerve. This with characteristic visual field changes and optic disc cupping is known as Glaucoma. The average IOP in a “normal” population is 15 mm Hg, however IOPs in the range of 10 to 21 mm Hg are still considered to be within the normal range. Pressures above this are often treated, but it is important to look at the retina and look for changes due to high pressure, as people with normal pressures may have eyes that are sensitive to "normal range" pressure, and will still produce changes characteristic of glaucoma. This is called "low pressure glaucoma". Intraocular pressure is measured by tonometry. A nu mber of instruments may be used for this purpose - Goldmann Applanation tonometer, the tonopen, Schiotz tonometer and non-contact tonometers. Goldman tonometry is most commonly used and is considered the “gold standard”. Its primary disadvantage, however, is that it requires special equipment and can only be used by an ophthalmologist. Furthermore, the instrument can not be used in individuals with corneal abnormalities such as scars. One relatively simple method of measuring IOP, and is generally used for screening purposes, is the Schiotz, or indentation tonometer (Fig. 7). The Schiotz tonometer with a given weight is placed on the anesthetized cornea. The instrument indents the cornea in an amount related to the IOP. The scale on the Schiotz tonometer indicates the amount of actual indentation. A printed conversion table that comes with the instrument is used to determine the IOP in millimeters of mercury from the indentation scale reading.

B

A

Fig. 7 A- Applanation tonometer B- Schiotz

In the absence of any instrument, however, IOP can be estimated by palpation. This is done by carefully applying pressure using your forefingers on the upper lid while the patient is looking down. Palpating a “soft” eye would be the same as palpating the tip of the nose. An eye perceived to be softer than these is considered to be “hypotonic”. If the eye is palpated to be hard on palpation, this is reported as “firm” and the patient is suspected to have an elevated IOP.

44

BASIC EYE EXAMINATION / 45

FUNDUS EXAMINATION The fundus can be examined by direct ophthalmoscopy using an ophthalmoscope, or by indirect (often binocular) methods such as an indirect ophtalmoscope (Figure 8). The main differences between the direct and indirect ophthalmoscope are listed in Table 4. It is essential that every physician learns and gain confidence in using an ophthalmoscope to perform direct ophthalmoscopy.

B

A

Fig 8. A – direct ophthalmoscope B – indirect ophthalmoscope

Table 4. List of Differences between the Direct and Indirect Ophthalmoscope Criteria Direct Ophthalmoscope Indirect Ophthalmoscope image upright inverted magnification larger smaller area viewed smaller larger depth absent present Before beginning, ensure that the ophthalmoscope is working properly: 1. 2. 3. 4.

Check the light source. Make sure that the light source is the largest circle. Adjust the lens setting to 0. When examining the patient’s right eye, hold the direct ophthalmoscope with your right hand and use your right eye to view the patient’s eye. Use the left hand and left eye to examine the patient’s left eye. 5. The patient’s glasses are removed, and, barring large astigmatic refractive errors, examiners prefer to remove their own glasses as well. Contact lenses worn by either the patient or the examiner may however be left in place.

To perform direct ophthalmoscopy, follow these steps. 1. The room should be dimly-lit and the patient comfortably seated. 2. The patient should be instructed to focus on a distant target. The patient should also be instructed to maintain their gaze throughout the examination. 3. Set the focusing wheel at 0 and select the large, round, white light. 4. Begin to look at the right eye about 1 foot from the patient. Use your right eye with the ophthalmoscope in your right hand. When you look straight down the patient’s line of sight at the pupil, you will see the red reflex. 5. You may place your free hand on the patient’s lid to keep the eye open, or on the patient’s forehead or shoulder to keep yourself steady. Hold the ophthalmoscope comfortably against the arch of your brow.

45

BASIC EYE EXAMINATION / 46

6. Slowly come closer to the patient at an angle of about 15° temporal to the patient’s line of sight. Try to keep the pupil in view at all times. Turn the focusing wheel with your index finger to bring the patient’s retina into focus. 7. When a retinal vessel comes into view, follow it as it widens to the optic disc, which lies nasal to the center of the retina. 8. Examine the optic disc, retinal blood vessels, retinal background and macula. 9. Repeat the same procedure for the left eye. RED REFLEX Light reflected off the fundus of the patient produces a red reflex when viewed through the ophthalmoscope at a distance of about one foot. A normal red reflex (Fig. 9) is evenly colored and is not interrupted by shadows. If there is any opacity in the lens or cornea this will appear black or create a silhouette against the red-reflex.

A

B

Fig. 9 A- normal red reflex B – red reflex obstructed by opacity

OPTIC DISC In most cases, when viewed through the ophthalmoscope, the normal optic disc (Fig 10) will appear slightly oval vertically and pink in color. A central depression in the surface of the disc is called the “physiologic cup”. The optic disc is often used as a “yardstick” of the ocular fundus. Lesions seen with the ophthalmoscope are measured and described in terms of disc diameters. Examine the disc as follows: 1. Look at its margins and note whether they are well defined. Also note for the presence of any changes such as pigmentation around the disc margins. 2. Look at the cup: disc ratio and note whether the cup takes up a large part of the disc. 3. Note for the presence of any hemorrhages within the disc. There are numerous variations in the appearance of the disc. Some common variations are shown in Figure 11.

46

BASIC EYE EXAMINATION / 47

A

Fig 11 A- Retinal and choroidal pigment

B

Fov

Op

Mac

Fig. 12

RETINAL CIRCULATION The retinal circulation is composed of arteries and veins, visible with the ophthalmoscope. Figure 12 shows a picture of a normal fundus together with a schematic representation. The branches of the central retinal artery supplying the four quadrants of the inner retina lie superficially in the nerve fiber layer. The retinal veins collects at the optic disc and follows a similar system of arrangement. The normal artery-to-vein diameter ratio is 2:3. Arteries usually appear lighter in color than the veins and have a more prominent light reflex from their surface. Aside from noting the caliber of the vessels and the light reflexes, the examiner should also note the

47

BASIC EYE EXAMINATION / 48

arteriovenous crossing patterns. The examinee should also note the presence of hemorrhages and exudates.

exudates

hemorrhages

A

B

C

D

E

F

Fig 13. Retinal Changes A – blot hemorrhages B – increased vessel tortuosity C – pigment changes and boat shaped hemorrhage D – boat shaped hemorrhage E – flame shaped hemorrhages F – arterial beading G – pigment changes

G

48

BASIC EYE EXAMINATION /49

RETINAL BACKGROUND The normal retinal background is a uniform red-orange color. The retinal pigment epithelium, blood and pigment of the choroid contribute to the appearance of the retinal background. One condition which changes this color is retinal detachment which produces a dull, grayish appearance (Fig. 14).

Fig. 14 Detached Retina

MACULA The normal macula is located temporal to the optic disc (Fig. 12) and appears darker than the surrounding retina. This is due to the specialized retinal pigment epithelial cells of the macula that are taller and more heavily pigmented. The central depression of the fovea may act as a convex mirror and produce a light reflection known as the foveal reflex. The foveal reflex should be noted by the examiner. REPORTING OF FINDINGS Findings on fundus examination are typically reported in a systematic manner, as follows: STRUCTURE Red Orange Reflex (ROR) Media Optic Disc • disc margins • cup:disc ratio Retinal Vessels • A:V ratio • median light reflex • AV crossing defects • hemorrhages and exudates Retinal background Macula • foveal reflex

NORMAL present clear

ABNORMAL absent hazy

distinct 0.3 – 0.5

indistinct, blurred > 0.5

2:3 normal absent absent red orange

any other ratio widened present present gray, pale

present

dull or absent

49

BASIC EYE EXAMINATION / 50

SUMMARY Acquiring the skills to be able to properly perform the basic eye examination will allow physicians to recognize potentially vision threatening conditions early so that such cases are referred to the ophthalmologist for appropriate management. Below is a summary of the steps in performing the basic eye exam: 1. 2. 3. 4. 5. 6. 7. 8. 9.

Measure the visual acuity of each eye. Inspect the lids and surrounding tissues. Palpate the orbit if necessary. Inspect the conjunctiva, sclera, cornea and iris. Inspect the pupil and check the pupillary reflexes. Assess the chamber for clarity and depth. Check the ocular alignment and test the extraocular movements. Perform tonometry. Assess the lens for clarity through direct ophthalmoscopy. Examine the fundus, disc, vessels, retinal background and macula.

The following is an example of the results of an eye exam:

IOP OD = 14 mm OS= 16 mm Funduscopy: OU – (+) ROR, clear media, distinct disc borders, CD=0.4, AV= 1:3, (-) hges/exudates, good foveal reflex REFERENCES 1. Berson, Frank G. Basic Ophthalmology for Medical Students and Primary Care Residents. 6th ed. San Francisco: American Academy of Ophthalmology; 1993. Chapter 1, “The Eye Examination”. 2. Newell FW: Ophthalmology: Principles and Concepts. 7th ed. St. Louis, Mosby Co; 1992. 3. Pavan-Langston, D. Manual of Ocular Diagnosis and Therapy, 3rd ed. Boston: Little, Brown & Co; 1991. Chapter 1, “Ocular Examination Techniques and Diagnostic Tests”. 4. Vaughn DG, Asbury T, Riordan-Eva P. General Ophthalmology, 13th ed. Norwalk, CT: Appleton & Lange; 1992.

50

BASIC EYE EXAMINATION / 51

SELF–TEST 1. Given the following choices, arrange them starting from the procedure that should be done in someone with near normal vision, ending with someone who is nearly blind. _____ A. Snellen chart at 6 meters _____ B. perceives light _____ C. finger counting _____ D. Snellen chart at 1 meter _____ E. hand motion 2. If a patient’s visual acuity is 6/21, this would mean that the person can read A. at 6 meters what a normal person sees at 21 meters B. at 21 meters what a normal person sees at 6 meters 3. Proptosis can be seen by looking at a patient from the following positions, EXCEPT A. back B. side C. front D. top 4. A patient’s left eye constricts upon shining your penlight directly on it. Shining a light on the patient’s right eye also causes constriction of the left pupil. These findings indicate that the patient has A. functioning left and right optic nerves B. intact right oculomotor nerve C. non-functioning right optic nerve D. none of the above 5. The patient in this picture is looking to her left. She has a problem of the A. R medial rectus B. L medial rectus C. R lateral rectus D. L lateral rectus 6. In examining a patient’s right eye, the examiner should hold his ophthalmoscope with his A. right hand and use his left eye B. left hand and use his right eye C. right hand and use his right eye D. left hand and use his left eye. 7. If a patient has a corneal leukoma, the best instrument to use that will give more accurate IOP reading is A. Applanation tonometer B. Schiotz tonometer C. digital tonometer D. any one of the above 8-9. On funduscopy, these were the disc findings. Use the following choices for your answers to items 4 and 5. A. 0.9 B. 0.5 C. 0.3 D. 1.0 8. What is the cup:disc ratio for the left eye? 51

BASIC EYE EXAMINATION / 52

9. What is the cup:disc ratio for the right eye? 10. The following are characteristics of the Direct Ophthalmoscope EXCEPT A. provides a larger image than the indirect ophthalomoscope B. provides an upright image of the retina C. it gives the examiner a better view of the peripheral retinal structures D. none of the above ANSWERS TO SELF-TEST 1. 2. 3. 4. 5. 6. 7. 8. 9. 10.

A – 1, B – 5, C – 3, D -2, E – 4 A A A D C B A B C

52

DISORDERS OF THE CORNEA / 53

DISTURBANCE IN VISION DISORDERS OF THE CORNEA Ruben Lim Bon Siong, MD INTRODUCTION This self instructional material is designed for undergraduate medical students (Learning Unit IV of the UP College of Medicine). This material is intended to serve as supplementary reading for the students as part of the unit on DISTURBANCES IN VISION and focuses primarily on providing the learner the knowledge in understanding how corneal pathologies affect vision and in knowing the common corneal diseases or problems that can affect vision. Aside from providing the learner with text on the subject matter, pictures of typical cases will also be presented. Students are however encouraged to apply knowledge that they will acquire from this SIM to actual and simulated clinical cases during their rotation. OBJECTIVES After going through this material, the learner is expected to: 1. Explain how corneal pathologies and diseases affect vision. 2. Discuss the common corneal diseases that affect vision based their basic pathophysiology 3. Discuss the principles on how these corneal diseases are managed. RECOMMENDED PREPARATION You are advised to review the anatomy and physiology of the cornea and principles and definitions of basic physical optics before going through this SIM. INTENDED USERS This SIM was primarily designed for LEARNING UNIT IV students of the UP College of Medicine. These students are expected to have completed basic units in ophthalmology, namely Ophthalmic Anatomy and Physiology, Ophthalmic Optics and Objective and Subjective Examination of the Eye. CONTENT The eye is basically an optical instrument wherein light reflected from an object are transmitted and refracted so that the image is focused on the retina. Light is then converted to photo-electrical signals which are transmitted to the occipital cortex and is perceived as vision. The two main refracting components of the eye are the cornea and the lens. Of the two, the cornea, together with the pre corneal tear film is responsible for two-thirds of the total refractive power of the eye thus making it more powerful than the lens. Main Functions of the Cornea 1. Transmission of Light: The cornea is a highly transparent tissue and this is largely due to its unique anatomy. It is devoid of blood vessels and has a paucity of cellular elements. It is composed of collagen fibrils with uniform diameter and uniform spacing and arrangement. In between the collagen fibrils are the extracellular matrix ground substance composed of proteoglycans (PG) with a constant water content of 78% which is a lot less than the water content of other tissues of the body. This relative dryness or deturgescence is the one responsible for maintaining the homogeneity of the collagen fibrils. Since the diameter of the collagen fibrils and the distance between them is less that ½ the wavelength of

53

DISORDERS OF THE CORNEA / 54

visible light, light is not diffracted and is therefore transmitted (Fig 1). Any disruption of the normal configuration of the collagen fibrils will therefore cause failure in the efficient transmission of light and the cornea will appear hazy or opaque. 2. Refraction of Light: In order for light entering the eye to be focused on the retina, incoming parallel light rays have to be bent or refracted. This is made possible by the smooth anterior shape/curvature of the cornea which is convex (thus light is bent inward or converged) and by the difference in the indices of refraction between air, tear film, cornea and aqueous humor (Fig 2). So any disturbance in the shape/curvature of the cornea and the indices of refraction of the different structures can cause the failure of the converged light rays to be focused on the retina leading to blurry vision. Common Causes of Blurring of Vision due to Corneal Pathology or Diseases Conceptual Framework: The foregoing discussion will be divided into the major headings based on how certain corneal disorders affect the two main functions of the cornea as mentioned above. The aim is to make the learner appreciate the close relationship between the structure and function of the cornea and to know the common etiologies that can affect each function. It is not the main aim of this material to make the learner adept in diagnosing these conditions since specialized instruments and tests, which are not available to the learner, are usually required to make an accurate diagnosis. Outline: A. Disruption in the transmission of light 1. Corneal scars 2. Corneal edema 3. Corneal deposits 4. Corneal melt 5. Corneal tumors B. Disturbance in the refraction of light 1. Abnormalities of the corneal epithelium and tear film 2. Abnormalities of corneal size/ shape / curvature A. Disruption in the Transmission of Light: Corneal Scars Corneal scars are usually tan to white in color and may involve different areas of the cornea and may come in different shapes and sizes depending on the etiology and pathology. Scars are usually formed after an inflammatory process when fibrosis sets in. In fibrosis, new and abnormal collagen fibrils and other cellular elements are laid down in a “haphazard” manner causing the disruption of the normal homogenous arrangement of the collagen fibrils. This prevents light to be transmitted properly and is reflected back thus making the cornea appear opaque (Fig 3). Etiologies: 1. Microbial keratitis: these are infections of the cornea which can be caused by viruses, bacteria, fungi and protozoans. Invasion of microorganisms into the cornea induces an intense inflammatory response and with the actions of microbial toxins and enzymes causes corneal tissue necrosis, melting and sometimes rupture of the cornea. During the active infection, corneal opacity is associated with marked eye redness, tearing and photophobia (Fig 4). Once healing ensues, either due to treatment or the natural course of the disease, redness and other symptoms will disappear except for blurring of vision (more so if lesion is overlying or affecting the pupillary area) which is caused by the corneal opacity.

54

DISORDERS OF THE CORNEA / 55

2.

Corneal trauma: this includes mechanical and chemical njuries to the cornea and is among the major causes of corneal problems in the Philippines. Corneal lacerations and perforations are usually caused by sharp objects (knife, scissors, nails, glass etc) and high velocity projectiles (iron nails due to hammering, shattered glass, darts, bullets etc). Most of these injuries have to be managed surgically by doing corneal repair/suturing to close the wound and restore the integrity of the globe (Fig 5). Similar to skin lacerations, healing will bring about fibrosis and subsequent corneal scarring along the wound and suture track. Depending on the extent of involvement, vision will be variably affected either by the opacity itself or the distortion of the shape of the cornea. Acid chemical burns on the cornea causes denaturation and precipitation of the collagen and will lead to scar formation. In alkali burn, corneal destruction is generally more severe since it causes corneal necrosis and melting due to its ability to penetrate deeply into the cornea. Contact burns from molten metal and other heated materials can also cause severe corneal injuries and scarring.

3. Exposure keratopathy: the cornea is prone to desiccation if left exposed to the environment. Failure of the lids to close properly (lagophthalmos) will lead to a condition called exposure keratopathy wherein the exposed cornea (usually the inferior half ) will opacify due to the drying effect (Fig 6). This condition can also lead to severe scarring if secondary microbial keratitis occurs. This problem is usually seen in comatose patients, those with CN VII palsies, lid and orbital deformities, acute proptosis, nocturnal exposure etc. Bilateral inferior corneal scars are usually seen in patients with a history of severe measles occurring in childhood. This usually starts as exposure keratitis in a very sick child compounded by dehydration, pneumonia, malnutrition, immunodeficiency, vitamin A deficiency and lack of tears. The exposed cornea develops secondary bacterial infection and usually ends in corneal rupture and subsequent scarring. It should be noted that exposure keratopathy can be prevented. 4. Lid margin and lash disorders: Abnormalities of the lid margins may cause misdirection of the eye lashes towards the cornea and the constant and chronic rubbing will lead to surface vascularization and eventual scarring. This condition is seen in trichiasis, trachoma, chronic blepharitis, epiblepharon, entropion, sequelae of Stevens Johnson Syndrome etc. 5. Congenital corneal scars: corneal scars can develop while in utero due to anomalies during embryogenesis. Peters anomaly is part of a spectrum of disorders known as anterior segment dysgenesis and presents as central corneal leukoma with defects in the posterior stroma, Descemet’s membrane and the corneal endothelium. Strands of iris may attach to the posterior border of the leukoma. This condition is bilateral in 80% of cases and more than half has glaucoma. Corneal Edema The corneal endothelium is the single most important structure of the cornea and is the one responsible in active pumping of water away from the cornea to the aqueous humor thereby maintaining corneal clarity. The human corneal endothelial layer does not regenerate when lost or injured and decreases in number as a person ages. An adequate number must be present throughout life in order for it to function properly. When the cell density dips below a critical level, the direction of the flow of water is reversed and the cornea will retain water and swell like a sponge (corneal decompensation) causing disruption of the normal arrangement of the collagen fibrils. The edematous and markedly thickened cornea will appear hazy or ground glass in appearance. Affectation may be diffuse or focal depending on extent of involvement. Patient will usually complain of marked blurring of vision and recurrent eye pain, redness, foreign body sensation and tearing.

55

DISORDERS OF THE CORNEA / 56

Etiologies: 1. Congenital or Hereditary: Infants with bilateral diffuse corneal haziness and ground glass appearance, thickened cornea with normal corneal diameter and eye pressure and absence of birth trauma should be suspected to have Congenital Hereditary Endothelial Dystrophy (Fig 7). Fuchs’ Endothelial Dystrophy, on the other hand, is seen usually after age 50, more so in females. These patients have an abnormally fast rate of endothelial cell loss as compared to the normal population. All cases will initially present with central corneal guttata located at the posterior side of the cornea. The corneal guttata will increase in number and spreads toward the periphery. Total endothelial cell density will decrease and corneal decompensation may ensue in due time. 2. Post Surgical: Anterior segment intraocular surgery such as cataract extraction can cause trauma to the corneal endothelium. Trauma can be direct in the form of mechanical contact of instrument or lens implants or intraocular structures like vitreous to the corneal endothelium. It may be indirect like inadvertent introduction of noxious chemicals into the eye (Fig 8). 3. Chronic Increased Eye Pressure (Glaucoma): Abnormally high eye pressure can affect corneal metabolism and damage the corneal endothelium by causing relative hypoxia and hypoglycemia in the anterior chamber of the eye. Since the cornea is avascular, the corneal endothelium depends on the aqueous humor for its nutrients and oxygen supply. Corneal decompensation are seen in both congenital glaucoma in children and chronic glaucoma in adults. Corneal Deposits Aside from it acting like a sponge, the cornea is also like a sieve. Substances can get trapped within the corneal collagen lamellae or within intracytoplasmic vacuoles or organelles and cause opacities that will lead to blurring of vision. Etiologies: 1. Corneal Dystrophy: this is a large group of hereditary corneal disorders sharing several common features like bilateral and symmetrical involvement, absence of any form of inflammation, absence of vascularization, and with proven or suspected chromosomal abnormalities. Chromosomal defects cause problems in normal metabolism leading to deposition of substances at different layers of the cornea. Diagnosis is based on clinical presentation and by histopathology. Each form affects vision differently primarily due to difference in appearance and location. (Fig 9) 2. Lipid Keratopathy: this is usually seen in vascularized corneal scars of various etiologies (trauma, infection, immune-mediated). The invasion of blood vessels into the cornea will lead to leakage of glycoproteins, cholesterol and neutral fat into the cornea and to eventual deposition of these substances. The opacities are usually yellow or cream-colored located at the corneal stromal layer typically associated with a branch or several branches of blood vessels at the core. (Fig 10) 3. Calcific Band Keratopathy: this is usually seen in eyes with chronic inflammation like anterior uveitis and in patients with high serum calcium and disorders in phosphate metabolism. Calcium hydroxyapatite deposits at the Bowman’s layer starting at the 3 and 9 o’clock area and meeting at the center over time forming a white horizontal band across the cornea with a Swiss cheese pattern. (Fig 11) 4. Corneal Staining: hyphema (blood in the anterior chamber) coupled with high eye pressure will lead to deposition of heme in the corneal stroma. This appears as a central golden brown to yellow discoid opacity on the cornea. (Fig 12) 5. Metabolic disorders: systemic metabolic disorders can cause alterations in corneal clarity due to abnormal storage of metabolic substances within the epithelium, stroma or endothelium. Abnormal

56

DISORDERS OF THE CORNEA / 57

substances typically accumulate in lysosomes or lysosome-like intracytoplasmic structures as a result of single enzyme defect. Most of these conditions are rare and are autosomal recessive. Examples are systemic mucopolysaccharidoses, hyper and hypolipoproteinemias, sphingolipidoses, mucolipidoses, and an array of protein and amino acid metabolic disorders. Corneal Melt Apart from microbial keratitis, corneal melt or necrosis can be brought about by Vitamin A deficiency, chemical burn and autoimmmune diseases. If the process is arrested, usually by prompt and appropriate therapy, then healing and fibrosis will take place leaving the cornea scarred and opacified. Corneal melting is brought about by a complex interaction and cascade of enzymatic and molecular events involving a myriad of substances and is beyond the scope of this material. (Fig 13) Etiologies: 1. Vitamin A Deficiency: this leads to xerophthalmia, which is responsible for at least 20,000 – 10,000 new cases of blindness worldwide each year. At greatest risk are the malnourished infants and the baby born to a Vitamin A-deficient mother, especially if the infant has another biological stress such as measles or diarrhea. Prolonged vitamin A deficiency leads to external eye involvement, including xerosis (dryness of the conjunctiva and cornea), metaplastic keratinization of areas of the conjunctiva (Bitot spots), corneal ulcers and scars and eventually diffuse corneal necrosis (keratomalacia). 2. Alkali Chemical Burn: strong alkali raise the pH of tissues and cause saponification of fatty acids in cell membranes and ultimately cellular disruption. Alkaline solutions rapidly penetrate the corneal stroma destroying the proteoglycan ground substance and collagen fibers. It also destroys the limbal stem cells preventing normal epithelial healing. Severe scarring and corneal vascularization are seen in severe cases. (Fig 14) Corneal Tumors New growths, either benign or malignant can occur on the ocular surface involving the cornea and conjunctiva. Vision may or may not be affected depending on the location of the lesion whether it will distract the transmission and refraction of light. These lesions are easily diagnosed by their clinical presentation and appearance and is confirmed by histopathology. Etiologies: 1. Dermoid Choristoma: this congenital lesion typically occurs on the inferotemporal limbus as a smooth, elevated, tan to fleshy color, round to oval solid mass embedded in the superficial cornea and sclera. Dermoids are composed of fibrous tissue and hair with sebaceous glands that is covered by conjunctival epithelium. Epibulbar dermoids are located over the central cornea and can severely affect vision. (Fig 15 L) 2. Corneal Intraepithelial Neoplasia: suspected to be caused by the human papillomavirus, lesion appears as translucent, gray or frosted epithelial sheet starting from the limbus and extending onto the cornea with fimbriated or scalloped borders and pseudopodialike extensions. Blurring of vision will occur once the growing epithelial sheet reaches the central area. (Fig 15 C) 3. Pterygium: Strictly speaking, this condition is primarily a conjunctival disorder but due to its intimate relationship with the cornea, it will be included here. Pterygium is a benign conjunctival lesion that behaves malignantly. It is a wing-shaped or triangular fold of conjunctiva and fibrovascular tissue with its apex invading the superficial cornea. (Fig 15 R) Strong correlation with UV exposure has been documented. It affects vision in two ways. One, it grows progressively towards the center of the cornea

57

DISORDERS OF THE CORNEA / 58

and covers the pupil area. And two, it pulls on the peripheral cornea and cause distortion of the shape resulting in astigmatism. B. Disturbance in the Refraction of Light Abnormalities in the Corneal Epithelium and Tear Film The main refractive surface of the eye is the anterior cornea. An important prerequisite for any refractive surface is that it should be smooth. Presence of cracks, smudges and the like will cause degradation in the quality of the image. The outermost layer of the corneal epithelium per se is not smooth. When seen under scanning electron microscopy, the surface is thrown into dense microvilli. To make the corneal surface a perfect refractive surface, it is intimately related to the tear film, which coats the anterior surface of the cornea. Additionally, the tear film has a high index of refraction when compared to air. This physical attribute allows further convergence of light rays as it passes from air then through the tear film and the cornea. Thus, any abnormalities of the tear film and/or the corneal surface epithelium, especially over the central area overlying the pupil can cause blurring of vision. Etiologies: 1. Deficiency in Tear Volume: Dry eye syndrome is a common disorder with different etiologies wherein there is decreased tear production. Most cases are mild and just cause occasional symptoms. In severe and chronic cases however, it can cause irregularities on the corneal surface due to microtrauma to the epithelium. (Fig 16) Patient complains of dryness, foreign body sensation, burning sensation and blurring of vision. This condition is also known as keratoconjunctivitis sicca. 2. Tear Quality Abnormalities: the tear film is actually a complex structure composed of three main components: lipid, aqueous and mucin. The lipid prevents rapid tear evaporation and mucin allows molecular interaction between the tear film the corneal surface (surface tension). If one or both of these are abnormal, despite adequate tear volume, patient will complain of dry eye symptoms. They will also complain of fluctuating vision due to rapid evaporation and break up of the tear film in between blinking. In some patients, all three components are affected. As in dry eye syndrome, the corneal surface epithelium can also be injured. Mucin deficiency can be caused by vitamin A deficiency, severe dry eye, alkali chemical burn, Stevens-Johnson syndrome (Fig 17), and cicatricial pemphigoid. Lipid deficiency is caused by blepharitis and meibomian gland dysfunction. 3. Toxic Keratitis: this pertains to microtrauma to the corneal surface epithelium secondary to contact to chemicals (such as alcohol based products like hair spray) or topical medications (active ingredient, preservatives or both). Presence of these superficial punctate lesions make the surface irregular and will thus affect vision. Abnormalities of Corneal Size/Shape/Curvature The total refractive power of the eye is around 60 diopters (D) and cornea and tear film contributes 40 D and the rest by the crystalline lens. This 40 D is a result of a constant range of radii of curvature and indices of refraction of the cornea as provided by a normal anatomy. If the radius of curvature of the cornea decreases (steep cornea), light rays will converge more, moving the focal plain in front of the retina resulting in myopia. If the radius of curvature increases (flat cornea), the opposite effect will occur which is hyperopia. If light rays pass through a cornea with variable radii of curvature, light rays will converge on different focal plains. This is known as astigmatism. Myopia, hyperopia and astigmatism are collectively known as Errors of Refraction and will be discussed in another SIM. What will be discussed here are those corneal diseases and disorders that result in severe errors of refraction and therefore severe blurring of vision.

58

DISORDERS OF THE CORNEA / 59

Etiologies: 1. Keratoconus: this is a common disorder where the central or paracentral cornea undergoes progressive thinning and bulging, so that the cornea takes on the shape of a cone. (Fig 18) Hereditary pattern is not prominent. Etiology is unknown and likely multifactorial. Due to the progressive bulging of the cornea, the cornea becomes abnormally and irregularly steep resulting in very severe myopia and astigmatism. It is almost always bilateral but usually asymmetrical. Patient complains of progressive rapid blurring of vision during adolescent years and then stabilizes when patient has attained full growth, although progression can occur at any time. The cornea usually appears grossly clear, except in severe cases where an apical scar is visible, but on profile one may see a focal area of bulging. It is best appreciated when examined using specialized instruments. 2. Corneal distortion due to peripheral scars or lesions: this is to emphasize the fact that you do not need to have direct involvement, like a scar, on the center of the cornea to cause disturbance in vision. This concept becomes easy to understand if one imagines the cornea as a silver mylar balloon. By looking at your reflection at the flat surface of the balloon, you will notice that the image can be distorted by either pulling or pushing the sides of the balloon, because by doing so, you change the curvature of the center. Always remember that the cornea is one whole integral structure. Flattening of one meridian causes a corresponding steepening in the meridian 90 degrees away and vice versa. This effect is seen in pterygium, scars from peripheral corneal perforations, effect of sutures after cataract surgery, peripheral thinning disorders and others. Principles of Management: Corneal scars, as a rule, are permanent. There are no medical therapies available to turn scar tissue to normal corneal tissue. Depending on the location, size and degree of visual involvement, management is geared towards two goals, either improvement of vision or cosmetic or both. To improve vision, management is usually surgical which may range from manual excision to laser removal to corneal transplantation. To improve appearance, corneal scars may be covered with cosmetic contact lenses, dyed by tattooing or by corneal transplantation. (Fig 19) Corneal edema due to endothelial damage or dysfunction is also permanent because the cells do not regenerate. Definitive treatment is by doing a corneal transplantation. The role of medical therapy is to minimize pain and discomfort and to prevent secondary infections. If it is due to high eye pressure, then the therapy should be directed towards lowering the pressure to appropriate levels. Corneal deposits due to systemic metabolic disorders or from corneal dystrophy may recur after a corneal transplant since the underlying condition is not corrected. Corneal transplantation is reserved only for those conditions with significant visual loss. Calcium deposits can be removed by chelation since the location is superficial while deeper ones like lipid and heme cannot be removed. Corneal transplantation may be the only option. In treating corneal melting, the underlying etiology should immediately be addressed. Surgical management is usually done later to treat sequelae, unless it is for an emergency procedure to restore the integrity of the globe. Corneal transplantation is usually done. Corneal masses are usually treated by surgical removal if the indications for surgery are present. Sometimes corneal grafts are also used to restore corneal clarity. Dry eye syndrome is treated by using topical lubricating agents and/or by preserving existing natural tears by preventing tear drainage. With adequate lubrication, the health of the corneal epithelium will be restored.

59

DISORDERS OF THE CORNEA / 60

Moreover, all other possible risk factors that may aggravate tear quality should be removed or minimized. This principle also holds true in treating toxic keratitis. There are several options when treating corneal disorders with abnormal shape or curvature in order to improve vision. Goal is to correct the refractive errors so that the image will be sharply focused on the retina. This is done by employing optical appliances or by surgically altering the cornea so that the shape will be restored to normal. Depending on the indications, choices may include glasses and contact lenses, incisional or laser refractive procedures, or corneal grafting and transplantation. SUMMARY The main function of the cornea is to transmit and refract light. If the normal anatomy of the cornea is altered, it will lead to blurring of vision. Disorders that can disrupt the normal transmission of light are corneal scars, corneal edema, corneal deposits, corneal melt and corneal tumors. Scars are caused by infection, trauma, exposure, vascularization or can be congenital. Corneal edema can be congenital, due to surgical and non surgical trauma or due to chronic glaucoma. Deposits in the cornea can be caused by metabolic products, calcium, hemoglobin, iron, lipids, proteins, amyloid and other amorphous substances. Corneal melting caused by vitamin A deficiency, chemical burn or autoimmune diseases can lead to permanent corneal scarring. Masses on the cornea like dermoid, pterygium and neoplasia can block the transmission of light or alter the shape of the cornea. Disorders of the cornea that can disturb the refraction of light includes dry eye disease, corneal epithelial dysfunction, and corneal disorders that affect its normal size, shape and curvature. Management of these disorders involves the restoration of the cornea’s normal anatomy primarily through corneal tissue transplantation; excision of abnormal growths or tissues; control or removal of etiologic and contributing factors; surgical interventions to improve corneal shape and curvature and use of optical appliances to enhance the transmission and refraction of light into the eye. REFERENCES 1. Krachmer JH, Mannis MJ, Holland EJ. (eds) Cornea Vol 1 to 3. 2. Smolin G, Thoft RA. (eds) The Cornea 3rd ed. SELF- TEST 1. The main refractive component of the eye is: A. tear film B. cornea C. lens D. retina 2. The water content of the normal cornea is: A. 60% B. 67% C. 78% D. 90%

60

DISORDERS OF THE CORNEA / 61

3. The following affects the normal refraction of light into the eye, EXCEPT: A. quality of the corneal surface B. radius of curvature of the cornea C. index of refraction D. color of the iris 4. The major causes of corneal scarring in the Philippines is/are: A. corneal infection and trauma B. anterior segment dysgenesis C. Stevens Johnson Syndrome D. trachoma 5. Which statement about the corneal endothelium is FALSE? A. the human corneal endothelium retains its mitotic activity throughout life B. the corneal endothelium is responsible for actively pumping water out of the corneal stroma C. the corneal endothelium is a monolayer of cells lining the posterior surface of the cornea D. the corneal endothelium can be damaged by intraocular surgery 6. What is the main mechanism of corneal scarring in a patient suffering from measles? A. direct invasion of the cornea by measles virus B. misdirection of eye lashes towards the cornea C. exposure of the cornea due to poor lid closure with secondary bacterial infection D. deposition of blood in the cornea 7. Which substance is usually deposited at the Bowman’s Layer of the cornea? A. iron B. lipid C. amyloid D. calcium 8. Peripheral corneal lesions can cause blurring of vision by: A. blocking the transmission of light B. inducing astigmatism by changing the central corneal curvature C. decreasing tear production D. increasing central corneal thickness 9. Steepening of the cornea caused by keratoconus leads to what form of refractive error? A. myopia and astigmatism B. presbyopia C. hyperopia and astigmastism D. monocular diplopia 10. The following are treatments options on the cornea to improve vision, EXCEPT: A. Hard contact lenses B. corneal transplantation C. excimer laser photoablation D. corneal tattoo

61

DISORDERS OF THE CORNEA / 62

Answer to Self Test: 1. B 2. C 3. D 4. A 5. A

6. 7. 8. 9. 10.

C D B A D

Pictures:

Fig. 1 Theory of Corneal Transparency

Fig. 3 (L) Typical appearance of corneal scar

Fig. 4 (L) Active fungal keratitis

Fig. 2 Radius of curvature the cornea

(R) Hypertrophic scar

(R) Active bacterial keratitis

62

DISORDERS OF THE CORNEA / 63

Fig. 5 (L) Corneal perforating injury

(R) Sutured corneal perforating injury

Fig 6 Exposure keratopathy secondary to lagophthalmos due to CN 7 palsy

Fig. 7 Congenital hereditary endothelial dystrophy (CHED)

Fig. 8 Post cataract extraction corneal edema

Fig. 9 Corneal dystrophy (L) Granular Type 3

( R ) Avelino Stromal

63

DISORDERS OF THE CORNEA / 64

Fig. 10 Lipid keratopathy secondary to herpes simples keratitis

Fig. 11 Calcific band keratopathy

Fig. 12 Corneal staining secondary to heme deposition

Fig. 13 Autoimmune peripheral corneal melting

64

DISORDERS OF THE CORNEA / 65

Fig. 14 Acute chemical burn

Fig. 15 (L) Dermoid Choristoma (C) Corneal and conjunctival neoplasia ( R ) Pterygium

Fig. 16 Dry eye disease with ocular surface damaged (stained with Rose Bengal dye)

Fig. 17 Severe dry eye disease with ocular surface damage due to Stevens-Johnson syndrome

65

DISORDERS OF THE CORNEA / 66

Fig. 18 (L) Keratoconus with acute hydrops ( R)

Slit lamp view of corneal profile in keratoconus

Fig. 19 Clear corneal graft. Post penetrating keratoplasty (full thickness corneal transplant). Note radial 10-0 nylon sutures.

66

CATARACT / 67

DISTURBANCE IN VISION CATARACT Leonardo R. Mangubat, M.D. INTRODUCTION This self-instructional material is designed for undergraduate medical students, particularly LEARNING UNIT IV students of the UP College of Medicine. This material is intended to serve as supplementary reading for the students as part of the unit on Cataract and focuses primarily on providing the medical student with the knowledge and skill in recognizing and assessing patients with cataract. Pictures of typical cases will also be presented. Students are encouraged to apply knowledge that they will acquire from this material to clinical cases that they will encounter during their rotation in the clinics of the department. OBJECTIVES After going through this material, the student is expected to: 1. Define cataract. 2. Identify the elements in a patient’s history and eye examination that leads to the formulation of cataract as a diagnosis. 3. Differentiate the kinds of cataract. 4. Based on information given, be able to analyze and interpret the provided data to make a diagnosis. RECOMMENDED PREPARATION Students are advised to review the anatomy and physiology of the lens before going through this material. You are further advised to review the parts of a clinical history, as well as the general eye examination so as to better appreciate this material. CONTENT What is Cataract? Cataract is defined as any opacification of the lens causing deterioration of Visual Acuity of < 6/9 or worse. Diagnosis of Cataract There are four (4) basic elements that one would need to consider to be able to formulate a diagnosis. 1. History – a comprehensive history is an important component specially the onset and the nature of the chief complaint 2. Ocular Examination – basic eye examination comprises of the following parts should be done on each patient. a. Gross eye examination – Biomicroscopic (Slit lamp) examination b. Visual Acuity testing (with and without correction) c. Intraocular pressure determination d. Movement of Extraocular Muscles e. Funduscopic examination 3. Ancillary examinations – special ophthalmological and laboratory examinations are done as aids in the formulation of the complete diagnosis. 4. Systemic examinations – general systemic examination is done in cases wherein the clinician suspects the lens problem to be part of the presentation of a systemic condition.

67

CATARACT / 68

1. HISTORY The patient’s history guides the clinician in arriving at a complete diagnosis, particularly as to the possible etiology of the condition. The following are important questions regarding the patient’s current illness that the clinician should ask. 1. What is the patient’s chief complaint? The most common presenting complaints of patients with cataract include progressive blurring of vision with no other associated symptoms. Patient may complain of black spots in the visual field which most of the time are fixed. Most of the onset of the complaints usually starts during the 5th decade of life and are usually progressive. 2. How long has the problem been existing? The duration of the problem should be extracted from the patient. This will help in eliminating the other differential diagnosis. 3. Which eye is involved? Usually both eyes are involved but one eye might have more blurring of vision that the other one. 4. Are there other associated eye problems? There are usually no associated eye problems except if the patient presents with complications like glaucoma or uveitis. 5. Does the patient have any prior consultations / surgeries? It is important to elicit any history of any prior consultations or surgeries. This will help in the determination of the initial onset of the cataract and the medications that may be the cause of the cataract. It is important equally important to know from the patient of any previous eye surgeries that may result in cataract. 6. Other aspects of the patient’s history that should be considered included: a. Family history. Is there a history of any similar illness in the family? Is there any history of any hereditary illness? b. Personal history. What is the nature of the work of the patient? Is there any exposure of the patient to intense UV light? c. Medical history. Has the patient suffered any systemic illness in the past? Has the patient any history of taking oral medications specially steroids for any systemic illness? 2. EYE EXAMINATION The basic tools that one would require in conducting an examination include the following: a. Slit lamp biomicroscope – Instrument used in the assessment of lenticular findings in the eye. b. Visual acuity charts – both for distance and near vision c. Tonometer – used to determine the patient’s intraocular pressure. d. Penlight – for gross examination of the eye e. Direct ophthalmoscope – used for fundus examination

68

CATARACT / 69

COMMON OCULAR FINDINGS IN CATARACT Typically, the patients with cataract present with the following findings: 1. Gross examination – Patients with cataract present with white pupil (leukocoria). Fig 1 shows a typical picture of leukocoria.

Fig. 1. White pupil (leukocoria) 2. Visual acuity – Most patients with cataract present with reduction of vision even with correction. 3. Intraocular pressure – Most patients present with normal intraocular pressures. However if complications of cataract set in, the intraocular pressure may vary. If there is glaucoma, the intraocular is elevated. If there is uveitis, the intraocular pressure is very soft or hypotonic. 4. Extraocular muscle movement – Since the extraocular muscles are usually not involved, patients exhibit full movement on all directions of gaze. 5. Funduscopic findings – Patients who have relative good visual acuity, the fundus is usually normal. However, if the patient has a relatively poor vision and the lens is very dense, the fundus can not be appreciated. If the patient has poor vision and the fundus can be appreciated, the fundus findings may vary depending on the posterior segment pathology. 6. Slit lamp findings – Patients with cataract present with varying degrees of lenticular opacifications. It may vary from a slight haziness of the lens to a dense opacification, from water clefts to vacuoles, and from a white to brunescent lens. The opacifications may also vary with location. They may be found in the cortex, nucleus, posterior capsule or a combination of the above. Usually, the anterior is quiet with no signs of inflammation like cells and flare. Fig 2-5 shows the different slit lamp findings in cataract.

Fig. 2 Cortical Cataract

69

CATARACT / 70

Fig. 3 Posterior Subcapsular Cataract

Fig. 4 Nuclear Cataract

Fig. 5. Intumescent Cataract 3. ANCILLARY EXAMINATIONS 1. Ultrasound of the eye In most cases of cataract, it is normal. However, this examination is important to determine the status of the posterior segment of the eye which includes the vitreous and retina. This is most useful if patient’s cataract is associated with trauma. 4. SYSTEMIC EXAMINATIONS Congenital Rubella is associated with development of cataract at birth. Diabetes is associated with cataract formation in the younger age group.

70

CATARACT / 71

TYPES OF CATARACT 1. Primary Cataract – These are cataracts with no known etiology. a. Congenital – This is present at birth or until 1 year of life. b. Senile / Developmental – This usually starts at age 60 and above. 2. Secondary Cataract – These are cataracts with known etiologies. a. Congenital Rubella – These are cataracts with history of maternal rubella. The rubella virus gains access into the lens. b. Trauma (Non-perforating, Perforating) – These are cataracts that are caused by direct and indirect disruption of the capsules of the lens. This results in the removal the normal membrane properties of the lens. c. Metabolic (D.M.) – These are cataracts wherein the sugar that is entrapped inside the lens is transformed to alcohol resulting in the disruption of the regular lens matrix architecture. d. Toxic (UV, Drugs) – These are cataracts wherein the different drugs / agents affects the enzyme system of the lens epithelial cells causes disruption of the active transport system and protein synthesis. SUMMARY It is the primary objective of this self-instructional material to provide the reader with a guide on how to diagnose patients with cataract. The importance of a comprehensive clinical history and ocular examination are emphasized to be able to diagnose cataract. One should be able to recognize the various ocular signs associated with cataract. RECOMMENDED FOLLOW-UP It is recommended that the students be given demonstration sessions on how to properly conduct historytaking and ophthalmological examination of patients. Following this exercise, it is further recommended that the students be provided clinical sessions to allow them to see actual cases of patients with cataract. CONCLUSION As medical practitioners, you may, in the future encounter patients who will seek consultation for eye problems. One should bear in mind that one should evaluate the patient further for any associated problems with cataract. It is also important that even with a simple problem of cataract, complications may set in without proper treatment and referral. It is imperative that immediate referral to an ophthalmologist should be done for further evaluation and possible treatment. By doing so, early intervention can be facilitated and permanent visual impairment may be avoided. SELF-TEST 1. The most common cause of cataract is is A. aging B. diabetes mellitus C. trauma D. smoking

71

CATARACT / 72

2. The most important examination in assessing the functional severity of the cataract is A. visual acuity B. slit lamp examination C. penlight examination with the light directed obliquely to the pupil D. direct ophthalmoscope 3. In a patient with mild to moderate cataract, the best way to assess the status of the retina is A. utrasonography B. ophtahlmoscopy with the pupils dilated C. electroretinography D. CT scan 4. The senile type of cataract is usually associated with what ocular disease ? A. glaucoma B. uveitis C. diabetic retinopathy D. no associated ocular disease 5. You have a 60 year old patient with cataracts. The vision in the affected eye is 20/50. Direct ophthalmoscopy showed normal fundus findings, The next step in the management of the patient is to refer the patient for A. cataract surgery B. ultrsonography C. refraction D. fasting blood sugar deternination ANSWERS TO SELF-TEST 1. A 2. A 3. B 4. D 5. C

72

DISORDERS OF THE RETINA,VITREOUS & CHOROID / 73

DISTURBANCES IN VISION DISORDERS OF THE RETINA, VITREOUS AND CHOROID Pearl T. Villalon, M.D. INTRODUCTION This is a self instructional material designed for the undergraduate medical student. It merely provides an overview of diseases of the posterior segment of the eye (retina, vitreous, choroid) that cause disturbances in vision, and serves as a guide to further reading. It would be ideal if the basic knowledge acquired from this material will serve as a spingboard for further self study and research on the topics covered. Photographs representative of various retinal problems will be presented, with typical cases discussed at the end of each clinical section. Students are encouraged to apply knowledge that they will acquire from this material and further readings, to cases that they will encounter in the actual clinical setting of the Department of Ophthalmology and Visual Sciences OBJECTIVES After reading this material, the medical student in ophthalmology is expected to: 1. Identify the various elements in a patient's history and ophthalmologic examination that contribute to the formulation of a working diagnosis of posterior segment l disease. 2. Differentiate the various conditions according to location, etiology, pathophysiology, appearance of lesions. 3. To analyze available data and formulate a diagnosis. 4. To recognize the peculiarities of the human retina, the choroid and the vitreous that lend it susceptible to certain problems. RECOMMENDED PREPARATION The student is advised to review the anatomy and physiology of the retina, vitreous and choroid, as well as the optic nerve, before going through this material. You are also advised to review the parts of a clinical history and the method of conducting a complete eye examination. CONTENT Part I. Diagnosis of a Retinal/ Choroidal/ Vitreous Disorder There are four elements necessary to consider the possibility of a retinal disorder. In this discussion, one will find frequent reference to the vitreous and the choroid. This is due to the intimate anatomic and physiologic relationship between the two structures. A. History : Good history taking is a must in a situation where loss of vision or changes in vision is the chief complaint. Many retinal diseases ( as well as vitreous) have peculiar affectations of vision which may immediately give a clue as to the type of problem. B. Ocular Examination : Basic eye examination should be done on all patients: 1. Gross Eye Examination 2. Visual Acuity 3. Intraocular Pressure Determination 4. Extraocular Muscle Movements

73

DISORDERS OF THE RETINA,VITREOUS & CHOROID / 74

5. Funduscopy C. Ancillary Examinations : Special ophthalmologic and laboratory examinations are done as aids in the formulation of an etiologic diagnosis for patients suspected to have retinal diseases. D. Systemic Examination : A thorough systemic examination is done in cases where the eye problem is suspected to be part of a systemic condition or distant trauma ( away from the eye). HISTORY TAKING IN CASES SUSPECTED OF RETINAL DISORDERS The importance of history taking cannot be overemphasized. The information gathered, either volunteered by the patient, or extracted by the clinician, helps in the formulation of a working diagnosis, therefore leading to the decision to take certain subsequent measures to confirm it. The information also gives the clinician an idea as to possible etiology, current state, treatment response and prognosis. The following are important questions to ask patients suspected afflicted with a retinal problem: 1. What is your patient's chief complaint? Patients with retinal problems usually complain of some type of visual disturbance. This may come in the form of: ƒ persistent blurry vision ƒ transient blurry vision ƒ distorsion or metamorphopsia : change in shape , change in size ƒ color vision problems : difficulty in identifying colors ( dyschromatopsia), change in shade, contrast, brightness ƒ visual field loss : central, peripheral, other patterns ƒ difficulty in the dark ( nyctalopia or night blindness) ƒ difficulty in bright light ( hemeralopia) ƒ floaters* and flashes ( photopsias ) * Floaters are black to gray spots and/or fibers that move about in the field of vision of the patient. They seem to float freely and may come in different numbers. Flashes (photopsias) caused by retinal problems are described as arcuate lightning like streaks of bright light in the periphery, noted with or without eye and/or head movements. Patients describe them as " gumuguhit sa gilid ". It must be remembered, however, that photopsias may be of optic nerve origin. Leukocoria or "white pupil" or a "cat's eye" reflex is a chief complaint usually brought not by the patient himself but by parents of a child. This is a significant complaint and may be reflective of a serious condition like an intraocular tumor ( ex: retinoblastoma), related diseases like retinal dysplasia, Coates Disease, Persistent Hyperplastic Primary Vitreous, etc. 2. Did the visual disturbance come gradually or suddenly? The development of the symptom, whether it was a sudden event or a gradually progressive event is very important information. One must take pains to extract this from the patient who may not volunteer this, thinking that it is not important or relevant. 3. What other eye problems accompanied the visual disturbance? Pain is rarely present in retinal disorders. The pain in Diabetic Retinopathy with Neovascular Glaucoma, for example, is due to the elevated intraocular pressure caused by the glaucoma part of the problem. Ocular discomfort and/ or redness may accompany intraocular inflammatory conditions with retinal involvement. 4. How long has this been going on? 74

DISORDERS OF THE RETINA,VITREOUS & CHOROID / 75

The duration of the problem is an important information. It is also important to know if this is the first time or if the problem is recurrent. If this is a recurrent affair, be sure to ask about timing and sequence of events , duration, interval and treatment as well as treatment responses. 5. Which eye is involved? Laterality is just as important as knowing if the same or similar problems have occurred in the fellow eye. Do not forget to ask when. If bilateral, ask which eye was affected first, then the sequence of events. 6. Were there previous consultations and treatments ? When ? What medications were used ? What medications are being used? Did the patient have any eye operation for this or other problems? This information has bearing on the present state of the eye, if you are seeing a patient who was or is under treatment by someone else. Try to get very accurate information about medications, surgeries, other forms of treatment ( lasers , gas injection, cryotherapy,etc). 7. Other aspects of the patient's history that should be considered include: a) Family History : Is there a history of similar illness in the family? Is there any history of any hereditary illness? b) Social History: Does the patient smoke? Does he have pets ? Does he eat raw food? Does he engage in contact sports or other unusual sports like deep sea diving, bungee jumping, competitive weight lifting, boxing, etc, and have been injured in the head and/or eyes? Does he take prohibited drugs or has ever taken any ? What is his occupation? Is there exposure to chemicals or toxic fumes ? Does he travel often and has been to places like Africa, Middle East, China ? c) Medical History: Does he have cardiac disease ? Hypertension? Diabetes Mellitus? Asthma? Hematologic Disease? Cancer ? Pulmonary Tuberculosis ?Other diseases? Accidents that required surgery and/or hospitalization? Any operations and for what? Does he have symptoms like angina, pedal edema, joint pains, oral ulcers, intractable fever, to name a few. Has he ever had a blood transfusion and when? Has he ever had cobalt therapy ? Is he on any medications for other illnesses? What are these medications ? How long has he been taking these? d) Sexual History: The presence of a history of any sexually transmitted disease should likewise be elicited from the patient. What is his sexuality? During history taking keep your mind open to the possibility that the patient may not be volunteering information because he does not think that it is relevant or important, or the patient is deliberately withholding information. The clinician must be observant of patient responses, reactions, demeanor, attitude during history taking and find clues that will lead him to conclude these. For example, history taking of AIDS patients is particularly challenging. OPHTHALMOLOGIC EXAMINATION OF THE PATIENT The basic tools needed for an ophthalmologic examination of the suspected retina patient are 1. Visual Acuity Charts : both for distance and near vision 2. Penlight : for gross examination of the eye and adnexae 3. Tonometer: used to determine intraocular pressure 4. Ophthalmoscope: used to examine the fundus ( retina and optic disc) 5. Slit Lamp : instrument used in the assessment of the ocular media, the retina in high magnification( with special lenses) 1. Gross eye Examination: The ocular adnexae are not affected in retinal disease. Except in some retinal disorders of inflammatory origin, there will be no ciliary flush or redness. The pupils must be examined for direct and consensual light reactions. Severe retinal damage as well as some optic nerve diseases can cause abnormal afferent pupillary

75

DISORDERS OF THE RETINA,VITREOUS & CHOROID / 76

responses . A "white pupil" in a small child is always an alarming and significant finding. 2. Visual Acuity: Vision is usually affected in some way in most retinal disorders. Typical affectations in vision are seen in specific retinal diseases. For example retinal edema at the macula can cause metamorphopsia besides actual blurring recorded as loss of lines in the Snellen chart. In retinal detachments, many patients report seeing floaters and light flashes before the onset of "wavy vision", actual blurring, visual field "cuts". Macular disease causes blurry central vision while extramacular retinal diseases will cause peripheral visual loss. Vitreous opacities also cause blurry vision. Many retinal disorders come with vitreous problems. 3. Intraocular Pressure: Again, as with Visual Acuity, there is no set pattern for intraocular pressures in retinal diseases. Retinal detachments usually cause some amount of hypotony due to the accompanying inflammatory changes and involvement of the ciliary body in the detachment. The elevated intraocular pressures in end stage Proliferative Diabetic Retinopathy with Neovascular Glaucoma are due to blocking of the drainage angles by neovascular elements of the proliferative disease. Macular diseases usually do not cause pressure changes. 4. Extraocular Muscle Movements: Patients exhibit normal EOMs in retinal disease. 5. Funduscopic Findings : These depend on the retinal problem. The vitreous always has to be considered in assessing the status of the retina. These will be discussed in a separate section. 6. Slit Lamp Biomicroscopy with special lenses: These depend on the retinal problem and will be discussed in a separate section. Problems in the vitreous and anatomic /pathologic vitreoretinal relationships can be determined with this examination. ANCILLARY EXAMINATIONS Additional examinations may be necessary to make a diagnosis . The more commonly requested examinations are: 1. Fundus Fluorescein Angiography : Fluorescein ngiography is a procedure that involves the injection of a dye, sodium fluorescein, into the antecubital vein. This dye is carried with the circulation and outlines the retinal and choroidal vascular system. This then picked up by a special camera and recorded in film. Abnormalities in retinal and choroidal circulation are then highlighted, as well as abnormalities in structure and form. Retinal and choroidal diseases have typical fluorescent qualities and manifestations, thereby allowing the observer to confirm certain clinical observations. As with the retina and choroid, the optic nerve head also has typical angiographic fluorescent qualities so that deviations may be interpreted as possibly pathologic and correlated with clinical findings.

76

DISORDERS OF THE RETINA,VITREOUS & CHOROID / 77

2. Ocular Ultrasonography : In the presence of media opacities the ultrasound is a useful tool to evaluate the anatomic relationships in the posterior pole. For example, in the presence of a very dense cataract, the presence or absence of a retinal detachment can be determined. It is used to assess the density of tumors, giving clues as to the type of tumor, and possibly a diagnosis. It is also very useful in assessing the amount of and density of intravitreal material such as blood, and to locate intraocular foreign bodies in trauma cases.

SYSTEMIC EXAMINATION

Many retinal diseases are part of systemic problems like Diabetes Mellitus, Hypertension, Systemic Lupus Erythematosus and other collagen diseases, Pulmonary Tuberculosis, Malignant Disease, Hematologic Disorders . Some of these like Diabetic Retinopathy, Hypertensive Retinopathy, Collagen Disease, may present typical funduscopic findings. Blood chemistry (blood sugar, cholesterol, triglycerides, BUN, creatinine, etc. ) and x-rays form an important part of "systemic examination" and must be done when a patient is suspected of having a certain disease. Patients can present with pedal edema, skin discoloration of the lower extremities, non healing ulcers and wounds in the lower extremities in Diabetes Mellitus.

Part II. Peculiarities of the Retinal Microstructure A. The 3 Neuron Relay System: The human retina is a tall structure that has ten layers. In the ten layers are three main cell types that relay visual/chemical information through the retina , from outside to in, and to the optic nerve and finally into the brain. Any breakdown in the neurons, the supporting cells and the relay system itself can manifest as visual disorders. The ten layer system is supplied by two different vascular systems. The outer retinal layers derive nutrition from the choriocapillaris of the choroid. The inner layers, from the inner nuclear layer inwards, are supplied by the retinal vasculature itself. Certain retinal diseases can be traced to problems of perfusion , and , knowing the extent of supply of each system, one can then predict the depth of retinal involvement. B. The Retinal Pigment Epithelium: The retinal pigment epithelium (RPE) is the first of the ten layers, and is the outermost layer, adjacent to the choriocapillaris of the choroid. It takes care of most metabolic processes of the outer retinal layers, keeps the retina “dry” with its “outer blood-retina barrier”, participates in the recycling of retinol, disposes of metabolic wastes of the other cells. A breakdown of the “outer blood-retina barrier” can lead to accumulation of fluid in the subretinal space and the sub RPE space. This manifests as retinal edema. A physiologic “pump” also exists in the human RPE layer and this contributes to keeping fluid away from the retina.

77

DISORDERS OF THE RETINA,VITREOUS & CHOROID / 78

C. The Blood –Retina Barriers: There are two so called blood-retina barriers that keep the retina ‘dry”: D. 1. The Inner Blood-Retina Barrier : This is attributed to the tight endothelial cell junctions of the retinal capillaries. Any disturbance in the integrity of these tight attachments leads to oozing of fluid and/or blood, as well as lipids and proteins from the retinal vascular tree. This manifests as retinal edema, hard exudates, hemorrhages. 2. The Outer Blood-Retina Barrier: This is found in the RPE. The tight intercellular attachments between RPE cells, called Zonnula Occludens keeps the RPE layer leakproof from the highly vascular choriocapillaris of the choroid, to which it is adjacent. An additional feature is the presence of a physiologic “pump” that keeps fluid contained outside the retina.

E. The Vitreoretinal Relationship : The globe is filled with a gel called the viteous. This gel is 99% water and only 1% solid, the solid part being collagen fibrils that make up the meshwork, and the very few cells called hyalocites. The vitreous is firmly attached to the optic nerve, the macula, along blood vessels and also at the so called vitreous base (where the retina ends peripherally and anteriorly). Many disorders in vision originate from problems with the vitreoretinal interface. For example, vitreous liquefaction and anterior displacement can cause undue traction on the vitreous base and may end up in a retinal tear or dialysis, leading to retinal detachment. Another example is undue traction of the vitreous gel and posterior hyaloid of the vitreous, on the macula, causing the formation of a macular

hole. Part III: Diseases of the Retina, Choroid and Vitreous Examples of Retinal and Vitreous Disorders with typical fundus findings will be described briefly in this section. Please read about details and other retinal and vitreous disorders. A. Retinal Vascular Disease : Diabetic Retinopathy, Hypertensive Retinopathy, Central Retinal Vein Occlusion, Central Retinal Artery Occlusion, Retinopathy of Prematurity B. Maculopathies: Age Related Macular Degeneration, Central Serous Chorioretinopathy, Macular Holes, Cystoid Macular Edema C. Heredodegenerative Diseases of the Retina: Retinitis Pigmentosa, Color Blindness, Juvenile Foveal Retinoschisis, Best’s Vitelliform Macular Degeneration D. Tumors: Metastatic Tumors, Melanomas, Retinoblastoma E. Retinal Detachments , Retinoschisis F. Vitreous Opacities / Degeneration: Vitreous Detachment and Syneresis, Vitreous Hemorrhage, Asteroides Hyalitis, Synchisis Scintillans, Vitritis G. Retinochoroiditis : Toxoplasmosis, Toxocariasis H. Uveal Effusion : Vogt-Koyanagi-Harada Syndrome H: Infectious Retinopathies : HIV Retinopathy

78

DISORDERS OF THE RETINA,VITREOUS & CHOROID / 79

DIABETIC RETINOPATHY Diabetic Retinopathy is a complication of Diabetes Mellitus and manifests mainly as vascular changes in the retina. The onset of the disease is usually after many years of diabetes. Good blood sugar control has been proven by wide based studies, to be a key modifiable factor, and must be maintained to delay the onset of the complications of diabetes mellitus or their progression. Diabetic Retinopathy is one such complication, and is closely related to the onset of diabetic nephropathy. There are two stages of diabetic retinopathy : non proliferative and proliferative stages, with each subdivided further into minor stages: mild, moderate,severe, very severe for the former and early and high risk for the latter. The typical fundus findings in non proliferative DM retinopathy are: microaneurysms, dot and blot retinal hemorrhages, hard exudates, soft exudates. The hallmark of the proliferative stage is the growth of abnormal new vessels (neovascularization) either on the disc or on the retina. Vision deteriorates when the patient has macular edema., retinal hemorrhages, vitreous hemorrhage, vitreoretinal traction and detachment, neovascular glaucoma. The only proven treatment is laser treatment . Panretinal treatment is performed when the disease reaches the proliferative high risk stage, while macular focal and grid laser treatment can be done at any stage if maculopathy causes significant visual loss. Treatment for vitreoretinal traction comes in the form of vitreoretinal surgery. Surgery is also done for non clearing vitreous hemorrhage, dense premacular hemorrhage, macular traction.

Non proliferative DM retinopathy

Proliferative DM retinopathy

proliferative DM retinopathy with vitreoretinal traction

79

DISORDERS OF THE RETINA,VITREOUS & CHOROID / 80

CENTRAL RETINAL VEIN OCCLUSION Central Retinal Vein Occlusion or CRVO manifests in two ways or types: ischemic and non ischemic ( also called "stasis") . The more severe type is the ischemic type where the fundus is covered with splinter hemorrhages radiating from the optic disc and which appear to be following the disposition of the nerve fiber layer. One typically finds soft exudates ( "cottonwool spots") indicating retinal ischemia. Retinal veins are tortuous and dilated. The disc may have blurry borders covered with the retinal hemorrhages. Vision is very poor in this situation. In contrast,in the non ischemic type , there are fewer retinal hemorrhages and usually no soft exudates. The disc borders are not blurry, but there is definite retinal venous tortuosity and dilation. Vision is much better than in the ischemic type. CRVO can happen to both elderly and young individuals. In the elderly, one thinks of systemic vascular diseases such as hypertension , diabetes, arteriosclerosis as the cause of the episode. In the younger individual, one thinks if other things such as inflammatory disease, collagen disease, hematologic disorders, etc. A fundus fluorescein angiogram will help confirm the type of CRVO. Treatment comes in the form of laser treatment ( scatter treatment similar to PRP of DM retinopathy) when certain criteria are met.

CRVO CENTRAL RETINAL ARTERY OCCLUSION Central Retinal Artery Occlusion or CRAO , is one of only two "absolute" emergencies in ophthalmology. It does not occur as frequently as CRVO. CRAO manifests as sudden painless loss of vision. Visual loss is severe, and remaining vision just after the episode is usually in the area of count fingers and hand motions. Some patients report a prodrome of temporary "wipeout" of vision several times in a span of days to weeks before the actual CRAO. This "wipeout" is very brief and lasts for a few seconds. At this time the patient's vision appears white or gray with all details gone temporarily. CRAO is usually related to systemic vascular disease like hypertension, arterioscelrosis, collagen disease, hematologic disorders. Embolic phenomena must also be entertained, such as in patients with internal carotid stenosis, arrhythmias, cardiac valvular disease, peripheral vascular disease, others. Prognosis for visual

80

DISORDERS OF THE RETINA,VITREOUS & CHOROID / 81

recovery is very poor. Treatment must be instituted within 5 minutes of the attack and comes in the form of immediate lowering eye pressure to improve intraocular perfusion.

CRAO

AGE RELATED MACULAR DEGENERATION Age Related Macular Degeneration( ARMD) is one of the major causes of central visual loss in the Western world, in people over 50 years of age.. It is more common in the elderly and the incidence rises sharply after the age of 75 years. There are two types, the Dry Type and the Wet , Exudative or Hemorrhagic Type. The Dry type is the more common one and visual loss is not as severe as in the Wet type. The Wet Type comprise only 10% of all ARMD but is resposible for 90% of those with vision of 20/200 and less. The basic pathology of the Dry type is the accumulation of cellular debris and formation of "drusen" under the retina. There is also atrophy of the retinal pigment epithelium. In the Wet Type a neovascular complex grows under the retina from the choroid,. This bleeds and causes scarring of the retina. In ARMD, age is a definite risk factor. Other risk factors implicated are: smoking, cardiovascular disease, ethnicity, undue exposure to UV light, lack of vitamin C and other antioxidants. Nothing has been proven about these risk factors but recent studies show that there may be proof of its relationship to smoking. Treatment of the Wet type is thermal laser treatment to the abnormal vascular complex, if extrafoveal. The subfoveal types are treated using a new modality called Photodynamic Therapy(PDT), which involves the injection of a photosensitive dye and a special type of laser.

Dry ARMD

Wet ARMD

RETINITIS PIGMENTOSA 81

DISORDERS OF THE RETINA,VITREOUS & CHOROID / 82

Retinitis Pigmentosa( RP) is a retinal disorder that belongs to the family of Heredodegenerative Diseases and “ Tapetoretinal Diseases”. It is characterized by progressive degeneration of the rods and cones, and in most cases is associated with migration of pigment epithelial cells into the retina. There are several modes of transmission: autosomal recessive, X-linked recessive, autosomal dominant patterns of inheritance. The first two have the earliest onset and worst prognosis. Patients with the autosomal dominant form may be symptom free till middle age. Genetic analysis has shown that the defect in most affected individuals is in the gene that codes for rhodopsin. There are 2 types of RP: type I in which the rods are affected earlier than the cones, and type II in which the cones are affected earlier than the rods. Symptoms include night blindness or nyctalopia for most cases of type I , progressive contraction of peripheral visual fields, blurring of vision in some cases ( with macular involvement, and development of cataract) . Absence of night blindness is possible especially in type II RP, and remains so until visual fields are markedly contracted. Fundus findings include vitreous cells and opacities, narrowed arteries, diffuse pigmentation of the retinal pigment epithelium( RPE), bone spicule and comma shaped Intraretinal proliferation of pigmented cells, and waxy pallor of optic disc in late stages. RP with very little or no pigment is possible ( sine pigmento) and is usually type II. Other causes of night blindness must be ruled out ( ex. vitamin A deficiency, RP systemic syndromes, congenital stationary night blindness). RP is a bilateral disease and “unilateral RP” does not exist. If findings are unilateral other causes must be ruled out. (ex. blunt trauma, uveitis, long standing retinal detachment). There is no known treatment of Retinitis Pigmentosa. Cataract extraction may help improve vision. Daily doses of 15,000 IU of vitamin A helps slow down deterioration of ERG, with no known demonstrable effect on vision.

Retinitis pigmentosa

Retinal Detachment Retinal Detachments are conditions where the retinal pigment epithelium is separated from the inner retinal layers, with accumulation of fluid in the “subretinal space” (space between RPE and photoreceptor layer). 82

DISORDERS OF THE RETINA,VITREOUS & CHOROID / 83

Retinal detachments can be “rhegmatogenous” (with retinal break) or “non rhegmatogenous” ( without a break). Causes of Non rhegmatogenous detachments are exudation and traction. Causes of exudation are inflammatory disease of the choroid and retina, subretinal tumors, accelerated hypertension as in ecclampsia, other vascular disease like diabetes mellitus. Causes of traction detachments are diabetic retinopathy, trauma, ischemic retinopathies. Rhegmatogenous retinal detachments are associated with retinal thinning and peripheral retinal degeneration, and vitreoretinal traction during vitreous liquefaction and detachment, myopic degeneration, trauma. Symptoms of rhegmatogenous retinal detachments are: sudden onset of blurry vision, wavy vision, visual field cuts, flashes and floaters Symptoms of exudative and traction detachments are determined by the cause. As a rule, patients experience blurry vision of variable onset depending on cause. Management of rhegmatogenous retinal detachment is surgical. The principles of retinal reattachment surgery are: finding the break (s), with closure of the retinal break, sealing of the retinal breaks with a chorioretinal scar. Management of Traction detachment is surgical as well, while management of exudative detachment is primarily medical, addressing the primary cause.

Retinal detachment

Vitreous Hemorrhage Decreased vision and floaters caused by non traumatic vitreous hemorrhages are common causes of emergency consults with the eye doctor. In adults, the most frequent cause is diabetic retinopathy. Other causes are retinal break(s) without detachment, posterior vitreous detachment, rhegmatogenous retinal detachment, neovascularization due to central and branch retinal vein occlusion. Any cause of peripheral neovascularization may cause vitreous hemorrhage, including the chronic stages of pars planitis and other uveitides. Trauma ( including the Battered Child syndrome) should always be ruled out in children. Symptoms come in the form of sudden onset of blurry vision, frequently with floaters in the form of “many dust like particles” which are dispersed RBC, or “streaming dark lines”. Massive vitreous hemorrhage can cause significant loss of vision without the floaters. Occasionally there are reports of light flashes in the peripheral field of vision. Management will be directed at: finding the cause and clearing the blood. In most cases retinal examination will permit adequate assessment of the situation and identification of cause. When vitreous hemorrhage is too dense an ocular ultrasound must be done to determine if retinal detachment, tumors, other abnormalities are

83

DISORDERS OF THE RETINA,VITREOUS & CHOROID / 84

present behind the blood. Immediate management comes in the form of strict bedrest with head elevation ( high back rest), bilateral patching. This allows faster resorption of blood. The general rule is, if no retinal detachment or tumor is detected, the hemorrhage may be observed for 4-6 months and left to absorb. In the presence of retinal detachment, tumor, other disease requiring immediate attention, vitreoretinal surgery is advised. If a retinal break without a detachment is found, the break is sealed with laser. If this is a hemorrhage accompanying the traction detachment and/or retinal neovascularization, surgical removal of the vitreous hemorrhage with /without laser treatment is considered within the context of management of the secondary retinopathy.

Ocular Toxoplasmosis Ocular toxoplasmosis, caused by an intraocular obligate parasite is one of the more common forms of posterior uveitis ( inflammation of the uveal tract).The organism Toxoplasma gondii, is a protozoan that has predilection for the retina. The classic finding is retinochoroiditis. There are two types of Ocular Toxoplasmosis: congenital and acquired. The congenital type is acquired by the pregnant immunoincompetent mother, and passed on to the unborn child. The later in pregnancy the infection is acquired by the mother, the more serious the ocular problem. The acquired type is via ingestion of the organism in uncooked food or infected fomites. The infection is easily quelled by the patient but the organism finds its way into the eye and causes an inflammatory reaction in the vitreous, retina and choroid hence, a retinochoroiditis. Scarring of the retina and choroid are prominent in the quiescent phase. Recurrences of activity are possible and can be found at the edges of scars from previous attacks. Visual affectation is dependent on the part of the retina affected. Macular involvement will certainly lead to poor vision. Diagnosis is best made clinically although immunologic examinations are helpful. Treatment is in the form of oral antibiotics that interfere with the organism’s pathway for protein transcription, such as sulfadiazine, clindamycin,pyrimethamine. The use of steroids whether oral, topical or peribulbar are not without hazard.

Retinochoroidal scars in “quiet” toxoplasmosis

“Headlight in the fog” in active toxoplasmosis

Vogt-Koyanagi-Harada Syndrome Vogt-Koyanagi Harada Syndrome (VKH) is a rare and unusual form of diffuse granulomatous uveitis. This is found more in pigmented individuals.

84

DISORDERS OF THE RETINA,VITREOUS & CHOROID / 85

The patient usually complains of sudden onset of blurry vision, photophobia, perhaps floaters, sometimes with headache, neck stiffness, tinnitus. The disease presents as serous retinal detachments with yellowish white patches of edema of the retinal pigment epithelium, retinal vasculitis, optic disc edema. There may be a heavy anterior uveitis. Later in the course of the disease the patient develops vitiligo ( white patches on the skin) and poliosis (whitening of eyelashes). A high index of suspicion, based on clinical presentation and findings, a spinal tap and immunologis tests will help make a diagnosis of VKH. Fundus fluorescein angiography may help define structures involved. Patients are put on systemic steroids and immunosuppressives. The final outcome depends on control of the inflammation and scar formation., Vision is relatively good except where the macula has been affected. The course is long term and indolent.

Serous detachments in VKH

“Sunset Glow” in VKH

HIV Retinopathy The Human Immunodeficiency Syndrome (HIV) may affect the eye directly. More frequently because of the underlying systemic immunodepression, the eye becomes vulnerable to a number of opportunistic infections and neoplastic conditions.These include CMV retinitis, Toxoplasmosis, Candida Retinitis, Pneumocystis carinii infection, Kaposi’s sarcoma. The most consistent feature are cottonwool spots (retinal nerve fiber infarcts) which are present at some time during the course of the systemic HIV infection. These localized ischemic areas are located at the posterior pole and may be related to circulating immune complexes. In most cases cottonwool spots are asymptomatic and blurry vision comes only if the lesion is at or close to the macula. Diagnosis must exclude other causes of cottonwool spots (diabetic retinopathy, collagen vascular disease, retinal vaso-occlusive disease). The manifestations of disease differ with the type of opportunistic infection. Management comes in the form of AIDS management such as systemic gancyclovir and foscarnet. The eye disease is managed with intravitreal implants of these medications in addition to the systemic management. Vitreoretinal surgery is indicated when there is vitreoretinal traction and non clearing vitreous hemorrhage.

85

DISORDERS OF THE RETINA,VITREOUS & CHOROID / 86

Cottonwool spots in HIV Retinopathy

CMV retinitis in HIV patient

SUMMARY It is the primary objective of this material to provide the medical student a guide on how to recognize the retina patient with a good history and thorough eye examination. The importance of comprehensive history taking cannot be overemphasized, as many retinal diseases have very typical stories to tell. CONCLUSION As medical practitioners we should be aware of the different types of retinal diseases and their manifestations. More than this ,the medical practitioner should realize that many problems of the retina can be traced to systemic diseases or problems of parts of the body distant from the eye, as in distant trauma.. This being so, it is then possible for a systemic disease to be diagnosed because of the typical retinal changes, as in diabetic retinopathy. It is therefore very important for the medical practitioner to recognize these retinal problems so that proper management can be instituted immediately, especially in systemic diseases. REFERENCES 1. Newell,Frank.Texbook of Ophthalmology. Latest edition 2. Podos, Yanoff.. Textbook of Ophthalmology, Retina and Vitreous. Vol 9. 1994 3. Tasman, Jaeger. Atlas of Clinical Ophthalmology. 2nd ed. 2001 4. Vaughan,Asbury,Riordan-Eva. General Ophthalmology. 15th ed. 1992 EXERCISES Case 1. A 50 year old female comes to you with complaint of sudden onset of blurry vision of the right eye since a week prior to consult. She reports that she has had diabetes mellitus for the past 15 years and is on insulin, with poor control of blood sugar. Vision

OD: Count Fingers 2 feet OS: 20/20 J1 with correction Intraocular Pressure OU: 15 mm Hg

86

DISORDERS OF THE RETINA,VITREOUS & CHOROID / 87

Slit Lamp findings: OD: clear lens; no anterior chamber cells; hazy vitreous with no view of the retina OS: clear lensq ; no anterior chamber cells ; clear vitreous Funduscopy OD: negative ROR OS: occasional microaneurysms noted all over the fundus. Several cottonwool spots and blot hemorrhages are also seen all over the fundus. EOMs OU: normal Based on the data given above: 1. What is your working diagnosis of the case ? 2. Why ? 3. Is there anything else that you would want to ask the patient? 4. What ancillary procedures would you ask for? Case 2. A 25 year old jeepney driver consults you for sudden blurring of vision of the left eye, of 1 week duration. He says that straight lines are distorted and the image size in the affected eye is smaller than the normal eye. He denies any history of diabetes, hypertension, any other diseases, any history of surgery or intake of medications. Vision OD: 20/20 J1 without correction OS: 20/70 J5 corrected to 20/25 with a refraction of + 1.50 D sphere Intraocular Pressure OU: 17 mm Hg. Slit Lamp Findings OU: normal cornea, anterior chamber, lens. vitreous Funduscopy OD: normal disc, macula, vessels OS: normal disc, normal vessels; macula is elevated (edematous), with fine white stippling EOMs OU: normal Based on the data given above: 1. What else would you want to know during your history taking? 2. Why? 3. What ancillary procedures will you order for the patient? 4. If the patient were 70 years old, what will your working diagnosis be?

87

GLAUCOMA / 88

DISTURBANCES IN VISION GLAUCOMA Norman M. Aquino M.D. Marissa N. Valbuena M.D., MHPEd INTRODUCTION Glaucoma is a neurodegenerative disease characterized by death of retinal ganglion cells and loss of their axons. This leads to the formation of defects within the nerve fiber layer of the retina (RNFL) with subsequent cupping, or excavation, of the optic nerve head. As a result, functional visual field disturbance or visual field loss occur. OBJECTIVES After reading this material, the medical student in ophthalmology is expected to: 1. discuss the pathophysiology of glaucoma 2. given a patient with glaucoma, be able to extract a complete history and perform an ocular examination. 3. discuss the examinations for glaucoma. 4. discuss the principles of management of glaucoma RECOMMENDED PREPARATION The student is advised to review the anatomy and physiology of the anterior chamber angle, retina and optic nerve before going through this material. He is also advised to review the parts of a clinical history and the method of conducting a complete eye examination. CONTENT Glaucoma and The Optic Nerve The assessment of the morphologic features of the optic disc are essential in glaucoma. (Table 1). The optic disc can be examined clinically with a direct ophthalmoscope, an indirect ophthalmoscope, or a posterior fundus lens (with a slit lamp). The direct ophthalmoscope, although providing high magnification, does not provide sufficient stereoscopic detail. The main disadvantage of the indirect ophthalmoscope is the small image size. The best method to examine the optic disc is with the posterior fundus lens with a slit lamp. This system provides high magnification, excellent illumination and a stereoscopic view of the disc. Table 1: Clinical Evaluation of the Optic Nerve Head 1. Size and shape of the optic disc; 2. Size, shape, and pallor of the neuroretinal rim; 3. Size of the optic cup in relation to the area of the optic disc; 4. Configuration and depth of the optic cup; 5. Cup-to-disc diameter ratio and cup-to-disc area ratio; 6. Position of the exit of the central retinal vessel trunk on the lamina cribrosa surface; 7. Presence and location of splinter-shaped hemorrhages; 8. Occurrence, size, configuration, and location of parapapillary chorioretinal atrophy; 9. Diffuse and/or focal decrease ot the diameter of the retinal arterioles; and 10. Visibility of the RNFL

88

GLAUCOMA / 89

The optic disc is usually round or slightly oval in shape and contains a central cup. The tissue between the cup and the disc margin is the neuroretinal rim (Fig.1). In normal patients, this rim has a uniform width and a color that ranges from orange to pink. It is common practice to grade an optic disc by comparing the diameter of the cup to the diameter of the disc in both the horizontal and vertical meridians. This is usually expressed as a ratio such as 2/10 or 0.2. Usually, a horizontal cup-to-disc ratio of 3/10 or 0.3 is considered normal. Cup-to-disc ratio increases slightly with age. There are also racial differences in cup-to-disc ratios.

Fig.1- Normal Optic Nerve. Arrows show borders of optic disc and cup Elevated intraocular pressure was traditionally considered to be solely responsible for the anatomic damage in glaucoma. However, it has now been shown that there are other factors which play a role in the production of these morphologic changes seen in the glaucomatous optic nerve. Inherent structural defects, genetic predisposition, vascular and biochemical insults have been found to play a significant role in the pathogenesis of the disease. (Fig.2)

Fig.2- Concept of proposed pathogenesis of glaucoma. European Glaucoma Society

The appearance of the optic disc often provides essential information about the existence, the stage, and course of the disease (Table 2).

89

GLAUCOMA / 90

Table 2: Optic Disc Signs of Glaucoma Generalized Signs 1. Large optic cup 2. Asymmetry of the cups 3. Progressive enlargement of the cups Focal Signs 1. Localized narrowing of the neuroretinal rim (notching) 2. Vertical elongation of the cup 3. Cupping to the rim margin 4. Regional pallor 5. Splinter hemorrhage 6. Nerve fiber layer loss Less Specific Signs 1. Exposed lamina cribrosa 2. Nasal displacement of vessels 3. Baring of circumlinear vessels 4. Peripapillary atrophy Generalized enlargement of the cup may be the earliest change detected in glaucoma (Fig. 3). This may be difficult to appreciate unless previous clinical records, diagrams or photographs are available. It is also useful to compare one eye to the fellow eye as disc asymmetry is unusual in normal individuals. Focal enlargement of the cup appears as localized notching or narrowing of the rim (Fig. 4). If this occurs at either (or both) the superior or inferior pole of the disc, the cup becomes vertically oval. In more advanced glaucoma, the tissue destruction extends behind the cribriform plate and the lamina bows backward. The optic nerve head then takes on an excavated and undermined appearance that has been likened to a “beanpot” (Fig. 5). Splinter hemorrhage usually appears as a linear red streak on or near the disc surface (Fig. 6). The hemorrhage clears over several weeks but is often followed by localized notching and pallor of the rim with subsequent visual field loss.

Fig. 3- Enlarged optic cup with narrow neuroretinal rim.

Fig. 4 - Localized loss of neuroretinal rim (notching)

90

GLAUCOMA / 91

F

Fig. 5 - Glaucomatous disc damage: Eye of patient; of a cadaver; and a histologic section showing excavation or cupping of the optic disc

Fig. 6 - Splinter Hemorrhage in the disc

In the normal eye, the nerve fiber layer can be best visualized with red-free illumination, and appears as a pattern of striations that radiate toward the optic disc. With the development of glaucoma, the nerve fiber layer thins and becomes less visible. Diffuse loss of the nerve fiber layer may be a very important sign of early glaucomatous damage (Fig.7).

Fig. 7 - Nerve fiber bundles

The Normal Visual Field With the eye open and looking straight, the visual field of that eye is defined as all the space that it can see. The dimensions of the normal field of vision are defined relative to fixation. The normal visual field extends approximately 60 degrees superior and nasal, 70 degrees inferior, and 90-100° temporal to fixation

91

GLAUCOMA / 92

(Fig. 8). . The blindspot occupies the area defined by the optic nerve head and is typically located 15 degrees temporal to fixation. Visual sensitivity is greatest in the center, the fovea, and decreases toward the periphery. By convention, the visual field of each eye is plotted as the patient sees it.

Fig. 8 - Normal limits of the visual field of the right eye

Traquair graphically described the visual field as a three-dimensioned “island of vision surrounded by a sea of blindness” (Fig. 9). He described the variable slope of the island, the “peak” of sensitivity at fixation, and the normal blind spot as a “gorge” that drops to sea level at a location approximately 15° to the temporal side of the highest peak.

Fig. 9 - Traquiar’s Island of vision/ Hill of vision

The size and contour of a visual field can be influenced by other factors in addition to glaucoma. These include facial structure, eyelid anatomy, pupil size, clarity of the ocular media, and refraction. Many neurologic and neuro-ophthalmologic conditions also alter the visual field. Testing the Visual Field Visual field testing, or perimetry, is an important diagnostic tool in glaucoma. It likewise plays a critical role in monitoring the disease’s progression. There are various ways of testing and mapping out a patient’s visual field. The confrontational method of visual field testing will quickly demonstrate gross field defects. It may be the only practical method to evaluate patients who are unable to perform well using the more sophisticated

92

GLAUCOMA / 93

instruments used in perimetry testing. The detection of small field defects in early glaucoma may be missed using this technique. Kinetic visual field testing is performed, as the name implies, with a moving test object. The object, usually a light of variable size and intensity projected on an evenly illuminated surface, is moved from a nonseeing area toward a seeing area. The location is recorded when the patient sees the object. The process is repeated until a boundary of seeing and non-seeing is determined. This boundary line is called an isopter. Several isopters are usually obtained using test objects of different size and or intensity (Fig. 10). The Goldmann perimeter is an example of a manual kinetic perimeter.

Fig. 10 - Kinetic perimetry

Fig. 11 - Static perimetry

Static visual field testing involves the use of non-moving test spots. Fixed test spots of varying intensity of light are presented for a short period of time. The patient responds when light is perceived in each test spot. Static testing attempts to find the light sensitivities of the eye at preselected locations in the visual field. It describes the contour of the hill of vision (Fig. 11). The Humphrey Visual Field Analyzer and the Octopus are examples of automated static perimeters. The Glaucomatous Visual Field Glaucomatous optic nerve damage produces characteristic changes in the contour and shape of the visual field. It is important to correlate changes in the visual field with changes in the optic disc. If an appropriate correlation is not present, other causes of visual field loss must be considered. Visual field changes seen in glaucoma reflect retinal and optic nerve anatomy. Retinal nerve fibers radiate from the optic nerve head and are distributed in an arcuate manner around the foveal region (Fig. 12). Glaucomatous damage at the optic nerve head produces visual field defects in the region subserved by the

93

GLAUCOMA / 94

affected nerve fibers. The location, distribution, size and shape of the resulting visual field defect, called a scotoma, is therefore determined by the location and extent of the anatomic defect. Typical glaucomatous visual field defects include localized paracentral scotomas, arcuate defects, nasal steps, and temporal defects.

Fig. 12 - Distribution of retinal nerve fibers

The most common location of glaucomatous visual field defects occurs within an arcuate area commonly referred to as Bjerrum’s area which includes that portion of the arcuate region that extends from the blindspot to the median raphe, where it extends 10-20 degrees nasally from fixation (Fig. 13) The arcuate scotoma often begins as a single area of relative loss which become larger, deeper and multi-focal (Fig. 14). In its full form, the scotoma arches from the blind spot and ends at the nasal raphe, becoming wider and closer to fixation on the nasal side (Fig. 15). Later in the disease, the arcuate scotoma may break through the nasal periphery. A paracentral scotoma is a discrete area of absolute or relative visual loss within 10° of fixation. The scotoma can be single or multiple and can occur as an isolated finding or can be associated with other visual field defects. Paracentral scotomas may be missed because of their location and small size. With progression, paracentral scotomas become deeper and larger and may gradually coalesce forming an arcuate or Bjerrum scotoma. A nasal step is a depression in the superior or inferior nasal field near the horizontal raphe. It can occur as an isolated finding in early glaucoma or can be associated with a paracentral or arcuate defect. A temporal defect is a wedged-shaped defect extending from the periphery toward the central visual field (Fig. 16). These defects are produced by damage to the nasal neuroretinal rim. Nerve fibers radiate without deviation from the optic nerve; therefore wedge-shaped defects develop. Temporal defects are usually found as a late manifestation of glaucomatous visual field loss.

Fig. 13 - Bjerrum’s area

94

GLAUCOMA / 95

Fig. 14 - Beginnings of an arcuate scotoma

Fig. 15 - Full arcuate scotoma

Fig. 16 - Temporal wedge defect

Summary: The Optic Nerve and The Visual Field The visual field is the functional correlate of the anatomical change brought about by the glaucomatous damage to the optic nerve. Visual field examination, or perimetry, is useful in assessing optic nerve function. Correlating the information obtained by careful examination of the optic nerve with its function is far superior to using either piece of information alone. Primary Open Angle Glaucoma The chief pathologic feature of primary open angle glaucoma is a degenerative process in the trabecular meshwork resulting in reduction of aqueous drainage leading to a rise in intraocular pressure. Raised intraocular pressure preceded the optic disc and visual field changes by months to years. Many patients are asymtopmatic. They may have blurring of vision which they attribute to an error of refraction. Focal loss of vision such as in scotoma, unless severe is rarely noticed by the patient. Diagnosis is established when glaucomatous optic disc or visual field changes are associated with elevated intraocular pressure, a normal appearing anterior chamber angle by gonioscopy and no other reason for elevated intraocular pressure. Normal Tension Glaucoma (Low Tension Glaucoma) Some patients may have glaucomatous optic disc of visual field changes But have IOP consistently below 22 mm Hg. Pathogenesis of low tension glaucoma may involve abnormal sensitivity to IOP because of vascular or mechanical abnormalities in the optic nerve head, or this may be a purely vascular disease. Although intraocular pressure is within the normal range, reduction of intraocular pressure may still be beneficial.

95

GLAUCOMA / 96

Primary Acute Angle Closure Glaucoma When sufficient iris bombe develops to cause occlusion of the anterior chamber angle by the peripheral iris. Aqueous outflow is blocked and IOP rises causing severe pain, redness and blurring of vision. Attack is precipitated by papillary dilatation, which can occur spontaneously, by reduced illuminations in the evenings or due to medications. Patients complain of sudden severe blurring of vision, with severe pain, nausea and vomiting. Ocular findings include ciliary injection, a hazy cornea, shallow anterior chamber and an elevated IOP. Acute angle closure glaucoma is an emergency. Treatment is directed at reducing IOP. Laser peripheral iridotomy , which creates a permanent connection between the posterior and anterior chamber is the definitve treatment. In many cases, the fellow eye should receive a prophylactic laser iridotomy. Chronic Angle Closure Glaucoma Some patients with anatomic predisposition for angle closure do not develop acute rises in intraocular pressure but develop instead progressive peripheral synechia resulting in gradual rise in intraocular pressure. They may have attacks of subacute angle closure. Patients maybe asymptomatic or may have recurrent short episodes of unilateral pain, redness and blurring of vision associated with halos around lights. Attacks may resolve spontaneously. On examination, patients have elevated intraocular pressure, narrow or closed anterior chamber angle with varying amounts of peripheral anterior syneechia, optic disc and visual field changes. Intraocular pressure is controlled medically if possible. Extensive peripheral synechia may require drainage surgery. Treatment of Raised Intraocular Pressure I. Medical Treatment A. Suppression of aqueous production 1. beta adrenergic blocking agents – timolol, betaxalol, levobunolol 2. alpha adrenergic agonists- apraclonidine, brimonidine 3. carbonic anhydrase inhibitors- dorzolamide, oral acetazolamide B. Facilitation of aqueous outflow. 1. Parasymathomimetic agents – pilicarpine 2. Prostaglandin analogs – bimatoprost, latanoprost, travaprost C. Reduction of vitreous volume 1. Hyperosmotic agents – oral glycerol, IV mannitol II. Surgical and Laser Treatment A. Peripheral iridotomy, Iridectomy B. Laser trabeculoplasty C. Glaucoma Drainage Surgery 1. Trabeculectomy 2. Glaucoma valves D. Cyclodestructive Psocedures

96

GLAUCOMA / 97

SUMMARY Glaucoma is an important ocular disorder that can cause preventable blindness. It results from ganglion cell loss with resultant optic nerve damage and functional visual field loss. REFERENCES 1. American Academy of Ophthalmology Basic and Clinical Science Course Glaucoma Section 10, 2004-2005. 2. Epstein, David L. (ed). Chandler and Grant's Glaucoma. Williams and Wilkins. 1997. 3. Riordan-Eva, Whitcher, John. Vaughn and Ashbury’s General Ophthalmology , 16th Edition, New York: Lange Medical Books/ McGraw Hill, 2004 4. Ritch, R.; Shields, B.M.; Krupin, T. (eds) The Glaucomas. Mosby. 1996. SELF-TEST 1. Signs of acute angle closure glaucoma include the following, EXCEPT A. Ciliary injection B. Irregular miotic pupil C. Hazy cornea due to bedewing D. Ocular pain E. Blurring of vision 2. The following are methods for viewing the optic disc, EXCEPT A. indirect ophthalmoscopy B. Schiotz tonometry C. Direct ophthalmoscopy D. Use of posterior fundus lens and slit lamp 3. You were assigned to see 4 patients at the OPD. Which of the following findings in your patients will make you suspect that he has glaucoma ? A. 20 year old ; C/D ratio 0.5 OU, IOP = 15 mm OU B. 45 year old ; C/D ratio 0.3 OU, IOP = 22 mm OU C. 60 year old, C/D ratio 0.3 OD, 0.8 OS, IOP =12 mm OD, 28 mm OS D. 60 year old, C/D ratio 0.3 OD. 0.4 OS, IOP=16 mm OD, 18 mm OS 4. Using the direct ophthalmoscope , you will be able to examine the following EXCEPT A. scotoma B. cup disc ratio C. nerve fiber layer D. pallor of the optic nerve 5. The typical visual field defect in glaucoma is a/an A. arcuate scotoma B. nasal hemianopsia C. quadrantanopia D. any of the above, depending on the severity of the disease. ANSWERS TO SELF-TEST 1. B 2. B 3. C 4. A

5. A

97

DISORDRS OF THE OPTIC NERVE / 98

DISTURBANCE IN VISION DISORDERS OF THE OPTIC NERVE Raul D. Cruz, M.D. INTRODUCTION The optic nerve is part of the brain. Intracranial diseases frequently cause visual disturbances because of destruction of or pressure upon some portion of the optic pathways. Similarly, the eye can give important diagnostic clues to central nervous system disorders. OBJECTIVES Upon completion of this unit of instruction, the student should be able to 1. discuss the basic examination of optic nerve function (e.g. visual acuity, pupillary reactions, visual fields, ophthalmoscopy). 2. detect abnormalities of the optic nerve 3. recognize the signs and symptoms of optic nerve disorders 4. discuss the common causes of optic disc edema such as papilledema and papillitis RECOMMENDED PREPARATION The student is advised to review the anatomy and physiology of the retina, the optic nerve and basic neuroanatomic architecture of the visual pathway, as well as topographical localization of lesions before going through this material. The student is also advised to review the parts of a clinical history and the method of conducting a complete eye examination. CONTENT OPTIC NERVE The optic nerve consists of about 1 million axons originating from the ganglion cells of the retina. The optic nerve can be divided into the intraocular (anterior) and the retrobulbar (posterior) portions. The intraocular portion is commonly referred to as the optic disc (papilla) is visible with an ophthalmoscope. The retrobulbar portion starts behind the eyeball and can be further divided into the intraorbital, intracanalicular and intracranial segments. The longest division is the intraorbital portion. As it enters the optic canal it is known as the intracanalicular portion. The intracranial portion begins from the optic canal just before the optic nerves converge toward each other to become the optic chiasm. HISTORY The most important initial procedure in the evaluation of a patient begins with obtaining a complete and careful history. Pertinent details such as eye pain, headache, unilateral or bilateral involvement must not be neglected. Review of past medical, family, social and personal history and other contributory factors will certainly yield valuable information. EXAMINATION 1. Visual acuity

98

DISORDRS OF THE OPTIC NERVE / 99

An absolute requirement in any eye examination is to measure best corrected visual acuity. It is easily and best tested by the conventional Snellen’s eye chart. Corrected visual acuity should be 6/6 or better. A subnormal vision should be properly evaluated. 2. Pupils The pupillary light reflex is initiated (afferent limb: cranial nerve II) by light stimulation. The pupil normally reacts to light by constricting (efferent limb: cranial nerve III). In normal room light, the pupil is round about 2-3 mm and equal in size to the fellow eye. The pupillary reaction to light is properly done in a dim room. The response of each pupil to a bright light stimulus is observed. The response in the eye tested (direct light reflex ) and also the response in the opposite eye (consensual light reflex) is measured The pupils should equally constrict briskly both directly and consensually to a light stimulus.

Figure 1

Diagram of light reflex pathway

A dilated pupil usually indicates a cranial nerve III (efferent limb of the light reflex) lesion. If a history of trauma is elicited, an herniation of the temporal lobe compressing the third nerve must be considered. Other causes affecting the third nerve should be investigated. A benign lesion of the ciliary ganglion known as Adie’s pupil (tonic pupil) and instillation of mydriatic drops cause a dilated pupil. A miotic pupil associated with ptosis and anhydrosis is known as Horner’s syndrome. The lesion is an interruption of the sympathetic pathways anywhere from the hypothalamus to the dilator muscles of the pupil. A simple, reliable and inexpensive method of detecting an optic nerve disorder is performing the swinging light test. This test is done in a dark room using a bright light stimulus. The patient fixates on a distance object. The bright light is rhythmically directed from one eye to the other eye for several times until the examiner is satisfied with the responses. A normal response shows initial constriction of both pupils. An abnormal response is a dilation without the initial constriction. This produces a paradoxical dilation of the abnormal eye. The finding is known as a relative afferent papillary defect (RAPD or APD) or Marcus Gunn pupil.

99

DISORDRS OF THE OPTIC NERVE / 100

Figure 2. Normal response to swinging light test

Figure 3. RAPD defect in left eye using swinging light test 3. Visual Fields The visual fields measures the area of vision of each eye during central fixation. The confrontation test is a simple clinical tool routinely done to check the visual field. The method of confrontation compares the examiner’s field of vision and that of the patient. Any dissimilarity between the examiner’s and the patient’s visual field should be recorded. To obtain a more objective detailed quantitative assessment of the visual fields perimetry testing can also be done.

Figure 4. Confrontation Test

100

DISORDRS OF THE OPTIC NERVE / 101

The following terms are used to describe visual field problems 1. Scotoma - area of reduced or absent vision Central – involving fixation Cecocentral – involving fixation and the blind spot Paracentral - around the center 2. Hemianopia – loss of one half of the visual field Homonymous hemianopia – loss of the right or left half of the visual field in both eyes Bitemporal hemianpia - loss of the right half of the visual field in right eye and the left half of the visual field in the left eye. 3. Quadrantanopia – loss of one-fourth of the visual field 4. Ophthalmoscopy Direct ophthalmoscopy is an indispensable diagnostic tool in evaluating the optic nerve. Baseline recording of the optic disc should include the color, cup and size. The presence of optic disc elevation, the appearance of the arteries and veins and the presence of hemorrhages and exudates are noted. The optic disc is usually the initial landmark structure identified by the examiner. The optic disc is nonseeing and results in the physiological blind spot. The normal optic disc is pink in color and its borders should be clear, distinct and regular. At the center of the optic disc is a pale depression known as the physiologic optic cup where the retinal vessels enter the eye. The normal optic cup to disc ratio is 0.3 or 0.4. The artery to vein diameter ratio is 2:3 to 3:4. In majority of individuals spontaneous venous pulsations are seen and this indicate a normal intracranial pressure.

Figure 5. Normal optic disc

5. Other Tests Ancilliary test such as color vision, contrast sensitivity, visual evoked response, and imaging studies (ultrasound, CT scan and MRI )are very helpful and can provide useful information. SIGNS AND SYMPTOMS The main complaint of patients with optic nerve disease is poor vision. Eye findings will include subnormal best corrected visual acuity, abnormal papillary responses, including RAPD.

101

DISORDRS OF THE OPTIC NERVE / 102

Visual pathway lesions from the optic nerve, optic chiasm, optic tract, lateral geniculate body, optic radiation and the occipital lobe produce characteristic visual field defects. Optic nerve disorders result in monocular loss of vision causing various nerve fiber layer abnormalities defects such as central, cecocentral, paracentral field defects. Complete destruction of the optic chiasm causes bitemporal hemianopia. Because of the crossing of the optic nerve fibers in the chiasm, disorders affecting the visual pathway behind the chiasm results in contralateral defects. A lesion involving the optic tract produces homonymous hemianopia. Partial involvement of the optic radiation results in quadrantanopia. A total involvement of the optic radiation and occipital cortex may produce a homonymous hemianopia. Macular sparing is associated with occipital lobe lesions.

Fig 6. Visual Pathway with corresponding visual field defects

Any injury, ischemia, trauma or irritation to the optic disc can cause swelling of the axons of the optic nerve. This condition is known as optic disc edema. On ophthalmoscopy the optic disc margins become blurred and indistinct. Pallor of the optic disc or optic atrophy is the end result of severe damage to the optic nerve. Degeneration of the nerve axons occurs. This condition leads to loss of vision and carries a poor prognosis.

Fig 7. Optic disc edema

Fig. 8 Optic Atrophy

102

DISORDRS OF THE OPTIC NERVE / 103

BILATERAL OPTIC DISC EDEMA 1. Papilledema The optic nerve is an extension of the brain and surrounded by meninges. Any increase in intracranial pressure will be transmitted to the subarachnoidal space surrounding the optic nerve. Papilledema is swelling of the optic disc secondary to increased intracranial pressure. This condition is seen in intracranial mass lesions, trauma, meningitis, hydrocephalus, subarachnoid hemorrhage and conditions that obstruct the flow of the cerebrospinal fluid. True papilledema is almost always bilateral and is associated with headache, nausea and vomiting. The severity of papilledema is proportional to the increase in intracranial pressure. In the initial stages visual acuity is normal. Visual field examination shows enlargement of the physiologic blind spot. Ophthalmoscopy findings include indistinct disc margins. The swollen disc obliterates the physiologic cup and displaces the central vessels forward. There is dilation and tortuosity of the veins. Fully developed papilledema will show a severely hyperemic disc with hemorrhages, nerve fiber layer infarcts (cotton-wool spots) and exudates. Once the diagnosis of papilledema is certain, neuro-imaging studies should be done. Treatment is directed to the cause. If left untreated the persistent increase in intracranial pressure will eventually lead to optic atrophy and loss of vision 2. Malignant Hypertension Malignant hypertension occurs in patients with accelerated blood pressure elevation. ophthalmoscopic picture is similar to papilledema but result from abnormal arterial changes.

The

Fig 9. Papilledema

UNILATERAL OPTIC DISC EDEMA Papillitis Inflammatory swelling or demyelination of the anterior portion of the optic nerve is known as papilltis or anterior optic neuritis. The main symptom is a sudden painful loss of vision. It is usually unilateral and a relative afferent papillary defect (RAPD or Marcus Gunn pupil) is often easily detected. Visual field examination commonly reveals a central scotoma. Ophthalmoscopy findings show a hyperemic optic disc with blurred disc margins. It may be difficult to differentiate it from papilledema by the ophthalmoscopic appearance alone.

103

DISORDRS OF THE OPTIC NERVE / 104

In numerous instances, a spontaneous resolution occurs even without treatment. Corticosteroids given intravenously may shorten the clinical course. If the posterior portion of the optic nerve is involved, the abnormality is called retrobulbar neuritis. The ophthalmoscopic appearance of the optic disc is normal. Other causes of optic disc edema include the following 1. Toxic substances – ethambutol, isoniazid, methanol, chloramphenicol 2. Nutritional deficiencies of vitamin B 3. Vascular –anterior ischemic optic neuropathy 4. Compressive – optic nerve meningiona, orbital tumors 5. Metabolic – diabetes mellitus 6. Ocular diseasae – uveitis, vein occlusion 7. Trauma 8. Hereditary PSEUDOPAPILLEDEMA Developmental optic disc anomalies may mimic true papilledema. These congenital disc disorders are called pseudopapilledema or structural congestion of the optic disc. On the basis of ophthalmological findings alone, they may erroneously lead to a diagnosis of papilledema. The common causes of pseudopapilledema are severe hyperopia, optic disc drusen and myelination of the optic disc. Severe hyperopic (far-sighted) eyes have smaller eyeballs than normal. This causes crowding of the optic disc structures resulting in blurring of the optic disc margins. Optic disc drusen (hyaline bodies) frequently cause the optic disc borders to be indistinct. Myelination of the optic disc results in white feathery opacified areas that may surround the optic disc margins.

Fig. 10. Myelinated optic disc

REFERENCES 1. 2. 3. 4. 5.

Berson, Frank G. (ed) Basic Ophthalmology for Medical Students and Primary Care Residents. 6th edtion. American Academy of Ophthalmology. Newell, Frank. Textbook of Ophthalmology. Latest edition. Farris, Bradley G. (ed) The Basics of Neuro-Ophthalmology. Mosby Year Book. 1991 Fajardo RV, Espiritu RB, Naval CIN Textbook of Ophthalmology. JMC Press Inc. 1980.

104

DISORDRS OF THE OPTIC NERVE / 105

SELF-TEST 1.

The optic nerve consists of about _____axons from the ganglion cells of the retina A. 1,000 B. 10,000 C. 1,000,000 D. 10,000,000

2.

The optic disc is the _____ portion of the optic nerve A. intraocular B. intraorbital C. intracanalicular D. intracranial

3.

The afferent limb of the papillary light reflex is the _____ nerve A. II B. III C. V D. VII

4.

The visual field defect that includes fixation and the blind spot is a ______ scotoma A. central B. cecocentral C. altitudinal D. junctional

5.

The physiologic blind spot in the visual field represents the A. macula B. optic disc C. central retinal vessels D. retina

6.

A RAPD is elicited by doing A. perimetry B. ophthalmoscopy C. a confrontation test D. a swinging light test

7.

Optic nerve defects result in _____ loss of vision A. monocular B. temporal C. binocular D. bitemporal

8.

The following are causes of optic disc edema EXCEPT A. papilledema B. retinitis pigmentosa C. papillits D. uveitis

105

DISORDRS OF THE OPTIC NERVE / 106

9.

Pallor of the optic disc is also known as optic A. degeneration B. neuritis C. dysplasia D. atrophy

10. Severe _____ can manifest as pseudopapilledema A. presbyopia B. hyperopia C. astigmatism D. myopia COMPLETE THE TABLE PAPILLEDEMA

PAPILLITIS

ASSOCIATED SYMPTOMS BILATERALITY VISUAL ACUITY PUPILS VISUAL FIELDS OPHTHALMOSCOPY ANSWERS 1. 2. 3. 4. 5.

C A A B B

6. 7. 8. 9. 10.

D A C D C

PAPILLEDEMA

PAPILLITIS

ASSOCIATED SYMPTOMS BILATERALITY

Headache, nausea vomiting Usually

Eye pain

VISUAL ACUITY

Early, normal

Blurring of vision

PUPILS

normal

RAPD

VISUAL FIELDS

Enlargement of Blind spot Swollen disc Indistinct disc margins

Central, cecocentral Paracentral scotoma Swollen disc Indistinct disc margins

OPHTHALMOSCOPY

106

Rarely

ERRORS OF REFRACTION / 107

DISTURBANCE IN VISION ERRORS OF REFRACTION Juan Ma. Pablo R. Nañagas MD, MHA,MSNA INTRODUCTION This self-instructional material is designed for undergraduate medical students particularly LEARNING UNIT IV of the UP College of Medicine. This material is intended to serve as supplementary reading for the students as part of the unit on the Eye as an Optical Instrument and focuses primarily on providing the medical students with knowledge on how to go about recognizing and assessing patients with errors of refraction. Aside from providing patients with text on the above-mentioned subject matters, diagrams of typical cases will also be presented. Students are however encouraged to apply knowledge that they will acquire from this material to clinical cases that they will encounter during their rotation in the clinics of the department. OBJECTIVES After going through this material, the student is expected to: 1. Formulate a working definition for error of refraction and other related terms. 2. Identify the various elements in a patient’s history and ophthalmologic examination that leads to the formulation of a diagnosis. 3. Differentiate the various errors of refraction and related conditions according to type. 4. Based on information given, be able to analyze and interpret provided data to formulate diagnosis. RECOMMENDED PREPARATION The student is advised to review the anatomy and physiology of the cornea, lens, and other refractive elements of the eyeball before going through this material. He is further advised to review the parts of a clinical history, as well as the general ophthalmologic examination to better appreciate this material. INTENDED USERS This program was designed primarily for LEARNING UNIT IV student of the UP College of Medicine.. These students are expected to have completed basic units in ophthalmology, namely Ophthalmic anatomy and Physiology, Objective and Subjective Examination of the Eye and Common Out-Patient Disorders. CONTENT PART I. WHAT IS AMETROPIA OR ERROR OF REFRACTION? This is a condition where the refractive elements of an eye at rest are unable to focus light rays from 6 meters or more onto the retina. It may be caused by abnormalities in length of the eyeball or of the refractive elements of the eye (mainly the cornea and crystalline lens) or both.

107

ERRORS OF REFRACTION / 108

PART II. DIAGNOSIS OF ERRORS OF REFRACTION There are three basic elements that one would need to consider to be able to formulate a diagnosis of error of refraction. 1. History – a comprehensive history is an important component of a work up for error of refraction. 2. Ocular Examination – basic eye examination comprised of the following parts should be done on each patient. ƒ Gross eye examination – slit lamp examination ƒ Visual acuity testing ƒ Intraocular pressure determination ƒ Movement of Extraocular muscles ƒ Funduscopic Examination 3. Ancilliary Examinations – manual or automatic refraction is an examination routinely performed to determine the presence and type of error of refraction. Other special examinations may include biometry, keratometry and corneal topography. Each of these aspects will be discussed subsequently. HISTORY TAKING IN A PATIENT WITH ERROR OF REFRACTION Extracting the history from a patient with probable error of refraction can be very useful. The patient’s history guides the clinician in arriving at a complete diagnosis, particularly as to the possible type of error of refraction. Furthermore, the history can provide the clinician with an idea as to the visual needs of the patient and the appropriate treatment modality. The clinician should ask each and every patient suspected to have an error of refraction the following questions. Answers to the following questions serve as aids in the formulation of a complete diagnosis. 1. What is your patient’s chief complaint? The most common presenting complaints of patients with errors of refraction include blurring of vision for distance, for near, or both. Clear near vision but blurred distance vision indicates near sightedness or myopia.. Hyperopics (far sighted individuals) may complain of early visual fatigue when performing visual tasks (especially at near). Headache especially after prolonged eye use is common with hyperopics and astigmatics but non-specific and will have to be differentiated from other causes. 2. How long has this problems been going on? The duration of the problem should be extracted from the patient. Errors of refraction usually present with a prolonged history of their complaints. Whether it is recurrent or progressive should also be noted. One should be aware of a condition of middle age called presbyopia where the lens of the eye gradually loses its ability to focus at near objects. . 3. Which eye is involved? Does the problem involve one eye or both eyes? Significant difference of refraction between the two eyes may be the cause of amblyopia or a lazy eye. 4. Are there other associated eye problems?

108

ERRORS OF REFRACTION / 109

Elicit from the patient whether or not there is any history of redness, ocular pain, glare or photophobia, trauma or any form of eye surgery in the past. 5. Does the patient have any prior consultations? One should be able to determine a patient’s previous correction of his/her error of refraction to help determine the course of the condition. 6. Other aspects of the patient’s history that should be considered include: ƒ Family History. Is there a history of any similar illness in the family? Genetic factor strongly determine refractive errors of the eye. Is there any history of any hereditary illness like diabetes? Changing blood sugar levels may affect a person’s refraction. ƒ Social History. What is the patient’s occupation? What are the patient’s usual visual tasks? ƒ Medical History. Has the patient suffered any form of illness in the past particularly diabetes? Is she under any form of medication for any illness? These may affect vision. OPHTHALMOLOGIC EXAMINATION OF THE PATIENT The basic tools that one would require in conducting an examination of the patient with suspected error of refraction include the following:

♦ ♦ ♦ ♦

Visual acuity charts – both for distance and near vision Occluder with pin-hole Penlight – for gross examination of the eye Ophthalmoscope – used for fundus examination

COMMON OCULAR FINDINGS IN AMETROPIA Typically, the patient with error of refraction will present with the following findings: 1. Visual Acuity. As has been mentioned earlier, most patients with would present with reduction in vision either for far, near, or both. Snellen charts are commonly used for distance vision testing and Jaeger or Snellen equivalent cards for near vision. If the vision improves when the patient looks through the pinhole, the patient most probably has an error of refraction that can be corrected with lenses. 2. Intraocular Pressure. Most patients would typically present with normal intraocular pressures 3. Extraocular Muscle Movement. The extraocular muscles are usually not involved in errors of refraction and a majority of patients will exhibit full movement on all directions of gaze. Some forms of heterophoria or heterotropia are associated with errors of refraction.

109

ERRORS OF REFRACTION / 110

4. Funduscopic Findings. Patients wherein the error of refraction is not severe will present with normal findings. In cases where the eyeball is elongated the fundus and optic nerve head may exhibit some changes 5. Retinoscopy will reveal the following findings:

“With” movement of reflex with the streak is seen in hyperopia and small myopias, “against” movement in myopia. Retinoscopy has achieved neutrality in A but still ‘with’ in B, indicating different refracting powers in the two meridia, a sign of astigmatism. ANCILLARY EXAMINATIONS Commonly, additional examinations may be requested to aid in the diagnosis of some conditions. The more commonly requested ancillary procedures include: 1. Keratometry The keratometer is an instrument that can measure central anterior corneal curvature. It can be used to check the type and amount of astigmatism. It is also used in fitting contact lenses, a form of correction for errors of refraction. 2. Corneal Topography It is a sophisticated instrument that produces a color-coded topographic map of the cornea that shows the pattern of the corneal curvature. It usually functions as a keratometer as well, measuring corneal curvatures. It demonstrates irregularities of curvature like keratoconus. 3. Biometry This instrument using sound waves provides measurement of the axial length of the eyeball, a determining factor of the refractive condition of the eye. REVIEW QUESTIONS: PART I & II 1. Error of refraction or ametropia refers to a condition where A. light rays are blocked from the retina B. light rays from a source more than 6 meters from an eye at rest do not focus on the retina C. there is an abnormality in the transparency of the cornea. 110

ERRORS OF REFRACTION / 111

2. In error of refraction a presenting symptom A. may be blurred distance vision, near vision, or both. B. is color aberration. C. is nasal scotoma 3. Intraocular pressure in plain errors of refraction is usually A. normal B. high C. low 4. Error of refraction has A. No hereditary pattern B. Strong genetic influence 5. Retinoscopy is A. the same as funduscopy B. an objective means of measuring errors of refraction C. a means to take retinal photographs to determine retinal changes due to errors of refraction 6. If the poor vision of a patient improves when viewing the test chart through a pin-hole A. it is the pupil that is at fault B. there may be error of refraction C. dark glasses are the cure 7. Measurement of refractive errors A. cannot be done by machines B. is usually very subjective C. can be performed using automatic refractors 8. In middle age, one may A. gradually lose the ability to focus for near or presbyopia B. usually develop myopia C. usually develop hyperopia 9. Normal or emetropic eyes looking at an object 20 or more feet away A. need to accommodate to focus light on the retina B. need to squint to focus light on the retina C. should be able to see appropriate sized Snellen letters with the eye at rest 10. If a diabetic consults for sudden blurring of vision after a rise in sugar level A. it is part of the aging process for diabetics B. the refractive state of the eye may have been affected by the sugar level C. it is surely diabetic retinopathy D. keratometry will demonstrate the changes in corneal curvature due to sugar levels

111

ERRORS OF REFRACTION / 112

PART III. CLASSIFICATION OF AMETROPIAS (ERRORS OF REFRACTION AND ASTIGMATISM) Diagrammatic Classification of Ametropias

The emmetropic eye focuses light rays from infinity on the retina even at rest or without accommodating.

The myopic eye focuses light rays from infinity in front of the retina. An object at a finite distance focuses on its retina.

The hyperopic eye theoretically focuses light rays from infinity behind the retina. It can only focus convergent light rays on the retina. ASTIGMATISM In astigmatism the refracting surface (usually the cornea) is toroidal (like the surface of an American football) rather than spherical, and therefore the refracting power of the surface is not the same for all meridians. a. In regular astigmatism, the refractive power changes successively from one meridian to the next and each meridian has a uniform type of curve. B. With the rule and against the rule astigmatism The term “with the rule” and “against the rule” refer to the position of the principal meridians. In with the rule, the vertical meridian is steepest and a correcting plus cylinder is located at or near axis 90°. In against the rule astigmatism, a correcting plus cylinder is located to or near 180° and the horizontal meridian is steepest.

112

ERRORS OF REFRACTION / 113

REVIEW QUESTIONS , PART III 1.

When the image of an object at infinity falls in front of the retina, the patient is A. hyperopic B. myopic C. toric D. phthsic 2. In astigmatism A. the curvature is the same in all meridians B. the curvature is flatter than usual C. the refracting power of the surface is not the same in all meridians D. the curves of the surface are convex 3. A patient can read books clearly but cannot read from a whiteboard at 20 feet away. A. He is probably normal B. He may be myopic C. He may be hyperopic D. He may have compound hyperopic astigmatism 4. On visual acuity testing, a 20 yr. old patient had 6/6 vision but difficulty with reading Jaeger 1 print. A. This is presbyopia B. He may be hyperopic C. He may be myopic D. This is normal 5. If the vertical meridian of the cornea is steepest A. it is ‘with the rule’ astigmatism B. it is against the rule’ astigmatism C. it is called ‘natural’ astigmatism EXERCISE

Case 1. A 12 year old boy complains that he cannot see the blackboard from his new seat in class located at the back row. The mother accompanying the boy wears glasses. Ophthalmologic examination showed: V OU: 20/200 or 6/60 improved to 6/7.5 with pinhole. Intraocular pressure: soft OU EOMs: full FOU: normal 1. With the given information would it be safe to say that the patient has an error of refraction? 2. What type would it be, hyperopia or myopia? 3. What examination can measure the error and lead to the prescribed correction? Case 2. A 24 year old female has been getting headaches after she finishes overtime work at her computer in a bank. She also says her eyes feel tired. She is not on any medication and her BP taken at the office was normal.

113

ERRORS OF REFRACTION / 114

Ophthalmic examination: V OU: 20/20 or 6/6, Jaeger 1 with difficulty Intraocular pressure: soft, OU EOMs: Full F OU: Normal 1. With the given information would you say the patient may have an error of refraction? 2. What type would it be, hyperopia, myopia, or astigmatism? SUMMARY It was the primary objective of this self-instructional material to provide the reader with a guide on how to diagnose patients with error of refraction. Basic examination, symptomatology, and classification of the different errors were presented. RECOMMENDED FOLLOW UP It is recommended that the students be given demonstration sessions on how to properly conduct history taking and ophthalmologic examination of patients. Following this exercise, it is further recommended that the students be provided clinical sessions to allow them to see actual cases of patients with errors of refraction. CONCLUSION As medical practitioners, you may, in the future encounter patients who will seek consultation for eye problems. One should bear in mind that many who complain of blurring of vision may have an error of refraction. It is the most common cause of visual disability. It is therefore your role to be able to properly recognize these conditions and differentiate them from permanently disabling visual conditions. Referral to an ophthalmologist or optometrist is necessary for proper correction through spectacles, contact lenses, or refractive laser procedures. Early intervention, especially in children, can prevent permanent amblyopia if anisometropia (significant difference in amount or kind of refractive error between the two eyes) is present. REFERENCES 1. Optics, Refraction and Contract Lenses. Basic and Clinical Science Course, Section 2. California: American Academy of Ophthalmology, c1990. 2. Espiritu, Romeo. Ophthalmologic Optics. Manila: Department of Ophthalmology & Visual Sciences, c2001. ANSWERS TO REVIEW QUESTIONS Part I and II Part III 1. D 6. B 2. 2. A 7. D 3. 3. A 8. A 4. 4. B 9. C 5. 5. B 10. B 6.

Case 1 1. Yes 2. Myopia 3. retinoscopy or autorefraction Case 2 1. Yes 2. Hypeopia or astigmatism

B C B B A

114

THE RED EYE / 115

THE RED EYE Leo Cubillan, MD, MPH INTRODUCTION The red eye is one of the most frequent clinical presentations of ocular disorders. The medical student will be able to differentiate these disorders from each other by asking certain questions from their patients during history taking and looking for specific signs when performing the ocular examination. OBJECTIVES After the completion of this learning material, the student sgould be able to : 1. illustrate the algorithm on the differential diagnoses of a red eye based on the following signs and symptoms: pain, eye discharge and itchiness. 2. enumerate the clinical clues, etiology / pathogenesis, signs and symptoms, diagnostic work-up, treatment, and complication of the common causes of a red eye a. Viral Conjunctivitis b. Allergic Conjunctivitis c. Dry Eye d. Bacterial Conjunctivitis e. Microbial Keratitis f. Acute Glaucoma g. Uveitis RECOMMENDED PREPARATION Before going to this material, the student must have previous knowledge of anatomy and physiology of the eye and the methods of history taking and ocular examination. CONTENT DIFFERENTIAL DIAGNOSES OF A RED EYE Red eye is one of the most common serious eye symptom of patients with eye disease. Red eye is seen as a result of dilation of conjunctival or scleral blood vessels in response to inflammation or infection. Subconjunctival hemorrhage as a result of injury to a conjunctival blood vessel also present as a red eye. The following are the differential diagnoses of a red eye: 1. Conjunctivitis a. Infectious i. Bacterial ii. Viral b. Non-infectious i. Allergic ii. Dry Eye iii. Toxic or Chemical Reaction iv. Contact lens use

115

THE RED EYE / 116

v. Conjunctival Neoplasm vi. Foreign Body 2. 3. 4. 5.

Uveitis Episcleritis /Scleritis Acute Glaucoma Keratitis a. Infecious i. Bacterial ii. Viral iii. Fungal iv. Acanthamoeba b. Non-Infectious i. Recurrent Epithelial Erosion ii. Foreign Body 6. Eyelid Abnormalities a. Entropion / Trichiasis b. Lagophthalmos 7. Orbital Disorders a. Preseptal and orbital cellulites b. Idiopathic orbital inflammation ALGORITHM FOR RED EYE Ophthalmologist are the experts in the diagnosis and treatment of eye diseases. In the Philippines, however, access to an ophthalmologist may not be easy. The primary care physician may still be the best first line medical worker in the identification and initial treatment of eye disease. This algorithm is presented to aid the primary care physician in the diagnosis of the most common cause eye disease presenting as a red eye. The common symptom of eye pain, eye discharge and itches are used for this algorithm. EYE PAIN pain- mahapdi not pain 2 kinds: associated with headache when u woke up at night corneal ulcer open eye, seen by doctor no pain, microbial keratitis- with pain

The first step in the algorithm is to determine whether the patient presenting with a red eye has eye pain or not. Eye pain may be described as a sharp localized pain, pain when exposed to bright light or a severe eye pain radiating to the head. The symptom of eye pain divides the common differential diagnoses of eye pain into two groups.

116

THE RED EYE / 117

EYE PAIN WITH DISCHARGE

pain affecting cornea, discharge associated with infection photophobia there is pain with presence of bright light inflammation of cilliary body causing pain with

For patients with red eye and eye pain, the next question would be the presence of absence of eye discharge. If there is eye discharge, microbial keratitis or infection of the cornea may be considered. The presence of abundant nerve fiber endings in the cornea (CN V) causes in the pain when infection is present. PHOTOPHOBIA

If there is no eye discharge in a patient with eye pain, the next question would be the presence or absence of photophobia. Photophobia is usually described as a dull eye pain when exposed to bright lights. This is a pain due to the pupillary constriction of an inflamed iris. If photophobia is present, uveitis or eye inflammation, may be considered. In the absence of photophobia, acute glaucoma may be entertained. The eye pain in glaucoma is moderate to severe and radiates to the head. NO EYE PAIN

117

THE RED EYE / 118

Dry eye syndrome may be considered in patients with red eye in the absence of eye pain and eye discharge NO EYE PAIN WITH DISCHARGE

Red eye patients with no eye pain but with mucopurulent discharge may have bacterial conjunctivitis. If the discharge is watery, viral conjunctivitis is a differential diagnosis. ITCHY EYE

The symptom of itchiness is asked among patients with red eye, no eye pain and with watery discharge. If itchiness is present, allergic conjunctivitis may be considered. VIRAL CONJUNCTIVITIS CLINICAL CLUES

118

THE RED EYE / 119

Viral conjunctivitis is the most common cause of acute onset of eye redness. The main feature of this disease is the presence of watery discharge in the early part of the course. The most important question in the history that would point to this disease, is the history of exposure to other patients with sore eyes. ETIOLOGY / PATHOGENESIS Epidemic keratoconjunctivitis which is the more common type of the viral conjunctivitis is caused by adenovirus types 8, 19, 29 and 37. Spread is with direct contact from eye discharge. The virus may not be neutralized by alcohol so the best preventive measure is hand washing. SIGNS AND SYMPTOMS

Onset of symptoms is acute. Duration may be from few days to 2-4 weeks. It usually starts on one eye and develops to become bilateral after few days. The severity of symptoms is worse on the first eye. During the early course of the disease, the discharge is watery. In some cases, discharge may become mucopurulent if it worsen or there is a secondary bacterial infection. DIAGNOSTIC WORK-UP The diagnosis of adenoviral conjunctivitis is mainly clinical based on the history, character of the discharge and the presence of follicular conjunctival reaction seen under the biomicroscope. TREATMENT Hand washing and avoiding direct contact to eye discharge is the best way to prevent the spread of viral conjunctivitis. There is not treatment needed. However, antibiotic eye drops may be given in patients with secondary bacterial infection or as a prophylaxis for bacterial infection. Mild steroid drops may be given to help reduce inflammation. COMPLICATION

119

THE RED EYE / 120

In majority of the patients, the disease resolves without sequelae. In few cases, subepithelial opacities may develop in the cornea. This may cause blurring of vision. The subepithelial opacities may persist for weeks or months but may heal without scars. ALLERGIC CONJUNCTIVITIS CLINICAL CLUES

Allergic conjunctivitis is a chronic or recurrent eye disease characterized by eye redness, itchiness and watery or stringy discharge. Eye itchiness is the most prominent feature. ETIOLOGY / PATHOGENESIS The etiology of this disease is immunologic in nature. Often, it is difficult to identify the specific allergen. In the hayfever type of allergic conjunctivitis, it is mainly a type I hypersensitivity reaction to airborne allergens. In atopic keratoconjunctiviis, the patient has a hypersensitive immune system which reacts to many antigens. In most patients, there may be an atopic predisposition or the patient has atopy. SIGNS AND SYMPTOMS

Eye itchiness is a prominent feature of the disease. Discharge is watery, mucoid and distinctively stringy. On ophthalmic examination, there is a pale papillary reaction on the upper bulbar conjunctivae. In atopic keratoconjunctivitis, there is a periocular scaly skin with thickening of the eyelids (allergic shiners).

120

THE RED EYE / 121

DIAGNOSTIC WORK-UP

Conjunctival cytologic studies from conjunctival scraping would show eosinophils. TREATMENT Treatment involves removal of environmental triggers. Cold compress and mast cell stabilizers / antihistamine eye drops may help control the symptoms. In moderate to severe cases, short term topical steroids are needed. COMPLICATION

In the vernal type of allergic conjunctivitis it often occurs to young children which would last for several years. In theses cases, when steroid drops are overused, cataract may develop early as a complication of the steroids.

121

THE RED EYE / 122

DRY EYE CLINICAL CLUES

Dry eye disease or keratoconjunctivitis sicca is a common ophthalmic condition seen among the elderly. It usually presents with foreign body sensation and mild eye redness. ETIOLOGY / PATHOGENESIS The main etiology for dry eye is the decreased tear production seen among the elderly. Other causes of dry eye include increased tear evaporation in patients with inability to completely close the eyelids and unstable tear film in patients with meibomitis. SIGNS AND SYMPTOMS Patients usually complain of a sandy and gritty sensation associated with slight eye redness. The symptoms are worse with wind and dry climates. DIAGNOSTIC WORK-UP Shirmers testing with or without anesthesia is done to confirm dry eye. Standardized strips of filter papers are used to absorb tears. Consistent measurements of less than 5 mm of wetting at 5 minutes may indicate a dry eye. Tear break-up time may also be done. Flourescein dye is instilled into the eye and the surface of the tear film is observed for areas of disruption. Normal tear break-up time is generally greater than 10 seconds. TREATMENT Treatment is mainly with the use of aqueous tear replacement or artificial tears. In severe cases, punctual occlusion may be don.

122

THE RED EYE / 123

BACTERIAL CONJUNCTIVITIS CLINICAL CLUES

Conjunctivitis caused by bacteria is usually seen with mucoid to muco-purulent discharge. Among children, it may be associated with upper respiratory tract infection (URTI). ETIOLOGY / PATHOGENESIS

In children with URTI, the most common etiology is hemophilus influenza. Among newborns, Chlamydia or gonococcocus may be the etiologic agent. SIGNS AND SYMPTOMS Most often, bacterial conjunctivitis is unilateral and is acute in onset. There is a mucoid to mucopurulent discharge associated with a red eye but no blurring of vision. DIAGNOSTIC WORK-UP Conjunctival scrapings among children with bacterial conjunctivitis and URTI might reveal gram negative coccobacilli. Bacteria from conjunctival scraping may be cultured on a blood agar plate. TREATMENT Antibiotic eye drops are used for bacterial conjunctivitis. Oral antibiotics are also administered in patients with hemophilus, chlamydia or gonococcal conjunctivitis.

123

THE RED EYE / 124

MICROBIAL KERATITIS CLINICAL CLUES

Microbial keratitis is an acute infection of the cornea with painful red eye associated with mucoid to muco-purulent eye discharge. White lesion is seen on the cornea. ETIOLOGY / PATHOGENESIS Pneumococcus, a gram positive organism is the most common cause of bacterial keratitis. Among contact lens wearers, pseudomonas is a common and dreaded organism. In some patients with a history of trauma and steroid eye drop use, fungal etiology may be considered. SIGNS AND SYMPTOMS Eye redness is moderate to severe associated with eye pain. Mucoid to muco-purulent discharge is seen. If the lesion is on the central cornea, the patient would present with blurring of vision. DIAGNOSTIC WORK-UP Corneal scraping is done to determine the etiologic agent. Gram stain is used to initially identify the bacteria but definitive diagnosis is achieved with culture studies in blood agar-plate and blood heart infusion medium. Sensitivity studies are done to determine the most sensitive antibiotic agent. TREATMENT Topical antibiotics are the mainstay of treatment. In these cases, antibiotic drops are used every 15 min during the first few days of treatment. In moderate to severe cases, keratectomy is done. COMPLICATION

124

THE RED EYE / 125

In severe microbial keratitis such as those caused by pseudomonas, corneal perforation occurs in few days without treatment. In these cases, corneal transplantation may be needed to save the eye. ACUTE GLAUCOMA CLINICAL CLUES

In acute glaucoma, the most prominent feature is a red painful eye associated with headache. There is blurring of vision and occasionally, rainbow haloes are seen (iridescent vision). ETIOLOGY / PATHOGENESIS

The eye pressure is abnormally elevated (intraocular pressure greater than 23 mmHg) due to an acute obstruction in the outflow mechanism of aqueous humor. Most often this is secondary to angle closure. Angle closure glaucoma is more prevalent among Asians including Filipinos because genetically shallower irido-corneal angle. The outflow mechanism is located at the area in this angle which when obstructed causes an increase in eye pressure. SIGNS AND SYMPTOMS When the eye pressure is acutely elevated, the patient would experience sudden onset of eye pain associated with headache. With the elevated eye pressure, the cornea becomes edematous resulting in an iridescent and blurring of vision.

125

THE RED EYE / 126

DIAGNOSTIC WORK-UP Applanation tonometry is done to measure the eye pressure. A gonioscope lens is also used to visualize and evaluate the angle structure. Visual field examination is done to determine the extent of damage to the nerve fiber layer of the optic nerve. TREATMENT Acetazolamide or hyperosmotic oral solutions may be used to lower the eye pressure initially. Topical ocular hypotensive agents are also used. In severe cases, laser iridectomy or glaucoma filtering surgery may be done. COMPLICATION

When the glaucoma becomes chronic or left untreated, there is damage to the optic nerve causing initial blurring of vision of the peripheral vision which eventually leads to blindness. UVEITIS CLINICAL CLUES

Uveitis is an ocular inflammation characterized by eye redness around the cornea associated with sensitivity to bright lights (photophobia)

126

THE RED EYE / 127

ETIOLOGY / PATHOGENESIS Almost half of the uveitis are idiopathic in nature. In some cases, uveitis is associated with systemic disease such as Vogt-Koyanagi-Harada, Bechet’s and collagen vascular disease. Others may be infectious in nature such as those caused by herpes virus and mycobacterium tuberculosis. SIGNS AND SYMPTOMS

Most patients complain of eye redness associated with photophobia. On biomicroscopic examination, keratic precipitates are seen on the inner surface of the cornea which indicated inflammatory reaction in the anterior chamber of the eye. Iris adhesion to the lens (posterior synechia) may also be seen. DIAGNOSTIC WORK-UP Laboratory work-up is directed to the most likely systemic associated disease. Although most uveitis are idiopathic, the identification of an associated systemic disease would help in the prognosis and treatment of the disease. TREATMENT Steroid is the mainstay of treatment in patients with uveitis. Topical steroid drops are used for uveitis located in the anterior part of the eye. Periocular steroid injection is recommended for patients with uveitis in the posterior pole. In patients with bilateral disease or there is a systemic evidence of inflammation, oral steroids are used. Immunosuppresive agents are also given in moderate to severe cases. COMPLICATION Moderate to severe uveitis is a blinding disease. Common complications include cataract and secondary glaucoma.

127

THE RED EYE / 128

OTHERS EPISCLERITIS / SCLERITIS

Epislceritis or scleritis, present with a red eye associated with eye pain. Some are associated with collagen vascular diseases. Topical steroids and oral NSAIDs are often prescribed for this condition. CONTACT LENS RELATED EYE REDNESS

Eye redness may be seen among contact lens wearer. This results from over-wear of the contact lenses which causes decrease in oxygen supply to the cornea. This condition resolves with rest from contact lens wear. Antibiotic eye drops may be given as a prophylaxis for eye infection. CORNEAL ABRASION / FOREIGN BODY

128

THE RED EYE / 129

Corneal abrasion from trauma presents with sudden eye pain and eye redness. Topical antibiotics are used with eye patching. This resolves in 24 to 48 hours. ENTROPION / TRICHIASIS

Entropion and trichiasis seen among the elderly presents with eye redness due to misdirected lashing which irritates the cornea. Removal of the misdirected lashes is done as an initial treatment. Permanent treatment may require cautery of the root of the eye lashes or surgery of the eyelids. REFERENCES 1. Vaughan DG, Asbury and Riordan-Eva P. General Ophthalomology. 14th ed. Appleton & Lange: Stamford, CT. c1995. 2. Chern K and Zeagans M. eds. Ophthalmology Review Manual. Lippincott Williams and Wilkins: Philadelphia, PA. c2000. SELF-TEST 1. A 20 year old female consulted for eye redness. A group of medical students were assigned to get the history of medical illness. Student A asked if the patient experienced eye pain. The patient answered “no” to the first question. Student B was able to elicit a history of watery eye discharge. The patient told Student C that she did not experience eye itchiness. What eye condition would you consider as your initial impression. a. Dry Eye b. Microbial Keratitis c. Viral Conjuncitivitis d. Allergic Conjunctivitis e. Acute Glaucoma 2. A 43 year old man consulted for sudden eye redness associated with eye pain and rainbow haloes in his vision. What eye condition would you consider as your initial impression based on the history? a. Dry Eye b. Microbial Keratitis c. Viral Conjuncitivitis d. Allergic Conjunctivitis e. Acute Glaucoma ANSWERS 2. E 1. C 129

UVEITIS AND SCLERITIS / 130

RED EYE Uveitis and Scleritis Teresita R. Castillo, MD, MHPEd INTRODUCTION This self-instructional material is designed for undergraduate medical students. This material is intended to serve as supplementary reading for the students as part of the unit on THE RED EYE and focuses primarily on providing the medical student with knowledge on how to go about recognizing and assessing patients with uveitis and scleritis. Aside from providing the students with text on the above-mentioned subject matter, pictures of typical cases will also be presented. Students are however encouraged to apply knowledge that they will acquire from this material to actual and simulated clinical cases that they will encounter during their rotation in the clinics of the department. OBJECTIVES After going through this material, the learner is expected to: 1. 2. 3. 4.

Formulate a working definition for uveitis, scleritis and related terms. Identify the various elements in a patient’s history and ophthalmologic examination that leads to the formulation of a diagnosis of uveitis and scleritis. Classify the various inflammatory conditions involving the sclera and the uvea according to their location, severity, course, pathology and etiology. Based on information given, be able to analyze and interpret provide data to make a diagnosis. RECOMMENDED PREPARATION

Students are advised to review the anatomy and physiology of the sclera and the uveal tract before going through this material. Students are further advised to review the parts of a clinical history, as well as the general ophthalmologic examination so as to better appreciate this material. INTENDED USERS This self-instructional material was designed primarily for LEARNING UNIT IV students for the OSI curriculum of the UP College of Medicine. It is a part of a series of modules on THE RED EYE. Thus, students are expected to have completed the materials on the overview of THE RED EYE. It is also expected that students already possess basic knowledge on ophthalmic anatomy and physiology, the basic eye examination, ocular symptomatology and ocular pharmacology. CONTENT PART I. WHAT IS UVEITIS? Uveitis is a nonspecific term used to denote intraocular inflammation. Although all layers of the eye may be involved by inflammation, strictly speaking, the term uveitis is reserved for inflammation involving the uveal tract, that is, the iris, ciliary body and the choroid. PART II. DIAGNOSIS OF UVEITIS There are four basic elements that one would need to consider to be able to formulate a diagnosis of uveitis. 130

UVEITIS AND SCLERITIS / 131

1. History – a comprehensive history is the one most important component of the uveitis work-up 2. Ocular Examination – basic eye examination comprised of the following parts should be done on each patient ƒ Gross eye examination – slit lamp examination ƒ Visual Acuity testing ƒ Intraocular pressure determination ƒ Movement of Extraocular Muscles ƒ Funduscopic Examination 3. Ancillary Examinations – special ophthalmologic and laboratory examinations are done as aids in the formulation of an etiologic diagnosis for uveitis patients 4. Systemic Examination – general systemic examination is done in cases wherein the clinician suspects the uveitis to be part of the presentation of a systemic problem. Each of these aspects will be discussed subsequently.

HISTORY TAKING IN THE UVEITIS PATIENT Extracting a comprehensive history from a uveitis patient can not be overemphasized. The patient’s history guides the clinician in arriving at a complete diagnosis, particularly as to the possible etiology of the condition. Furthermore, the history can provide the clinician with an idea as to the patient’s possible response to treatment regimen. The following are important questions regarding the patient’s current illness that the clinician should ask each and every patient suspected to have uveitis. Answers to the following questions serve as aids in the formulation of a complete diagnosis.

1. What is your patient’s chief complaint? The two most common presenting complaints of patients with uveitis include changes in vision and floaters. Often, the patient consults primarily because of blurring of his vision. Vision changes may come in the form of reduction in vision or distortion in vision. In addition to blurring of vision, patients may also complain of photophobia or eye pain in high illumination. Floaters on the other hand refer to black or white floating “objects” that the patients appreciate in their field of vision. These are often described by patients as “flies” or “insects” which move about in their field of vision. The onset of these changes should likewise be noted, that is whether or not they occurred suddenly or progressively over a prolonged period of time.

2. How long has this problem been going on? The duration of the problem should be extracted from the patient. Furthermore, it should also be determined whether or not a similar problem has occurred in the past.

3. Which eye is involved? Does the problem involve one eye alone or both eyes? If both eyes are involved, does the patient experience his or her symptoms in both eyes simultaneously or do they occur in one eye at a time?

4. Are there other associated eye problems? Elicit from the patient whether or not there is any history of redness, ocular pain, glare or photophobia, trauma or any form of eye surgery in the past.

5. Does the patient have any prior consultations? Where medications used? One should be able to determine how a patient responds to specific forms of therapy in the past so as to give the present physician an idea as to the patients’ prognosis. 131

UVEITIS AND SCLERITIS / 132

6. Other aspects of the patient’s history that should be considered include: Family history. Is there a history of any similar illness in the family? Is there any history of any hereditary illness? • Social History. Does the patient have any exposure to pets? Inquire also as to the patient’s diet, any history of alcohol intake, smoking, drug intake or travel. • Medical History. Has the patient suffered any form of illness in the past? Inquiry as to the presence of any systemic symptoms such as joint pains, oral ulcers or gastrointestinal problems to name a few should also be made. • Sexual History. The presence of a history of any sexually transmitted disease should likewise be elicited from the patient. •

OPHTHALMOLOGIC EXAMINATION OF THE PATIENT The basic tools that one would require in conducting an examination in the uveitis patient include the following: • Visual acuity charts – both for distance and near vision • Tonometer – used to determine the patient’s intraocular pressure • Penlight – for gross examination of the eye • Ophthalmoscope – used for fundus examination • Slit Lamp Biomicroscope – instrument used in the assessment of inflammatory findings in the anterior segment of the eye

D B E A

F

C

Figure 1. Basic instruments for an eye examination. A–Snellen Chart, BNear vision chart, C-penlight, D-tonometer, E-ophthalomoscope, F-slit lamp biomicroscope

COMMON OCULAR FINDINGS IN UVEITIS Typically, the patient with uveitis would present with the following findings: 1. Visual Acuity. As has been mentioned earlier, most patients with uveitis would present with reduction in vision. 2. Intraocular Pressure. Most patients would typically present with a very soft or hypotonic eyeball as a result of involvement of the reduced aqueous humor production in the ciliary body as part of the inflammatory process. On the other hand, there are patients with prolonged uveitis who may have developed complications of the disease process, and thus present with elevation of their intraocular pressures, a condition otherwise referred to as secondary glaucoma. 3. Extraocular Muscle Movement. Since the extraocular muscles are usually not involved in the inflammatory process, patients would exhibit full movement on all directions of gaze. 4. Funduscopic Findings. Patients wherein the inflammation is confined to the anterior segment would have normal funduscopic findings. In cases however, wherein the posterior pole is involved, the patients could present with any of the following: 132

UVEITIS AND SCLERITIS / 133 • • • • •

Cystoid Macular Edema – presents as either an elevation of the macular area or dull foveal reflex. Vasculitis – generally presents in patients with posterior uveitis and often comes in the form of perivascular sheathing or narrowing or obliteration of the retinal blood vessels. Granulomatous nodules – represents accumulation of inflammatory cells which may be located along the vitreous base or in the retinal pigment epithelium. Retinal Pigment Changes – generally represent areas of previously active inflammation; these lesions are typically referred to as retinal scars. Retinal Detachment – may come in the form of diffuse serous detachment of the retina or as multifocal areas of detachments.

Fig. 2. Cystoid macular edema

Fig. 3. Perivascular sheathing

Fig. 4. Severe periphlebitis

Fig. 5. Multiple retinal granulomas

Fig.6. Old syphilitic neuroretinitis

Fig.7. Multifocal detachment of the retina

5. Slit lamp findings. Patients with uveitis, particularly those wherein the primary location of the inflammatory process is in the anterior segment would typically present with the following findings. 133

UVEITIS AND SCLERITIS / 134 •

Ciliary perilimbal flush. This is characterized as redness of the eye that is more marked in the area around the limbus and decreases towards the fornices of the eyes. Redness occurs as a result of congestion of the deeper ciliary blood vessels. This type of redness does not blanch with pressure, nor does it diminish with the use of vasoconstrictors. Figure 8 shows a typical picture of a patient with perilimbal flush as contrasted to a patient whose eye redness is secondary to conjunctival congestion (Figure 9) wherein redness is more diffuse in nature.

Fig. 8. Ciliary injection



Fig,9, Conjunctival congestion

Keratic Precipitates (KP’s). These are deposits of inflammatory cells on the corneal endothelium. Although more often confined in the inferior portion of the cornea, they may also be diffusely distributed in certain forms of uveitis. Their appearance may also vary as to their size (fine or large) and degree of pigmentation (Figure 10). The size of KP’s provides the clinician with a clue as to the pathologic classification of the disease as will be further discussed in a later part. The presence of pigmentation, on the other hand, would generally indicate that the condition is a chronic one.

A B

C B

B

D B

Fig.10. Different kinds of Keratic Precipitates (KP’s) A. mutton fat KP’s B. Medium sized KP’s C. old pigmented KP’s D. old KP’s with ground glass appearance



Pupil abnormalities. Patients with anterior uveitis present with constricted irregularly shaped pupils as a result posterior synechiae formation or adhesions between the pupil and the anterior capsule of the lens (Fig 11). In some instances, an inflammatory membrane may also cover the pupil (Fig 12).

Fig.11. Posterior synechiae following an attack of acute anterior uveitis

134Fig. 12. Fibrinous membrane in pupillary area

UVEITIS AND SCLERITIS / 135 •



Signs of Inflammation in the anterior chamber. As a result of the inflammatory process, patients may present with haziness in the anterior chamber due to the leakage of protein from leaking blood vessels. This is referred to as flare. One may also note the presence of inflammatory cells in the anterior chamber as a result of the inflammation. When there are a large number of cells in the anterior chamber, they accumulate and assume a level in the anterior chamber referred to as hypopyon (Fig. 13). Cells may also be observed in the retrolental area which may indicate that the inflammatory process involves the ciliary body as well.

Fig. 13. Hypopyon

Fig.14. Koeppe nodules (located at the pupillary border

Fig.15. Busacca nodules in iris stroma

Fig.16. Fibrinous exudates in acute anterior uveitis

Iris changes. The iris undergoes changes as a result of the inflammatory process. They become thinned out and may be referred to as moth-eaten in appearance. Nodules may also be present in the iris, either within the stroma, called Busacca nodules (Fig. 15) or in the pupillary margin, referred to as Koeppe nodules (Fig. 14). These nodules represent cellular aggregates of inflammatory cells.

6. Intermediate Uveitis Findings. Patients with intermediate uveitis or wherein the inflammation involves the ciliary body or peripheral retina may present with haziness of the vitreous (Fig. 17) and one may note cells behind the lens on slit lamp examination. Fundus examination may reveal the presence of cellular aggregates in the periphery of the retina referred to as “snowbanking” (Fig. 16).

Fig. 18. Snowbanking

Fig.17. Vitreous haze

135

UVEITIS AND SCLERITIS / 136

ANCILLARY EXAMINATIONS for UVEITIS Commonly, additional examinations may be requested to aid in the diagnosis of some conditions. The more commonly requested ancillary procedures include: 1. Ultrasound of the Eye In most cases of uveitis, a view of the posterior portion of the eye is compromised due to the presence of anterior segment pathology. Ultrasonography gives the clinician a picture of the posterior segment. Uveitic conditions may present with the presence of a cataractous lens, vitreous condensations or haziness, choroidal thickening and retinal detachment upon ultrasonographic examination. 2.

Fluorescein Angiography Fluorescein angiography is a procedure wherein abnormalities in blood flow in the vasculature of the posterior segment of the eye may be detected. It also provides a method for detecting problems in the choroid and the retina. Some ophthalmologic conditions may present with typical fluorescein findings. Patients with inflammation of the posterior segment may present with findings compatible with retinal edema, retinitis and areas of focal retinal detachment. Optic disc as well as macular problems may also be encountered in these patients.

SYSTEMIC EXAMINATION There are a number of systemic conditions that may present with uveal tract inflammation. Examples of such would include various arthritic conditions, parasitic conditions and immunologic conditions. These diseases may present with mucocutaenous abnormalities, e.g. oral/genital sores, vitiligo, erythema nodosum; joint inflammations; pulmonary involvement, gastrointestinal pathologies to name a few. REVIEW QUESTIONS. PARTS I & II 1.

2.

3.

4.

5.

Uveitis refers to inflammation of A. conjunctiva B. cornea C. uveal tract D. optic nerve Eye redness seen in patients with uveitis is best described as A. diffuse congestion B. perilimbal flush C. confined to areas of extraocular muscle insertion D. blanches with pressure Keratic precipitates is a characteristic finding in uveitis. These are composed of A. red blood cells B. inflammatory cells C. corneal endothelial cells D. protein deposits Intraocular pressure in uveitis may be A. normal B. high C. low D. any one of the above The most frequent chief complaint of patients with uveitis is A. blurring of vision B. floaters C. photophobia D. eye redness

136

UVEITIS AND SCLERITIS / 137 A comprehensive history should be taken from patients suspected of uveitis. The history aids in all of the following, EXCEPT A. in the formulation of a possible etiologic diagnosis B. establishing the chronicity of the disease C. anatomic structures involved in the inflammatory process D. choice of treatment regimen for the patient 7. Pupil abnormalities are frequently seen in patients with anterior segment inflammation. The pupil is often A. constricted and irregularly shaped B. dilated C. eccentric in location D. normal in appearance 8. Inflammation of the uveal tract leads to the presence of flare. Flare results from A. deposition of inflammatory cells in the corneal endothelium B. presence of inflammatory deposits on the corneal endothelium C. adhesions between the lens and the pupillary border D. leakage of protein from the vascular barriers of the eye 9. Funduscopic changes which is frequently present in patients with posterior uveitis include A. hemorrhages B. exudates C. signs of vasculitis like perivascular sheathing D. abnormal optic disc appearance 10. Ultrasonography may be used as an ancillary tool in the diagnosis of uveitis. Commonly, patients who have choroidal involvement would present with which picture on ultrasound A. vitreous condensations B. retinal detachment C. choroidal thickening D. vitreous hemorrhage

6.

PART III. CLASSIFICATION OF UVEITIS Uveitis may be classified in a variety of ways. Classifying serves three main purposes namely (1) as a guide for the clinician in requesting for further diagnostic examinations (2) to serve as a guide in managing the patient and (3) for prognostication purposes. The uveitides may be classified based on the anatomical area or areas involved in the inflammatory process (Table 1); pathologic classification (Table 2) as to the severity and course of the disease (Table 3) or as to the etiologic cause (Table 4).

Table 1. Anatomic Classification of Uveitis Anatomical Classification Anterior Uveitis

Peripheral or Intermediate Uveitis Posterior Uveitis

Diffuse Uveitis

Anatomic Area Involved Anterior segment structures involved ƒ Iritis (iris) ƒ Cyclitis (ciliary body) ƒ Iridocyclitis (both iris and ciliary body) Pars Planitis Pars plana and peripheral retina involved Posterior Pole Involvement ƒ Retinitis (retina) ƒ Choroiditis (choroid) ƒ Chorioretinitis (both choroid and retina involved with the choroid being more affected) ƒ Retinochoroiditis (both choroid and retina involved with the retina being more affected) Panuveitis – all three structures of the uveal tract is involved in the inflammatory process

137

UVEITIS AND SCLERITIS / 138

Table 2. Pathologic Classification of Uveitis and their Corresponding Features GRANULOMATOUS UVEITIS • • •

Insidious in onset Usually involves the entire uveal tract Characterized by: ¾ Large greasy keratic precipitates ¾ Iris nodules ¾ Granulomas in the posterior pole

NON-GRANULOMATOUS UVEITIS • • •

Acute attacks with remissions and exacerbation Usually involves the anterior segment only Characterized by: ¾ Fine or small keratic precipitates ¾ Marked inflammation with tendency to hypopyon formation

Table 3. Classification as to Severity and Course of the Disease

Chronic Uveitis

• Inflammation presents as an initial attack • Condition has been going on for less than two months or eight weeks • Characterized by repeated attacks of inflammation wherein the eye persists to have

Recurrent Uveitis

• Condition has been going on for more than two months • Characterized by repeated attacks of inflammation wherein all signs of inflammation

Acute Uveitis

signs of low grade inflammation between bouts of exacerbation disappear between bouts of exacerbation

• Condition has been going on for more than two months

Table 4. Etiologic Classification of Uveitis Bacterial Viral Fungal Parasitic Immunologic Systemic

Neoplastic Miscellaneous conditions

ƒ ƒ ƒ ƒ ƒ ƒ ƒ ƒ ƒ ƒ ƒ ƒ ƒ ƒ ƒ ƒ ƒ ƒ ƒ ƒ ƒ

Tuberculosis Leprosy Syphilis Herpes simplex Herpes zoster Cytomegalovirus Histoplasmosis Coccidiomycosis Toxoplasmosis Toxocariasis Onchocerciasis Lens-induced Sympathetic ophthalmia Reiter’s syndrome Rheumatoid arthritis Sarcoidosis Collagen disease Reticulum sarcoma Lymphoma Heterochromic iridocyclitis Pigmentary syndrome

138

UVEITIS AND SCLERITIS / 139

REVIEW QUESTIONS. PART III 1. Patients with primary inflammation involving the iris and ciliary body are labeled as A. intermediate uveitis B. pars planitis C. iritis D. iridocyclitis 2. Characteristics of non-granulomatous uveitis A. of insidious onset B. presence of iris nodules C. presence of large keratic precipitates D. tend to have hypopyon in the anterior chamber 3. A patient presents with a history of a 4-week duration of eye redness. He claims that he has no history of any similar attack in the past. The patient probably has A. Acute uveitis B. Recurrent uveitis C. Chronic uveitis 4. On ophthalmologic examination, a patient was found to have inflammatory changes in all parts of the uveal tract. The patient’s condition can thus be labeled as A. Anterior uveitis B. Peripheral or intermediate uveitis C. Posterior uveitis D. Diffuse uveitis 5. Uveitis may be associated with bacterial infections. Common bacterial infections which may manifest in the eye as uveitis is A. tuberculosis B. typhoid C. histoplasmosis D. sarcoidosis PART IV. SCLERITIS This section will deal with inflammatory conditions that involve the sclera and episcleral tissues.

A. EPISCLERITIS This refers to a benign inflammatory disease characterized by edema and cellular infiltration of the episcleral tissue. The condition usually occurs in young adults, with females being more affected by males. The condition is usually bilateral in one third of cases and patients often complain of mild discomfort, specifically a slight ache, feeling of heat or irritation in the involved eye. Eye redness is commonly confined in the interpalpebral area and would blanch with instillation of 10% phenylephrine. The condition can further be classified as follows: simple episcleritis • more common • often self-limiting (2-19 days) • may be recurrent • episodes become less frequent after the first 3 to 4 years B. nodular episcleritis • usually takes longer to resolve (4-6 weeks) • underlying sclera is often normal • associated with movable, non-tender nodule A.

139

UVEITIS AND SCLERITIS / 140

Fig. 19. Simple Episcleritis

Fig. 20. Nodular Episcleritis

Systemic Associations are found in about one third of patients. This is however not correlated to the type, laterality or chronicity of the disease. Systemic conditions associated with episcleritis include 1. connective tissue diseases ( rheumatoid arthritis, SLE, relapsing 2. polychondritis) 3. B27-associated conditions (spondyloarthropathies, inflammatory bowel disease) 4. vasculitic diseases ( polyarteritis nodosa, Behcet’s disease, Wegener’s granulomatosis, giant cell arteritis, Cogan’s syndrome) 5. infections ( herpes, bacterial, fungal, parasitic) 6. miscellaneous conditions ( atopy, rosacea, gout) Patients with episcleritis generally do not develop ocular complications, although uveitis, peripheral corneal inflammation and glaucoma may occur in a small percentage of cases. There have also been reports on a small number of these patients progressing to scleritis.

B. SCLERITIS Scleritis is a severe ocular inflammation which consists of edema and inflammatory cell infiltration of the sclera. These changes are often immunologically mediated, or less commonly, the result of infection. Local trauma can also precipitate the inflammation. Without treatment it may be progressively destructive. It may also be the presenting symptom of an underlying lethal systemic condition or it may herald the onset of a ‘flare’ of an already diagnosed systemic disease thought to be in remission. Unlike episcleritis, patients affected with this condition are usually in their fourth to sixth decades of life. Females outnumber males to a small degree. The condition has been reported to occur bilaterally and is recurrent in about one third of cases. The main symptom of this condition is severe, penetrating eye pain which may radiate to the forehead, jaw and sinuses. Redness is gradual in onset and has a characteristic bluish-red tinge which may best be observed under natural light. Such redness fails to blanch with instillation of 10% phenylephrine or vasoconstrictors.

Classification Scleritis has been classified as follows (Watson and Hayreh) A. anterior • diffuse • nodular • necrotizing with or without inflammation ( scleromalacia perforans) B. posterior Each of these types will be discussed in detail in the subsequent section. 1. Diffuse anterior scleritis is the most common type of scleritis. It is of insidious onset and may develop over a 5 to 10 day period. Often misdiagnosed as episcleritis, it is associated with the best visual prognosis as ocular complications rarely occur. It is also least associated with a systemic disease. 140

UVEITIS AND SCLERITIS / 141

2. Nodular anterior scleritis occurs less frequently than the diffuse anterior type but is more common than necrotizing scleritis. The inflammatory process is localized to a nodule(s) which is immobile, firm and tender to touch. As in the diffuse anterior type it is of insidious onset. In terms of severity of the disease, it is intermediate between that of the necrotizing type and posterior scleritis. 3. Necrotizing scleritis with inflammation is the most severe and destructive form. It is characterized by the presence of white avascular areas surrounded by swollen inflamed sclera (Fig. ). More than half of patients with this condition have associated systemic conditions such as rheumatoid arthritis, Wegener’s granulomatosis and relapsing polychondritis. It is also the most frequently associated with ocular complications such as peripheral corneal thinning or stromal keratitis, uveitis, cataract, and glaucoma.

Fig. 21. Anterior Nodular Scleritis

Fig. 22. Necrotizing Scleritis with Inflammation

4. Necrotizing scleritis without inflammation or Scleromalacia perforans is painless condition characterized by the appearance of yellow-gray nodules that gradually develop a necrotic slough or sequestrum without surrounding inflammation of the sclera. The necrotic tissue eventually separates leaving the choroid bare, covered only the conjunctiva. Spontaneous perforation rarely occurs, although traumatic perforation can easily occur. This condition is almost always associated with rheumatoid arthritis.

Fig. 23.

Necrotizing Scleritis without Inflammation (Scleromalacia perforans

5. Posterior scleritis has the lowest incidence among these conditions. It occurs twice as often in females and affects both eyes in about one third of cases. It may also be associated with anterior scleritis. Frequently, patients are older than 50 years and have and increased risk of visual loss. Patients with this condition complain of periocular pain, pain with eye movements and decreased vision. Other eye symptoms include conjunctival chemosis, proptosis, lid swelling, lid retraction and limitation of extraocular movements. Fundus examination may reveal disc swelling, choroidal folds, serous retinal detachment, uveal effusions and macular edema. Ultrasonography is the key to making a diagnosis of posterior scleritis. Associated Diseases Systemic conditions have been associated with approximate 50% of all scleritis cases. Table 5 shows a listing of these conditions.

141

UVEITIS AND SCLERITIS / 142

Table 5. Systemic Diseases Associated with Scleritis Connective tissue diseases • • • • •

rheumatoid arthritis (RA) systemic lupus erythematosus B27 spondyloarthropathies inflammatory bowel disease relapsing polychondritis (RP)

Vasculitic diseases • • • • •

Wegener’s granulomatosis polyarteritis nodosa Behcet’s disease giant cell arteritis Cogan’s syndrome

Infectious • viral ( herpes zoster, herpes simplex, mumps) • bacterial ( tuberculosis, syphilis, pseudomonas,

Hemophilus, Borellia) • fungal • parasitic

Miscellaneous • • • • •

rosacea gout chemical injury Psoriatic arthropathy Psoriatic rash

REVIEW QUESTIONS. PART IV 1. The main difference in the clinical presentation of scleritis from episcleritis is the presence of A. nodules B. eye redness C. eye pain D. eye discharge 2.

The condition associated with the worst visual prognosis is A. nodular scleritis B. Scleromalacia perforans C. diffuse anterior scleritis D. necrotizing scleritis with inflammation

3.

The systemic condition that is more frequently associated with scleritis is A. allergy B. rheumatoid arthritis C. diabetes mellitus D. thyroid disease

4.

A patient presents with localized eye redness of about five days duration. On examination, you note the presence of a movable, non-tender nodule. Your impression would be A. diffuse episcleritis B. nodular episcleritis C. diffuse anterior scleritis D. nodular scleritis

5.

The inflammatory condition of the sclera which is associated with blurring of vision, proptosis and pain on eye movement is A. diffuse anterior scleritis B. nodular scleritis C. Scleromalacia perforans D. posterior scleritis

142

UVEITIS AND SCLERITIS / 143

PRACTICE CASES Case 1. 50 y.o./F presents with a history of two-month duration of blurring of vision and redness in the left eye. She denies any other associated symptoms nor any history of a similar episode in the past. Ophthalmologic examination revealed the following findings: Best corrected vision OD = 6/21 OS = CF at 1 meter Intraocular Pressures OD – soft OS – hypotonic EOM’s OU – full Funduscopy OD – essentially normal funduscopic Findings OS – no ROR noted due to a very hazy media Slit Lamp Findings: OD – clear cornea, no cells or flare, no KP’s, normal appearance of the iris, (+) lens opacity OS – clear cornea, (+) perilimbal flush, (+) cells/ (+) flare, (+) fine non-pigmented keratic precipitates, (+) cells in the retrolental area, (+) vitreous condensation 1. Based on the given information, how would you classify the patient’s condition on the basis of disease activity? 2. Assuming that this patient has normal funduscopic findings in the left eye, what is the anatomic classification of this patient’s uveitis? 3. What ancillary procedure may be useful in determining the presence or absence of posterior segment inflammation? Case 2. 24 y.o./M presents with a chief complaint of bilateral blurring of vision. History reveals that the patient first experienced eye redness in the left eye about one year prior to consultation that was associated with photophobia and blurring of vision. Consult was done at that time and symptoms allegedly resolved with unrecalled topical medication. About 1 month prior to consultation, however, the patient noted recurrence of symptoms in the left eye and a week later, experienced similar symptoms in his right eye. He also noted the presence of oral ulcers. The rest of the history was unremarkable. Ophthalmologic Examination at time of consultation revealed the following findings: Visual Acuity IOP EOM”S Funduscopy

Slit Lamp Findings

Right Eye 6/21 Soft Full (+)ROR, slightly hazy media, distinct disc borders, CD 0.3, AV 2:3, no hemorrhages, no exudates, (+) note of perivascular sheathing, slightly dull foveal reflex Clear cornea, formed anterior chamber, (+) small pigmented KP’s, (+)cells,(+) flare (-) iris nodules, moth eaten iris

Left Eye 6/30 Soft Full (+)ROR, slightly hazy media, distinct disc borders, CD 0.3, AV 2:3, no hemorrhages, no exudates, (+) note of perivascular sheathing, slightly dull foveal reflex Clear cornea, (+) small pigmented KP’s, (-)cells, (+)flare (-) iris nodules, moth eaten iris

1. What parts of the uveal tract is involved in the inflammatory process? Based on this, how would you classify the patient’s condition? 2. Is the disease condition chronic, recurrent, or acute? 3. Based on the above findings, is the disease condition granulomatous or non-granulomatous in nature? 4. What ancillary procedure may be requested in this patient to determine the extent of inflammation?

143

UVEITIS AND SCLERITIS / 144

Case 3. A 43 y.o female presents with a 6-month history of red, painful left eye. She previously had an episode of corneal ulcer in the same eye. Her past medical history was positive for sinusitis, rosacea, photosensitivity and pneumonia. She also had a hysterectomy about 2 years ago. Her family history was positive for glaucoma (grandfather), diabetes mellitus and tuberculosis (grandmother). The rest of her history was unremarkable. Ophtha Exam revealed the following findings: Visual acuity: OD – 20/20, OD – 20/25 Gross Exam: Rosacea facies; normal pupillary reflexes; normal eyelids EOMs: full on all directions of gaze IOPs: OD- 15 mm Hg, OS – 12 mm Hg Fundus Exam: Normal in both eyes Slit lamp Examination OD – essentially normal findings OS – localized redness temporally with note of corneal edema and infiltration (pls. refer to picture below)

1. Given the patient’s history and eye findings, what are your differential diagnoses for this case? 2. How would you differentiate one condition from another? PART V. SUMMARY It was the primary objective of this self-instructional material to provide the reader with a guide on how to diagnose patients with uveitis and scleritis. The importance of a comprehensive clinical history can not be overemphasized since the etiologic diagnosis of most of these ocular inflammatory conditions relies on the history. Secondly, one should be able to recognize the various ocular signs associated with these inflammatory conditions. RECOMMENDED FOLLOW-UP It is recommended that the students be given demonstration sessions on how to properly conduct historytaking and ophthalmologic examination of patients. Following this exercise, it is further recommended that the students be provided clinical sessions to allow them to see actual cases of patients with uveitis and scleritis. CONCLUSION As medical practitioners, you may, in the future encounter patients who will seek consultation for eye problems. One should bear in mind that not all conditions which present with a red eye is “sore eyes” and a patient may actually be suffering from another, more vision-threatening condition like uveitis or scleritis. It is therefore your role to be able to properly recognize these patients so that immediate referral to an ophthalmologist for further evaluation and management can be done. By doing so, early intervention can be facilitated and permanent visual impairment may be avoided.

144

UVEITIS AND SCLERITIS / 145

REFERENCES 1. Berson, Frank G. (ed) Basic Ophthalmology for Medical Students and Primary Care Residents. 6th edition. American Academy of Ophthalmology. 2. Kanski, Jack J. Clinical Ophthalmology A Systematic Approach. 4th edition. Butterworth Heinemann. Oxford, 1999. 3. Newell, Frank, Textbook of Ophthalmology. latest edition. 4. Nussenblatt, R.B., Whitcup, S.M. and Palestine, A.G. Uveitis: Fundamentals and Clinical Practice. 2nd ed. Mosby, Baltimore, 1996. 5. Sainz de la Maza M, Jabbur NS, Foster CS: Severity of scleritis and episcleritis. Ophthalmology 1994 Feb; 101(2): 389-96 6. Smith, Ronald E., and Nozik, Robert A. Uveitis: A Clinical Approach to Diagnosis and Management. Williams and Wilkins, Baltimore, 1983. 7. Watson PG, Hayreh SS: Scleritis and episcleritis. Br J Ophthalmol 1976; 60: 163-192 8. Vaughn, D., Asbury, T. and Riordan-Eva, P. General Ophthalmology. 15th ed. Appleton and Lange, Connecticut, 1992. ANSWERS TO REVIEW QUESTIONS PARTS I and II

PART III

PART IV

1. C 2. B 3. B 4. D 5. A

1. D 2. D 3. A 4. D 5. A

1. 2. 3. 4. 5.

6. C 7. A 8. D 9. C 10. C

C D B B D

ANSWERS TO QUESTIONS FOR CASES Case 1. 1. acute uveitis 2. anterior uveitis, specifically iritis 3. ultrasound, since there is no view through the pupil Case 2. 1. iris, ciliary body and the posterior segment (retina and/or choroid) – based on anterior and posterior segment findings 2. recurrent uveitis – assuming that the patient was inflammation free during the one year period 3. non-granulomatous type of uveitis 4. fluorescein angiography Case 3. 1. nodular episcleritis vs. nodular scleritis 2. > may instill vasoconstrictors and if blanches Æ episcleritis; if it does not blanch Æ scleritis > also check for tenderness since tenderness would support diagnosis of scleritis > check if nodule present is movable; if nodule is movable, would support diagnosis of episcleritis

145

TEARING / 146

TEARING Alexander Dy Tan MD INTRODUCTION Tearing is a common symptom of many ocular and sometimes even systemic illnesses. A thorough understanding of the lacrimal apparatus, tear film and lacrimal drainage system is necessary in order to determine the cause of a patient’s tearing. Tearing is often classified into two entities: lacrimation or excess production of tears, and epiphora which is overflow of tears due to blockage of the lacrimal drainage system. OBJECTIVES Upon completion of this instructional material, the student should be able to 1. define the symptom of tearing (differentiate between lacrimation and epiphora) 2. enumerate the common conditions that cause tearing. 3. enumerate and describe the different clinical examinations which can help determine the cause of tearing. 4. provide basic clinical (diagnosis and management) information regarding the different types of nasolacrimal duct obstruction. PREREQUISITE KNOWLEDGE AND PREPARATION 1. Anatomy of the eye, ocular adnexae, particularly the lacrimal drainage system 2. Physiology of the lacrimal drainage system 3. Basic ocular examination (normal and abnormal findings) INTENDED USERS Year Level IV UPCM Medical Students CONTENT Tearing or ‘watery eyes’ is a common ocular symptom which has numerous causes ( Appendix A). Chronic tearing can be a debilitating complaint which may be a nuisance (constant need to wipe off the tears), source of embarrassment and discomfort for the patient. Tearing is either due to increased production (lacrimation) or from impaired drainage (epiphora). The tear film has unique characteristics (Table 1). It is composed of three layers: an outer oily layer (oily secretions from Meibomian and Zeis glands), a middle aqueous layer produced by the lacrimal gland and glands of Krause and Wolfring, and an inner mucinous layer secreted by the goblet cells of the conjunctiva. The superficial oily layer functions to prevent the evaporation of the aqueous layer. The pH of tears averages 7.4. The mucous layer is important for proper wetting of the cornea. The lacrimal drainage system begins at the punctum leading to the canaliculus to the lacrimal sac down to the nasolacrimal duct. The duct opens at the inferior meatus under the inferior turbinate. See Illustration below, note 1 = valve of Rosenmuller, 5 = valve of Hasner (opening at inferior meatus underneath the inferior turbinate)

146

TEARING / 147

We must keep in mind that the amount of tears in our eye is dictated by three factors: production, evaporation and drainage. Conditions that increase production and impair drainage will result in a surplus of tears in the eye, while increased evaporation (decreased humidity, prolonged reading, eyelid retraction, etc) will lead to a relative lack of tears. Increased Tear Production (Lacrimation) Tearing is more commonly caused by increased production. It is very important to get a thorough history and physical examination in order to determine the cause of tearing. It is important to ask for: history of trauma, eye redness, use of medication, photophobia, and blurred vision. Patients who complain of ‘foreign body sensation’ can have conditions such as: corneal or conjunctival foreign body, misdirected lashes, corneal ulcer, etc. Tearing in these situations is the body’s attempt to wash out the eye of the ‘irritant’. Important questions to be asked in a patient’s history include the following: 1. History of trauma – to rule out conjunctival or corneal foreign bodies, corneal abrasions. Certain occupations (i.e. carpenters, construction workers etc) are predisposed to foreign bodies in the cornea or conjunctiva. Lacerations may involve the lacrimal gland or any part of the drainage system which can affect the amount of tears present in the tear lake. Canalicular lacerations should be suspected when there is a lid margin laceration within 5 mm of the medial commissure. 2. Associated symptoms – discharge (water, mucoid or purulent), redness, eye pain, blurring of vision, foreign body sensation, itchiness. Exposure to someone with similar ‘red eyes’ may suggest a form of viral conjunctivitis, while allergies are often associated with itchiness. 3. Consultations with a doctor and medications used, history of ocular surgery. This may suggest a chronic process or may be a result of patients surgery. It is important to establish a timeline of consultations, surgeries, and medications used in order to figure out when the symptoms started and how they responded to therapy. 4. Onset and duration of tearing (Worsening during prolonged reading or when exposed to wind may indicate a ‘dry eye’. While reading or working with the computer, we tend to blink ‘less often’ resulting in more tear evaporation. These situations uncover the presence of dryness or ‘tear instability). Dry eye more common in the elderly, nasolacrimal duct obstruction is more common in women.

147

TEARING / 148

5. Other medical problems particularly allergies, sinus disease A thorough ocular examination (visual acuity, gross examination, EOM, tonometry and funduscopy) is necessary to determine the cause of a patient’s tearing. Examination for patients with lacrimation should focus on the anterior segment examination. Any injury or inflammation involving the lashes, cornea, conjunctiva, anterior chamber and iris will result in some form of tearing. Ectropion and entropion may cause ocular irritation resulting in tearing. Slit lamp biomicroscopy utilizes a microscope with a light in order to magnify the structures in the eyelids and anterior segment of the eye, with the slit beam providing other details as presence of anterior chamber inflammation, anterior chamber depth, corneal clarity, lens clarity, etc. A slit lamp may be used in conjunction with a Goldman’s applanation tonometer in order to determine a patient’s intraocular pressure. Note. Patients with Dry Eye can also manifest with tearing. Generally, patients with mild to moderate dry eye may complain of tearing which is mostly a reflex mechanism to compensate for the ‘lack of tears’. Slit lamp examination may show an abnormal tear meniscus, decreased production of tears may be confirmed by doing the Schirmers test. Tests for patients with Lacrimation Schirmers Test: uses a Whatman filter paper in order to measure the amount of wetting (from tears) in five minutes. A Schirmers strip is folded and placed at the junction of the lateral and middle third of the lower eyelid. Patient is asked to look straight ahead and avoid blinking. Schirmers Test can be done with and without anesthetic (Proparacaine). Schirmers Test with anesthetic is thought to measure basal secretion of tears while the test without anesthetic measures reflex tearing. A Schirmers Test with anesthetic wetting of 5 mm or less in 5 minutes is indicative of dry eye. Management of Lacrimation is generally directed to address the underlying cause of tearing. Management will range from antibiotic eyedrops for patients with bacterial infections, removal of foreign bodies for patients with conjunctival or corneal foreign bodies, eyelid surgery for patients with misdirected lashes, entropion or ectropion, etc. Correction of these conditions will result in the resolution of the patients tearing. Thickness Layers Thickness of the Aqueous Layer Tear Volume

7 – 10 um 3 6.5um 6 – 8 ul

Tear Production

1.2ul per minute

Table 1. Characteristics of the Tear Film

Decreased Tear Drainage (Epiphora) Tearing due to blocked tear drainage can be a very bothersome symptom. Patients frequently carry along boxes of tissue paper in order to constantly wipe off overflowing tears. Chronic epiphora can sometimes lead to infections of the lids and lacrimal sac. Any point of the lacrimal drainage system can be blocked, from the punctum to the canaliculus, lacrimal sac, and the nasolacrimal duct. Punctal stenosis can be visualized directly or with the use of a slit lamp. Canalicular stenosis can be confirmed with probing while a lacrimal apparatus irrigation is often necessary in 148

TEARING / 149

order to diagnose nasolacrimal duct obstruction. Blockage may be due to scarring from inflammation or topical medicines, from injury (canalicular transections), or from tumors (lacrimal sac tumors, dacryolithiasis). Punctal ectropion associated with lid laxity can contribute to decreased outflow of tears. Conditions such as Bells palsy or weakness of the CN7 may also cause physiologic pump failure of the lacrimal drainage system. Nasolacrimal duct obstruction should be suspected in patients with tearing who have normal anterior segment examination (i.e. no abnormal finding which can explain the patients tearing). Patients often have a long standing history of on and off tearing of the involved eye, with or without history of infection (mucoid discharge, conjunctivitis, dacryocystitis). Patients with dacryocystitis often have history of swelling of the medial canthal area which when exacerbated results in: pain swelling and erythema (acute dacryocystitis). Primary acquired nasolacrimal duct obstruction is more common in elderly females. Diagnostic Tests: 1. Probing : a fine blunt probe can be inserted through the punctum and canaliculus in order to determine the patency of the upper lacrimal drainage system. Probing may also be confirmatory for patients with canalicular transactions due to trauma.

Picture 1. (on left): A Gauge 25 Lacrimal Irrigating Canula is inserted through the punctum (exits at the distal cut end of the canaliculus), confirming presence of canalicular laceration

2. Lacrimal Apparatus Irrigation: involves the irrigation of normal saline solution through the punctum and canaliculus, reflux (either thru the upper canaliculus or lower canaliculus) would indicate an obstruction. 3. Palpation of the lacrimal sac area: applying pressure on a distended lacrimal sac may result in mucoid reflux, which confirms the presence of nasolacrimal duct obstruction. Obstructions involving the canaliculi or puncta will not result in distension of the sac since the tears will not be able to reach the sac. 4. Dye disappearance test: assesses the presence or absence of adequate lacrimal drainage. Fluorescein is instilled on the cul de sac of both eyes. Asymmetric clearance of the dye within 5 minutes indicates relative block in the side with dye retention. Retention of dye beyond five minutes in one eye is also indicative of blockage. The DDT does not distinguish between mechanical blockage and functional blockage. Management of nasolacrimal duct obstruction is surgical. A dacryocystorhinostomy is performed for adult patients with symptomatic and complete nasolacrimal obstruction. It involves a fistulizing procedure, connecting the lacrimal sac to the nasal mucosa. The surgery is performed under general anesthesia. Special Topic: Congenital Nasolacrimal Duct Obstruction: is present in up to 10% of all normal infants. Pathology is an imperforate membrane at the Valve of Hasner at the inferior meatus. Children presents with tearing or chronic eye discharge / recurrent conjunctivitis, while severe cases present with fulminant dacryocystitis. Diagnosis is usually confirmed by doing a dye disappearance test. Mucoid reflux upon digital pressure at the lacrimal sac area also confirms the diagnosis. About 90% of cases will resolve during the first year of life and spontaneous resolution continues even up to the 2nd year of life. Management of uncomplicated (i.e. no signs of infection, Dacryocystitis) is lacrimal sac massage. Persistent cases are managed with therapeutic probing

149

TEARING / 150

(rupturing the membrane at the Valve of Hasner) which is usually performed at 12 months of age. Patients who have undergone failed probings are managed with bicanalicular intubation with silicone tubes or by performing a dacryocystorhinostomy.

Picture 2. Congenital NLDO with dye retention in right eye in dye disappearance test

APPENDIX Differential Diagnoses for Tearing (from Ocular Differential Diagnosis, 7th edition by Frederick Hampton Roy) I. Hypersecretion of tears A. Primary (disturbance of the lacrimal gland) B. Central 1. Central nervous system lesions 2. Corticomeningeal lesions 3. Emotional states 4. Hysteria 5. Physical pain 6. Voluntary lacrimation, such as when acting C. Neurogenic 1. Ametropia, tropia, phoria and eyestrain and fatigue 2. Caloric, lacrimal and reflex tearing – bilateral lacrimation when syringing the ear with warm or cold water and during Tensilon testing 3. Crocodile or alligator tears – unilateral profuse tearing when eating a. Congenital, often associated with ipsilateral paresis of lateral rectus muscle b. Acquired with onset in early stage of facial palsy (Bells palsy) or sequela with parasympathetic fibers to the otic ganglion growing back into superficial petrosal nerve c. Duane retraction syndrome 4. Bells palsy 5. Marin-Amat Syndrome (inverted Marcus Gunn Jaw winking phenomenon) 6. Melkersson-Rosenthal syndrome (Melkersson idiopathic fibro edema) 7. Drugs: atenolol, ciprofloxacin, dexamethasone, diazepam, ketamine, heparin, morphine, nifedipine, naloxone, reserpine, etc 8. Exposure to wind, cold, bright light

150

TEARING / 151

9. Glaucoma 10. Horner syndrome 11. Inflammation or infection of the conjunctiva, uvea, cornea, orbit, lids, sinuses, teeth or ears 12. Lesions affecting the lids a. ptosis b. entropion/ ectropion c. facial paralysis d. trichiasis 13. Morquio-Brailsford syndrome (MPS IV) 14. Myasthenia Gravis – afternoon ectropion (Erb-Goldflam syndrome) 15. Ophthalmorhinostomatohygrosis syndrome 16. Parkinson’s disease 17. Reflex, such as vomiting and laughing 18. Sjogren syndrome 19. Stimulation of some cortical areas – thalamus, hypothalamus, cervical sympathetic ganglia, or the lacrimal nucleus a. encephalitis b. diencephalic epilepsy syndrome (Penfield syndrome) c. Giant cell arteritis (temporal arteritis) d. Hypothalamic tumors e. Meningitis f. Page syndrome g. Pseudobulbar palsy for Parkinson syndrome h. Sluder syndrome i. Tic douloureux j. various senile dementias 20. Gradenigo syndrome (temporal syndrome) 21. Raeder syndrome (paratrigemial paralysis, cluster headaches) 22. Retroparotid space syndrome (Villaret syndrome) 23. Rhabdomyosarcoma 24. Rothmund syndrome (telangiectasia-pigmentation-cataract) 25. Thermal burns D. Symptomatic 1. Bee sting of cornea 2. Tabes 3. Thyrotoxicosis (Basedow syndrome) II. Inadequacy of lacrimal drainage system A. Congenital anomalies of the lacrimal apparatus 1. Absence of atresia of lacrimal drainage apparatus 2. Amniotocele 3. Fistulas of lacrimal sac and nasolacrimal duct 4. Waardenburg syndrome (lateral displacement of the medial canthi with lateral displacement of puncta and lengthening of the canaliculi 5. Obstruction of nasolacrimal drainage system B. Complications from diseases such as pemphigus, Stevens-Johnson syndrome and lupus C. Dacryocystitis D. Distended canaliculi with obstruction, such as in Actinomyces israelii , papilloma or dacryolith E. Drugs such as: acyclovir, silver nitrate, triflourothymidine, neostigmine, fluorouracil, etc F. Punctal eversion G. Goltz syndrome H. Inadequacy of physiologic lacrimal pump I. Traumatic lesions of the lacrimal drainage system

151

TEARING / 152

J. Tumor obstruction K. Nasal disease 1. Sinusitis 2. Hypertrophic rhinitis REFERENCES 1. Roy, Frederick , Ocular Differential Diagnosis, 7th edition 2. Hart (Ed) Adlers Physiology of the Eye , 3. Nesi et al , Smith’s Ophthalmic Plasctic and Reconstructive Surgery SELF-TEST For questions 1-5 . What is the mechanism of tearing in the following conditions? Possible answers are A: increased production B: decreased drainage 1. 2. 3. 4. 5.

Conjunctivitis Corneal Foreign Body Dacryocystitis Canalicular Stenosis Glaucoma

6. In children the easiest way of confirming a nasolacrimal duct obstruction is with the use of: A. radiograph B. dacryocystography C. probing D. dye disappearance test E. slit lamp 7. Most common location of blockage in congenital NLDO is at A. canaliculus B. punctum C. lacrimal sac D. valve of Hasner E. valve of Rosenmuller 8. Best means of confirming the presence of a canalicular laceration is through A. slit lamp examination B. probing C. dye disappearance test D. Jones Test E. dacryocystography Answers to Self-test 1. A 2. A 3. B

4. B 5. A 6. D

7. D 8. B

152

STRABISMUS / 153

DEVIATION OF THE EYE STRABISMUS Marissa N. Valbuena M.D., MHPEd INTRODUCTION Level IV medical students of the U.P. College of Medicine are encouraged to go through this material. Ocular motility problems are among common conditions the students will encounter in the out - patient clinics. A clear understanding of the anatomy and physiology of the extraocular muscles, the knowledge and skills in history taking and physical examination of patients with motility problems, as well as a knowledge of the clinical manifestations of these conditions will help the student in his interactions with the patients. OBJECTIVES After going though this material, the student is expected to: 1. Enumerate the extraocular muscles, their origin, insertion, innervation, and actions. 2. Given an extraocular muscle, identify its synergist, antagonist and yoke. 3. Given a patient with ocular motility problem, be able to extract a relevant medical history, and be able to perform a complete ocular examination. 4. Discuss the most common types of strabismus, their clinical manifestations and principles of management. RECOMMENDED PREPARATION Before going to this material, the student must have previous knowledge of anatomy and physiology of the eye and the methods of history taking and ocular examination. CONTENT ANATOMY AND PHYSIOLOGY OF EXTRAOCULAR MUSCLES Origin, Insertion, Innervation and Action of Extraocular Muscles There are six extraocular muscles controllong eye movement : the four recti muscles, the two obliques. The lateral rectus muscle is innervated by the abduscens nerve (VI) , the superior oblique is innervated by the trochlear nerve (IV) and the remaining muscles are innervated by the oculomotor nerve (III). The primary position of the eye is defined as being that when the eye is directed straight ahead with the head also straight. The primary action of a muscle is the major effect on the position of the eye when the muscle contracts while the eye is in primary position. The secondary and tertiary actions are additional effects on the position of the eye in primary position. The eye can usually be moved about 50 degrees in each direction from the primary position. Ordinarily however, the eyes move only 15 to 20 degrees from primary position before head movement occurs. Table I summarizes the origin, insertion, actions and innervation of the extraocular muscles.

153

STRABISMUS / 154

Muscle

Origin

Insertion

Medial rectus (MR)

Annulus of Zinn

Lateral rectus (LR)

Annulus of Zinn

Superior rectus (SR)

Annulus of Zinn

Inferior rectus (IR)

Annulus of Zinn

6.5 mm from inferior limbus

23°

Superior oblique (SO)

Orbit apex above Annulus of Zinn (functional rigin at trochlea) Behind lacrimal fossa

Posterior equator at superotemporal quadrant

51°

Posterior to the equator in inferotemporal quadrant

51°

Inferior oblique (IO)

5.5 mm from medial limbus 6.9 mm from lateral limbus 7.7 mm from superior limbus

Direction of pull 90 °

Action from Primary Position Adduction

Innervation Cranial Nerve III

90°

Abduction

VI

23°

Elevation Intorsion Adduction Depression Extorsion Adduction Intorsion Depression Abduction

III

Extorsion Elevation Abduction

III

III IV

Table 1. Extraocular Muscles

Extraocular muscle is a specialized form of skeletal muscles which incorporates several fiber types. At one extreme is a slow tonic type resistant to fatigue and active in holding gaze straight ahead. At the other extreme is a muscle type is adopted for participation in extreme gaze. The high ratio of nerve fibers to eye muscle fibers (1:3 to 1:5) allows more accurate control than is found in skeletal muscles in which the ratio ranges from 1:50 to 1:125. Motor Physiology

A. Axes of Fick, Center of Rotation, Listing Plane and Median Plane The axes of Fick are X, Y and Z. (Fig. 1) The X axis is a transverse axis passing through the center of the eye at the equator ; voluntary vertical rotations of the eye occur at this axis ; involuntary torsional movements occur at this axis . The Z axis is a vertical axis; voluntary horizontal movements occur at this axis . Listing’s equatorial plane passes though the center of rotation and includes the X and Z axis. The Y axis is perpendicular to Listing’s plane. The median plane is a sagittal plane passing antero-posteriorly through the body, bisecting the head into symmetric parts.

Fig 1 Axes Of Fick

154

STRABISMUS / 155

B. Positions of Gazes 1. Primary position - straight ahead 2. Secondary positions - straight up, straight down, right gaze, left gaze 3. Tertiary positions - four oblique positions of gazes : up and right, up and left, down and right, down and left 4. Cardinal positions - up and right, up and left, right left, down and right, down and left

C. Primary, Secondary and tertiary actions With the eye in primary position, the horizontal recti are purely horizontal movers along the Z axis and have only a primary action. The vertical recti have a direction of pull that is primarily vertical as their primary action, but the angle of pull from origin to insertion is 23 degrees inclined to the visual axis, giving rise to torsion and adduction. The obliques are inclined 51 degrees to the visual axis, giving rise to torsion as their primary action but also some vertical and horizontal rotations as well. Table 2 lists the actions of the extraocular muscles. MUSCLE

PRIMARY

SECONDARY

TERTIARY

Medial rectus

Adduction

Lateral rectus

Abduction

Superior rectus

Elevation

Incycloduction

Adduction

Inferior rectus

Depression

Excycloduction

Adduction

Superior oblique

Incycloduction

Depression

Abduction

Inferior oblique

Excycloduction

Elevation

Abduction

Table 2 . Action Of Extraocular Muscles From The Primary Position

D. Eye movements 1. Monocular eye movements ( Ductions ) Ductions are monocular rotations of the eye. Adduction is movement of the eye nasally, while abduction is movement of the eye temporally. Elevation is upward rotation of the eye; depression is downward movement of the eye. Incycloduction ( intorsion ) is nasal rotation of the superior portion of the vertical corneal meridian. Excycloduction (extorsion) is temporal rotation of the superior portion of the vertical corneal meridian. An agonist is the primary muscle that is moving the eye in a given direction. The synergist is the muscle is the same eye as the agonist that acts as the agonist to produce the same movement. The antagonist is the muscle in the same eye as the agonist that acts in the direction opposite to that of the agonist. Table 3 shows the agonists with their respective synergists and antagonists. Sherrington’s law of reciprocal innervation states that and increased innervation and contraction of a given extraocular muscle is accompanied by a reciprocal dectrease in innervation and contraction of its antagonist. For example, as the right eye adducts, the right medial rectus receives increased innervation while the right lateral rectus receives decreased innervation.

155

STRABISMUS / 156

AGONIST MEDIAL RECTUS

SYNERGISTS Superior rectus Inferior rectus

ANTAGONISTS Lateral rectus Superior oblique Inferior oblique

LATERAL RECTUS

Superior oblique Inferior oblique

Medial rectus Superior rectus Inferior rectus

SUPERIOR RECTUS

Inferior oblique Medial rectus

Inferior rectus Superior oblique

INFERIOR RECTUS

Superior oblique Medial rectus

Superior rectus Inferior oblique

SUPERIOR OBLIQUE

Inferior rectus Lateral rectus

Inferior oblique Superior rectus

INFERIOR OBLIQUE

Superior rectus Lateral rectus

Superior oblique Inferior rectus

Table 3 . Agonists With Their Respective Synergists And Antagonists

2. Versions ( Conjugate binocular eye movements ) Yoke muscles are two muscles (one in each eye) that are prime movers of their respective eyes in a given direction of gaze. For example, as the eyes move to the left gaze, the right medial rectus and the left lateral rectus are simultaneously innervated and contracted. Each extraocular muscle in one eye has a yoke muscle in the other eye. Figure 2 shows the six cardinal positions of gaze and the yoke muscles whose primary actions are in that field of gaze.

Fig 2. Cardinal Positions And Yoke Muscles

156

STRABISMUS / 157

Fig 3. Cardinal Positions And Yoke Muscles Hering’s law states that equal and simultaneous innervation flows to the yoke muscles concerned in the desired direction of gaze BINOCULAR VISION The eyes are spaced 50 to 65 mm apart. The slightly different image originating in each eye is fused in the brain as a streoscopic image. Each eye must be directed simultaneously to the same object. The visual axis is an imaginary line that connects an object in space with the fovea. In a person with normal ocular, sensory and motor systems, the visual line in each eye intersects at the object in space and there is binocular fixation. If visual lines are not directed at the same fixation point, fixation is by one eye only. Normal development of stereoscopic vision requires binocular, simultaneous use of each fovea during the critical time that occurs early in life. Amblyopia is a condition in which there is a unilateral or bilateral decrease in visual acuity that is not fully attributable to organic ocular abnormalities. It is usually caused by opacities in the media, high refractive errors, anisometropia (difference in refractive errors of the 2 eyes) or ocular misalignment or strabismus during visual immaturity. The correction of amblyopia depends on the maturity of the visual system at the onset and the duration of the abnormal visual experience. Treatment consists of occlusion (patching) of the better eye to force the use of the amblyopic eye and the correction of the underlying cause. STRABISMUS Definition of Strabismus Strabismus means ocular misalignment of whatever cause. When the eyes are dissociated or not aligned, strabismus is present.

157

STRABISMUS / 158

Orthophoria refers to the ideal condition of ocular balance, so eyes are aligned in all directions of gazes at all distances even after occluding one eye. Types of Strabismus: A. According to direction of deviation: 1. Horizontal - esodeviation, exodeviation 2. Vertical - hyperdeviation, hypodeviation 3. Torsional - excyclodeviation, incyclodeviation B. According to age of onset: 1. Congenital, infantile - documented prior to age 6 months 2. Acquired

C. According to fusion status (whether the deviation can be controlled by fusuin mechanism. 1. Phoria - latent deviation, controlled by fusion mechanisn so that under binocular condition, the eyes remain aligned.

2. Intermittent phoria or tropia - fusion control present part of the time 3. Tropia - manifest deviation in which fusion control is not present. D. According to variation of deviation with gaze position or fixating eye 1. Comitant - deviation does not vary with direction of gaze or fixationg eye. 2. Incomitant - deviation varies with direction of gaze or fixationg eye. Most incomitant strabismus is paralytic.

E. According to fixation 1. Alternating - there is spontaneous alternation of fixation from one eye to the other. 2. Monocular - there is preference for fixation with one eye. Examination of the Patient

A. History taking : information should be obtained about the following 1. 2. 3. 4. 5. 6. 7. 8. 9. 10. 11. 12.

Chief complaint Age of onset - document onset with photographs Direction of deviation Constant or intermittent Magnitude of deviation Associated eye complaints – diplopia, blurring of vision Antecedent or concurring illness - seizures , diabetes, thyroid disease Trauma Previous consultation, treatment - patching, glasses, surgery Maternal and birth history - prematurity Developmental history Family history

B. Ocular examination 1. Visual acuity

158

STRABISMUS / 159

Various tests are available for visual acuity determination . Children often pose a difficult assessment problem. For verbal and cooperative children, charts using tumbling E or pictures can be used. The child’s fixation pattern will give a clue as to the comparative vision of the two eyes. For example, achild who can alternate fixation in his 2 eyes will probably have equal visual acuity. Another child who prefers one eye to fixate probably has a better vision in that eye compared to the fellow eye. 2. Ocular motility examinations The following clinical protocol may be used to assess ocular movements : a. Sit facing the patient. Hold your finger or a small fixation target 10-14 inches in front of the patient, with the patient in primary position (straight ahead) b. Ask the patient to follow target as you move it into the six cardinal positions. Elevate upper eyelid with your finger of your free hand to observe downgaze. c. Note whether the amplitude of eye movements is normal or abnormal in both eyes. Rate amplitude for all fields of gaze by considering normal amplitudes as 100%, and rate lesser amplitudes accordingly. To record relative over or underactions, designate normal as 0, that is , no over or underactions are present. Use 4 to designate maximum over or underaction. Underactions are rated 1 to -4 while overactions are rated +1 to +4. d. Note any nystagmus and if presen,t record its direction and amplitude in specific field of gaze.

3. Tests for ocular alignment A. Corneal light reflex test ( Hirschberg method ) a. Ask the patient to seat facing you with head straight and eyes directed in primary gaze. b. Hold a penlight in front of the patient’s eyes at a distance of about 2 ft, directing the light between the patient’s two eyes. Instruct the patient to look directly at the light.

c. Compare the position of the light reflex and record the estimated degrees of deviation. See Fig. 4. B. Prism Test ( Krimsky Test) This test is usually perfomed in patients unable to fixate with both eyes because of poor vision in one eye or in uncooperative patients. a. Ask the patient to fixate on a light. b. Place increasing amount of prism on the straight eye until the corneal reflex on the deviating eye is centered.. c. Prisms placed on the deviating eye is preferred in patients with incomitant or paralytic deviations. ( See Fig 5)

15 ° Esotropia

30 Esotropia

45 ° Esotropia Fig. 4 . Corneal Light Reflex

159

STRABISMUS / 160

B. Cover tests The validity of a cover test depends upon the patient’s ability to maintain constant fixation on an accommodative target. Each eye must be able to move adequately when fixating . The cover-uncover test is done to establish the presence of either a manifest deviation (heterotropia) or a latent deviation (heterophoria). The alternate tests are then performed to measure the deviation. B.1. Cover-uncover test a. Ask the patient to look at a distance fixation and position yourself directly opposite the patient, at an arm’s reach. b. Cover the fixating eye with an occluder or your hand and observe the other eye for any movement. Note its direction. c. Uncover the eye and allow about 3 seconds for both eyes to be uncovered. d. Cover the other eye and observe its fellow for any movement. e. After about one second, uncover the eye and observe it for any movement. f. Repeat the test for near, using a near fixation point. g. Repeat the distance and near tests using patient’s eyeglasses, if applicable. B.2. Alternate cover test (prism and cover test) a. With the patient seated upright and looking at a distance fixation point, rapidly shift the occluder from one eye to the other several times, not allowing any period of binocularity. The examiner should be seated slightly to the side of midline, facing the patient and at an arm’s length to the patient. b. Place a trial prism over one eye, while continuing to shift the cover from one eye to the other. Orient the prism apex towards the direction of the deviation. Choose the strength of the initial prism to approximate the deviation estimated by the Hirschberg’s test. c. Continue to place prisms of progressively higher power until no movement is noted in either eye (neutralization). d. Repeat test for near.

Figure

. Prism Test

Fig. 5. Krimsky Prism Test

4. Ophthalmoscopy Abnormalities in the fundus should be noted such as abnormal optic disc, macular lesions and retinopathy of prematurity.

5. Refraction

160

STRABISMUS / 161

It is important to know the refractive state of the patient in assessing his ocular deviation. Cycloplegia is an adjunct to refraction in young strabismic patients. Cycloplegic agents that can be used include atropine, cylopentolate and tropicamide. Common Types of Strabismus A.

Comitant Strabismus 1. Congenital Esotropia or Infantile Esotropia

Congenital esotropia is usually noted shortly after birth or up to 6 months of age. The esodeviation is big and constant . Cross fixation (infant uses right eye to look at left visual field and left eye to see right visual field) may be present. There may be overaction of the inferior obliques , causing elevation of the adducting eye. Refraction is usually appropriate for the patient’s age. Aside from the esodeviation, the patient is usually otherwise normal. The child is best treated with surgery before the age of 18 months.

Fig. 6. Congenital Esotropia

Right gaze

Primary Gaze

Left gaze

Figure 7. Overacting Inferior Obliques. Elevation of the adducting eye.

2. Refractive Accommodative Esotropia Refractive accommodative esotropia usually starts at age 2 years. The child has a significant grade of hyperopia ( +3.00 to +10.00 diopters) . In order to see clearly, he accommodates. Accommodation is however accompanied by convergence of the eyes. Convex or plus lenses are prescribed to correct the hyperopia.

A

B

Fig 8 Accommodative Esotropia A. Esotropia of the right eye B. Eyes aligned with eyeglsses

3. Sensory Esotropia

161

STRABISMUS / 162

An esodeviation occurs in a patient with monocular or binocular condition that prevents good vision ( e.g. corneal opacity, cataract, retinal scars, inflammation , tumors, optic mreuropathy, anisometropia). Treatment consist of the following : attempt to correct the cause of the poor vison, full cycloplegic refraction, muscle surgery to correct the deviation. 4. Intermittent Exotropia Exotropia is an outward deviation of the eye which usually starts out as intermittent. It becomes manifests when patient is fatigued, sleepy or inattentive. He closes one eye when exposed to bright sunlight . The frequency and the duration of deviation may increase as the patient grows older. The exotropia can later become constant. Usually, the patient can use either eye for fixation . Vision is usually good for both eyes. Treatment is surgical.

A

B

Fig 9.Alternating Exotropia A. Left eye fixating B. Right eye fixating 5. Sensory Exotropia An eye that does not see well for any reason may turm outward. Principles in treatment of sensory exotropia is the same as that of sensory esotropia. B. Incomitant Strabismus. 1.. Paralytic Strabismus There is limitation of action of involved muscle. The deviation is bigger when the involved eye is fixating and in the direction of action of involved muscle. Lateral rectus is the most frequently involved muscle as a result of abduscens nerve palsy. The patient should have a neurologic and systemic evaluation. Strabismic Syndromes Motility disorders may demonstrate typical feature of a particular syndrome. Examples are Duane syndrome, Brown syndrome, Mobius syndrome and congenital fibrosis syndrome. Duane syndrome is a congenital motility disorder, usually unilateral, characterized by limited abduction, or limited adduction or both. The globe may retract and the eyelid fissure may narrow on adduction.There may also be upshooting or downshooting of the eye. There may be a face turn to allow the patient to use both eyes together. Muscle surgery is indicated to correct significant face turn or a significant deviation on primary gaze.

162

STRABISMUS / 163

Left gaze

Primary Gaze

Left gaze

Fig.10 Duane syndrome, bilateral. Patient is orthophoric on primary gaze. There is limitation of abduction in both eyes. There is narrowing of palpebral fissures on adduction.

Systemic illness associated with strabismus A. Thyroid disease : Grave’s ophthalmopathy is an autoimmune disaease affecting the extraocular muscles, orbital fat, lacrimal glands and orbital connective tissue. Lid retraction, exophthalmos and ophthalmoplegia are some of the clinical findings. Limitation of elevation because of inferior rectus restriction is the most common motility findings. Patients complain of diplopia most severe in upgaze. B. Diabetes mellitus : Diabetes is a complex metabolic disease involving small vessels and causing widespread damage to tissues, including the eyes Patients may have acute onset diplopia due to infarction of a cranial nerve and subsequent paresis of an extraocular muscle. The abdusens nerve and the lateral rectus is most often affected. If the occulomotor nerve is involved, the pupil is usually spared. Recovery of ocular motor function usually happens within 6 months. C. Myasthenia gravis is characterized by abnormal fatigability of striated muscles which improves after rest. Presenting complaints are ptosis and diplopia from involvement of one or more extraoular muscle. D. Neurologic conditions : Cerebrovascular disorders and CNS space occupying lesions may have strabismus as one of the clinical presentations.. Principles of Management of a Strabismic Patient Aims of strabismus treatment: 1. Good vision 2. Binocularity 3. Good alignment 1. Accurate refractive correction. Treat amblyopia, if present, by patching the better eye. An alternative to patching in certain types of patients may be instilling atropine eye drops to the better eye. 2. Manipulation of accommodation. Esodeviations are treated with antiaccommodative therapy ( plus lenses for hyperopia ) and exodeviations by stimulating accommodation. (overcorrect myopia and undercorrect hyperopia) 3. Prisms. May be useful in patients with acute onset of strabismus and diplopia and those with small deviations. 4. Surgery. Muscles are chosen depending on the type and amount of deviation in the various directions of gaze.. A muscle is strengthened by resection. A measured amount is cut from the muscle which is then sutured back to its original insertion site. Recession in a weakening procedure whereby a muscle is detached from the eye, freed from its fascial attachments and then sutured to the eye at a measured distance from the original insertion.

163

STRABISMUS / 164

Fig 11. Muscle recession

Fig 12 Muscle resection

SUMMARY Under normal binocular viewing conditions, the eyes are aligned and the image of the object of regard falls simultaneously on the fovea of the two eyes. One of the eyes maybe misaligned (strabismus), so that only one eye at a time views the object of regard. Constant strabismus at an early age can result to amblyopia. In addition, any condition which can result to poor vision can lead to strabismus. It is important that a physician is able to detect strabismus at early age so that treatment can result in good vision, binocularity and good alignment. REFERENCES 1. Del Monte, M.A. ed Pediatric Ophthalmology and Strabismus. 1996, San Francisco : American Academy of Ophthalmology. 2. Riordan-Eva, P , Whitcher, J Vaughan and Sahbury’s General Ophthalmology. Lange Medical Books : New York , 2004. 3. Valbuena, M.N. Strabismus and Amblyupia :. An instructional module for Level V Medical Students. 1999 4. Wilson, F.F. ed. Practical Ophthalmology . 1996, Indianapolis : American Academy of Ophthalmology.

164

STRABISMUS / 165

SELF –TEST 1.

The agonist in elevating the right eye is A. R inferior oblique B. R superior rectus C. R superior oblique D. L inferior oblique

2.

The superior division of the oculomotor nerve supplies the superior rectus and A. superior oblique B. inferior oblique C. orbicularis oculi D. levator palpebrae

3.

This test will distinguish phoria from tropia A. cover uncover test B. alternate cover test C. prism cover test D. modified Krimsky test

4.

Example of incomittant squint is A. congenital esotropia B. intermittent exptropia C. accommodative esotropia D. Graves ophthalmopathy

5. Prism measurement of lateral rectus palsy is done with the prism’s base oriented A. in B. out C. up D. down 6. When the angle of deviation is greater in one direction of gaze the strabismus is A. monocular B. comittant C. incomitant D. sensory deprivation 7. After removing the cover in one eye, the eye moved inward. The patient is A. orthophoric B. hyperopic C. esotropic D. exotropic 8. Accommodative esotropia is best treated A. medically B. optically C. surgically D. by observation

165

STRABISMUS / 166

9. When doing the corneal light reflex test, and the light falls temporal to the pupil, the eye is A. esotropic B. exotropic C. hypotropic D. hypertropic E. aligned 10.Choose the correct statement about exotropia. A. Intermittent exotropia resolve spontaneously as the child grows older. B. Exotropia is best treated with spectacle correction. C. Intermittent exotropes close one eye on exposure to bright sunlight. D. Intermittent exotropes usually have amblyopia. ANSWERS TO SELF-TEST 1. B 2. D 3. A 4. D 5. B 6. C 7. D 8. B 9. A 10. C

166

PROPTOSIS / 167

PROPTOSIS Prospero Ma. C. Tuaño, MD INTRODUCTION This self-instructional material on Proptosis is designed for level IV medical students. It is part of a curriculum intended to introduce Ophthalmology through a problem-based approach. Instead of traditionally discussing the eye and its diseases, the medical student is informed of the various complaints that compels a patient to consult an eye specialist. In the real clinical setting there are several ways that the eye will exhibit a disturbance prompting the patient to seek consultation. One of them is proptosis or a bulging eye. OBJECTIVES Upon completion of this unit of instruction, the medical student should be able to recognize a bulging eyeball and explain the reasons for its presence. Specifically, he should be able to: 1. recall the relevant anatomy of the adult human orbit 2. define proptosis and recognize a bulging eye 3. differentiate between a true proptosis and a pseudoproptosis. 4. evaluate the bulging eye in terms of measurement, direction, and dynamics. 5. discuss the various clinical examinations that are utilized in the evaluation of proptosis. PREREQUISITE KNOWLEDGE AND PREPARATION The medical students must have a firm understanding of the anatomy of the eye and its periocular adnexae. He must be familiar with the basic eye examination and the instruments necessary to conduct them. He must be aware that there are other ocular complaints and he must have an overview and understanding of these so that he will be able to correlate and integrate the present subject matter with the rest of the ocular manifestations. INTENDED USERS This manual will be helpful to level IV medical students as an introduction into the subject of eye diseases. Although proptosis is not a common manifestation of eye diseases, it should nonetheless be included in the curriculum because it will contribute to a complete understanding to the problems of the eye. Since Ophthalmology is a subject that will still be included in the medical curriculum of upper year levels, this manual will likely serve to remind the students of the uncommon but significant manifestation of proptosis. CONTENT It is essential that the student reviews the anatomy of the orbit before a significant discussion on a bulging eye is started. The following is a schematic diagram of the adult orbit viewed from above with the roof removed (Fig.1).

167

PROPTOSIS / 168

Fig. 1

Fig. 2

The orbit is described as a 4-sided bony cavity located on both sides of the nose. It serves as a socket for the eye ball and a passage for nerves and blood vessels which supply the eyeball and the periocular adnexa. The orbit is shaped like a pyramid whose orbital margin serves as the base and the optic foramen as the apex. The globe, occupying one -fifth of the orbital volume, appears “connected” to the orbital apex by the optic nerve before the nerve enters the optic canal. The orbital cavity is tightly surrounded by bony walls. The medial walls are parallel to one another and are separated by the ethmoid sinuses. The lateral walls, which have the same length as the medial walls, are directed laterally and outwards, subtending an angle of 45° from the medial walls. Interestingly, a hypothetical posterior extension of the lateral walls makes them perpendicular to each other. The globe is located in the anterior portion of the orbit so that retrobulbar and peribulbar lesions will necessarily disturb the position of the globe. Oftentimes, it is an anterior displacement, either axial or off-axis; rarely do orbital lesions retract the position of the globe. The normal position of the globe in the orbit is marked by a line drawn from the superior to the inferior orbital margin. The straight line theoretically lies tangential to the most anterior portion of the globe, namely the cornea (Fig. 2). Proptosis is the hallmark of orbital diseases. Whereas there are obviously other manifestations of orbital diseases, such as visual loss and diplopia, it is the protrusion of the eyeball which is most unique to the orbit and most striking to the clinician (Fig. 3A). The forward displacement of the globe is also termed exophthalmos. Some physicians use these two terms interchangeably, but most prefer to reserve the term exophthalmos for the description of prominent eyes secondary to endocrine disorders, such as dysthyroid orbitopathy (Fig. 3B). On the other hand, the term proptosis denotes protrusion of the eyeball secondary to an orbital disorder other than dysthyroid orbitopathy.

Fig.3A

Mass in the right orbit

Fig.3B Proptosis secondary to dysthyroid orbitopathy

168

PROPTOSIS / 169

Proptosis may be real or apparent. On the initial encounter with a patient with a bulging eye, the first concern of the clinician is to determine whether the prominence of the globe is a true proptosis or a pseudoproptosis.

Pseudoproptosis

Pseudoproptosis Unilateral macrophthalmos

Lid fissure asymmetry

Contralateral enophthalmos

• Ipsilateral Lid Retraction

• Phthisis Bulbi

• Contralateral Horner’s

• Old Blow-out Fracture

Syndrome

• Scirrhous Adeno CA of breast, lungs, or stomach

• Myopia •Congenital Glaucoma

Table 1 There are three general eye conditions which may assume the image of a pseudoproptosis (Table 1). One may be able to eliminate the possibilities through the help of an accurate history and ocular examination, documentation of a previous ocular trauma or inflammation as well as explicit information regarding past medical histories and treatments. First, lid changes may lead to asymmetry of the lids. The presence of a unilateral lid retraction in dysthyroid orbitopathy may give the impression of an ipsilateral proptosis. On the other hand, a long-standing unilateral lid drooping in Horner’s syndrome may present a contralateral lid retraction (and apparent proptosis) through the principle of Hering’s law. Secondly, a huge globe may be misinterpreted as a proptosed eye. It is frequently observed among high myopes (near-sighted persons) and among pediatric cases with congenital glaucoma. Awareness of the spectacle history and/or the performance of a refraction procedure may detect the presence of myopia. The latter displays a huge globe or a longer axial diameter. An elevated intraocular pressure in an “expandable” pediatric eyeball, as in congenital glaucoma, may likewise lead to an enlarged globe. Lastly, the presence of an abnormally small globe in one side may give the impression of a prominent contralateral globe. Likewise, a normal-sized but retracted globe (enophthalmos) may put on the appearance of a prominent contralateral eye. This situation is caused by an old blow-out fracture or a metastatic tumor to the orbit from a primary scirrhous adenocarcinoma of the breast, lungs or stomach. The causes of true proptosis may be generally classified into the following: vascular, endocrine, inflammation and neoplasm. The mnemonic VEIN is recommended as a helpful tool in remembering the causes of real proptosis. Examples of each classification are mentioned. TRUE PROPTOSIS Vascular – carotico-cavernous fistula Endocrine – thyroid-related eye disease Inflammatory – pseudotumor Neoplasm – lacrimal gland tumor / cavernous hemangioma 169

PROPTOSIS / 170

After determining the presence of a true proptosis, the following evaluation procedure is undertaken to determine the correct etiology of the proptosis. The clinician describes the direction, measurement as well as the dynamics and clinical behavior of the proptosis. DIRECTION The direction of the proptosed globe is predicated on the knowledge of the four surgical spaces within the orbit (Fig. 4). 1. central surgical space or muscle cone – bounded by the 4 rectus muscles 2. peripheral surgical space – bounded by the 4 rectus muscles and the periorbita 3. Tenon’s space – a potential space bounded by the Tenon’s capsule and the outer coat of the eye 4. subperiosteal space- a potential space bounded by the periorbita and the bony orbital walls

Fig. 4

Surgical Spaces of the orbit

Fig. 5

Lacrimal gland tumor

This particular picture (Fig. 5) shows a proptosis which is off-axis. The globe is displaced forward and slightly medially and downward. It can be concluded that the orbital mass is located opposite the direction of the proptosis. The mass must be located superiorly, laterally and posteriorly and outside the muscle cone. Clinical deduction tells us further that the orbital mass is obviously a lacrimal gland fossa lesion, most likely a tumor derived from the lacrimal gland. MEASUREMENT An instrument called an exophthalmometer is used to quantify the amount of proptosis. There are several types. A Luedde exophthalmometer (Fig. 6) is similar to a millimeter ruler. It measures the globe position one at a time. The clinician stands at the side of the patient, places the recessed end of the instrument on the lateral orbital margin and measures the displacement of the globe by reading the millimeter markings on the instrument. The procedure is repeated on the opposite eye. The more common instrument used is the Hertel’s exophthalmometer (Fig. 7).Unlike the previous, it measures the displacement of the two globes at the same time and thus facilitates the comparison of proptosis between the two eyes. Similar to the previous model, the bases of measurement are the most anterior part of the cornea and the lateral orbital margin. The examiner stands in front of the patient and places the instrument on the lateral orbital margin. The image of the cornea is reflected on a mirror on the instrument. Above the mirror is a millimeter ruler which the examiner uses to measure the amount of forward globe displacement (Fig. 8).

170

PROPTOSIS / 171

Fig. 6

Luedde Exophthalmometer

Fig.7

Fig.8

Hertel’s Exopthalmometer

Fig. 9

In order to grossly determine the presence and extent of the proptosis without the benefit of an exophthalmometer, the examiner stands behind a seated patient and peers over the head of the patient. From this position behind the patient the examiner lifts both upper lids. The examiner stares from and above the head of the patient and observes and compares the degree of protrusion of either cornea (Fig. 9). The normal exophthalmometry values among Filipinos have been studied and the following measurements have been derived. In comparison, the values for Caucasians are slightly higher than those for Filipinos. (Table 2) Table 2

Exophthalmometry (mm)

FILIPINOS CAUCASIANS

Average Normal Measurement (mm) 13.5 16.0

Range (mm) 10.0 – 19.5 10.0 – 24.0

DYNAMICS In evaluating the dynamics of proptosis, the following characteristics are considered: 1. resiliency 2. intermittency 3. clinical behavior / pulsating proptosis 4. duration 6. clinical course Information regarding these features may provide valuable hint in the identification of the orbital disorder. They will eventually narrow down the choices in the differential diagnosis and provide a working impression from which a plan of work-up and management will begin.

171

PROPTOSIS / 172

RESILIENCY It is normal to be able to retrodisplace the globe by applying your thumb over the eyeball. In the absence of any orbital pathology, one can easily push the eyeball posteriorly because of the compressibility of the orbital tissues such as orbital fat. In the presence of a solid retrobulbar space-occupying lesion, the globe is prevented from being pushed backward towards the orbit. This is reported as negative resiliency. It is more convenient to push the globes simultaneously in order to facilitate comparison of the two orbits.

Fig. 10 Test for resiliency of the globe

Fig.11

Capillary hemangioma

INTERMITTENCY Intermittent proptosis refers to varying degrees of eye protrusion as a function of change in the immediate environment of the patient. A stimulus may be internal such as a systemic infection or external like a change in head posture or position. The proptosis is noted to increase in size followed by a spontaneous resolution after the stimulus. The proptosis in a child with capillary hemangioma may increase noticeably fast when he is crying but resolves soon after the effort (Fig. 11). Another instance is an adult patient with a varix or abnormally expansile venous channels. The proptosis exacerbates when he bends forward into a prone position or strains during a Valsalva maneuver. Certain vascular tumors, like a lymphangioma, may produce a sudden exaggerated eye protrusion in the presence of an upper respiratory tract infection. This expansion is caused by bleeding within the lymph channels of the tumor, leading to the formation of “chocolate cysts”. The lymphangioma is expected to resolve spontaneously within a few months of conservative management (Fig. 12 A-C).

Fig. 12 A Lymphangioma in a 10-year old child:

Fig 12 B

Fig 12 C

172

PROPTOSIS / 173

Fig 12A shows the child upon initial consultation with a proptosed right globe with extensive conjunctival chemosis. Fig 12B demonstrates spontaneous resolution after 2 weeks. Fig 12C exhibits complete resolution of proptosis and periorbital swelling. CLINICAL BEHAVIOR / PULSATING PROPTOSIS Pulsating proptosis, characterized by rhythmic pulsations of the globe, may occur in cases of caroticocavernous fistulas of high-flow quality. The globe exhibits dilated and tortuous episcleral vessels because of intense congestion and “arterialization” of the venous channels of the orbit (Fig 13). Pulsating proptosis may also be exhibited by congenital bony defects in the orbital roof of patients with orbital neurofibromatosis. The pulsations reflect the same cadence as the peripheral arterial pulsations.

Fig.13 Arterio-venous fistula with dilated & tortuous episcleral veins (corkscrew vessels)

Fig 14 Orbital cellulitis

DURATION How long the disease has been going on is a question which leads the clinician to ask about the duration of the orbital disorder. An acute onset is accompanied by a short history prior to consultation. It implies a rapidly-evolving disease entity like a malignancy or an inflammatory condition such as an orbital cellulitis or orbital pseudotumor (Fig 14). On the other hand, a chronic condition spanning years of clinical history before clinical consultation may allude to the possibility of a benign tumor. The most common primary benign tumors of the orbit include a cavernous hemangioma and a pleomorphic adenoma of the lacrimal gland. There are subacute orbital conditions which are neither acute nor chronic. These include orbital disorders like dysthyroid orbitopathy, lymphomas and some metastatic carcinomas. They have an insidious presentation prior to a more rapid progression in the later stages of the disease. CLINICAL COURSE The clinical course describes the growth characteristics of the tumor. It also provides information on the rate and direction of evolution of the mass from the time it is first noted by the patient up to the time when the clinician starts to observe the disease process. Many benign orbital tumors, such as cavernous hemangioma and pleomorphic adenoma of the lacrimal gland are slowly progressive. Some benign tumors, like neurofibromas and optic nerve gliomas, may remain stationary or at the very least, are slowly progressive.

173

PROPTOSIS / 174

Most malignant tumors, like rhabdomyosarcoma and adenoid cystic carcinoma of the lacrimal gland, besides having a short duration of history, exhibit rapid progression. The clinical course of orbital cellulitis may be even more rapid and appropriately described as explosive. On the other hand, there are tumors which may spontaneously stop and regress. This is exemplified by capillary hemangioma which grows rapidly during the first year of life, then stops and continues regressing till the early teens.

Fig. 16

Fig. 15

Fig.15 depicts a 56-year old male with a cavernous hemangioma which was noted 17 years prior to initial consultation. Fig. 16 shows 19-year old female diagnosed with diffuse toxic goiter. Her rapid proptosis of less than a month duration has resulted in severe bilateral lagophthalmos and injurious exposure changes to the cornea.

6 months old Fig 17 is a collage of pictures of a female with capillary hemangioma from the time it was detected at 6 months of age up to 10 years old. There are two lesions (indicated by the red arrows): one on the left upper lid and another on the scalp. The regressing size of the lesion on the left upper lid is noted as the patient grows older.

174

PROPTOSIS / 175

CLINICAL EVALUATION The patient with a true proptosis undergoes a thorough clinical evaluation. Utilizing the same routine eye examination applied to any eye patient, certain special considerations are emphasized. For instance, in the gross examination, there is particular attention focused on lid and conjunctival changes. Lid retraction and/or lid lag are almost pathognomonic of dysthyroid orbitopathy (Fig. 18). Pupillary reaction is also doubly appraised because of optic nerve dysfunction secondary to compression by a tumor or enlarged extraocular muscles. Furthermore there are novel maneuvers such as auscultation which elicit bruits over the periocular adnexae. This is indicative of an arteriovenous fistula (Fig. 19). This is reinforced by a sensitive hand in palpation which elicits a corresponding rhythmic pulsation from the same periorbital area. METHODS OF ORBITAL DIAGNOSIS: CLINICAL 1. Gross Examination - lid and conjunctival abnormalities 2. Palpation - pulsation, resiliency, anteriorly-located mass 3. Auscultation 4. Routine Eye Exam - Vision, extraocular muscles, intraocular pressure, ophthalmoscopy 5. Color vision 6. Cranial nerve function - pupil, corneal reflexes 7. Systemic examination

Fig. 18 Bilateral lid retraction

Fig. 19 Auscultation of the orbit in dysthyroid orbitopathy

METHODS OF ORBITAL EXAMINATION: RADIOGRAPHY 1. Venography 2. Arteriography 3. Plain film 4. Computerized tomography (CT scan) 5. Magnetic resonance imaging (MRI) Radiography is an essential tool among orbital patients. There is an absolute need to visualize the concealed structures of the orbital cavity. The usefulness of radiography was not evident with the first available machines for plain X-ray film because only the bony walls are readily seen. The advent of computerized tomography revolutionized the diagnosis and treatment of orbital diseases because it was able to view and distinguish the soft tissues within the orbit. Further enhancement of the visualization was achieved with magnetic resonance imaging which offered a discriminating picture of the orbital apex. Plain radiography has been relegated to detection of bony abnormalities such as fractures and bony growths.

175

PROPTOSIS / 176

Venography and arteriography have remained part of the history of orbital radiography. They are still both useful but limited in use for particular diseases with vascular dynamics. For instance, the management of carotico-cavernous fistulas demands the localization of the fistula through selective angiography before embolization treatment can be planned. Other non-radiographic tests are available to augment the information already derived from the previous examinations. They are essential in identifying the disease process so that the proper management can be instituted. In some cases, they may be used to monitor the progress of the disease in order to achieve proper timing of intervention. 1. Ultrasonography 2. Visual field examination – for optic nerve evaluation in dysthyroid orbitopathy, AV fistula, and optic nerve tumors 3. Electrophysiology – for optic nerve evaluation as well 4. Laboratory exams – thyroid function tests 5. Tissue biopsy SUMMARY Proptosis is an ocular complaint and manifestation which occurs rather infrequently in comparison with other common complaints such as headache, redness and blurring of vision. Despite its relative rarity, the complaint must be evaluated because it is peculiarly an orbital complaint. The most important value of learning the nature and behavior of proptosis is the knowledge that this manifestation contributes its own share of problems to the well-being of the eye. Proptosis carries a difficulty of identifying the lesion because the latter exists behind the eyeball where it is “invisible” to routine eye examination. As such, the clinician needs to conduct not only the routine eye examination but also other recommended steps in a systematic evaluation of the bulging eye. It is primarily important to determine if the proptosis is real or apparent. The differential diagnoses of pseudoproptosis include unilateral myopia, lid fissure asymmetry and contralateral enophthalmos. True proptosis, on the other hand, undergoes the further clinical evaluation, namely, measurement, direction and dynamics or clinical behavior. In general, the clinical considerations include lesions which are vascular, endocrine, inflammatory and neoplasm. Clinical examination is followed by the use of ancillary procedures such as CT scan or MRI of the skull and orbit. Other laboratory examinations include visual field tests, biopsy procedures on accessible tumors, ultrasonography, electrophysiology and selective carotid angiography. REFERENCES 1. American Academy of Ophthalmology. Basic and Clinical Science Course Sec 7 Orbit, Eyelids and Lacrimal System, 2003. 2. Jones IS and Jakobiec, FA, eds. Diseases of the Orbit, Harper & Row, 1979. 3. Laws, ER Jr., ed. The Diagnosis and Management of Orbital Tumors, Futura Publishing Co;1988. 4. Vaughn DG, Asbury T, Riordan-Eva P. General Ophthalmology, 13th ed. Norwalk, CT: Appleton & Lange; 1992

176

PROPTOSIS / 177

SELF-TEST 1. The axis of the adult bony orbit is directed forward and: A. laterally B. medially C. upward D. in the same axis as the globe. 2. The volume of the orbit is approximately 30 cc. The volume of the globe is around: A. 2 -3 cc B. 6- 7 cc C. 10 – 11 cc D. 14 – 15 cc 3. The antero-posterior diameter of the globe is around 24 mm. The maximum depth of the orbit is approximately : A. 20.0 mm B. 30.0 mm C. 40.0 mm D. 50.0 mm 4. The normal range of exophthalmometry measurements among Filipinos is: A. 9.0 – 14.5 mm B. 10.0 – 19.5 mm C. the same as Caucasians D. still unknown / unreported 5. Pseudoproptosis is evident in the following situation: A. ipsilateral Horner’s syndrome B. pseudotumor C. axial myopia D. elevated intracranial pressure 6. A proptosis which increases after a Valsalva maneuver is probably due to: A. a cavernous hemangioma B. a capillary hemangioma C. a thyroid-related eye disease D. an inflammatory pseudotumor 7. Acute proptosis (rapidly-progressive proptosis) of the globe is noted in the following, except: A. cavernous hemangioma B. thyroid-related eye disease C. inflammatory pseudotumor D. rhabdomyosarcoma 8. Enophthalmos after a blunt trauma to the anterior part of the orbit is due to: A. a fracture of the roof and lateral wall of the orbit B. a fracture of the floor and medial wall of the orbit C. rupture of the orbital fat cells D. paresis of all the EOM’s

177

PROPTOSIS / 178

9. One suspects a lacrimal gland tumor if the globe is proptosed forward… A. downward and medially B. upwards and medially C. downwards and laterally D. upwards and laterally 10. CT scan of the orbit is most needed in the following situations: A. proptosis with palpable lid masses B. proptosis with negative resiliency C. proptosis with positive resiliency D. pseudoproptosis with palpable lid masses ANSWERS 1. 2. 3. 4. 5.

A B D B C

6. B 7. A 8. B 9. A 10. B

178

RETINOBLASTOMA / 179

RETINOBLASTOMA Rolando Enrique D. Domingo M.D. INTRODUCTION This self instructional material is designed for the undergraduate medical student. It gives an overview of the life threatening eye disease known as retinoblastoma. Although it possesses basic information on this disease it is also meant to encourage the student to further reading. Knowledge acquired from this material will be useful in clinical practice as well as further training in Ophthalmology. OBJECTIVES After reading and understanding this module, the medical student is expected to: 1. Identify through history taking and physical examination clues, signs and symptoms leading to a diagnosis of retinoblastoma. 2. Recognize and differentiate conditions which may present with signs and symptoms similar to retinoblastoma. 3. Develop an appropriate sense of urgency once faced with a patient possibly suffering from retinoblastoma. 4. Have a general idea of the therapeutic modalities available to a patient with retinoblastoma. RECOMMENDED PREPARATION The student must have working knowledge of the anatomy of the eye. He must also possess basic skills in medical history taking and comprehensive eye examination, especially in children. INTENDED USERS This module is designed primarily for the undergraduate medical students of the UP College of Medicine rotating in the Department of Ophthalmology and Visual Sciences. CONTENT I. Background Retinoblastoma is the most common intraocular malignancy of childhood. Its incidence is approximately 1:17000 live births, however some series have reported an increased incidence in the last few decades. In the PGH setting, being a tertiary center, we see around 100 new patients every year, or an average of two per week. II. Genetics The development of retinoblastoma can be traced to mutations on chromosome 13. The retinoblastoma gene is one the best studied genes in the human genome. It is a tumor suppressor gene whose presence (even of a single allele) protects against the development of the tumor.

179

RETINOBLASTOMA / 180

Most cases (70%) of retinoblastoma are sporadic mutations. This means that one single retina cell suffers a mutation on one allele of the RB gene, then after some time suffers another mutation on the remaining allele. The loss of the tumor suppression from both alleles then allows that particular cell to multiply uncontrollably (Knudson’s Two Hit Hypothesis). However, in 30% of cases the condition is heritable. In this instance the child inherits a normal chromosome 13 from one parent and a mutated chromosome 13 from the affected parent. Therefore, all the retina cells (in fact, all cells in the body) have one mutation at birth. Retinal cells are very metabolically active and are constantly exposed to light energy, in most instances these children will develop a second mutation in one or more cells producing retinoblastoma. Although the average age for the diagnosis of retinoblastoma is 18 months, patients with sporadic mutations develop tumors later (mean age at diagnosis 24 months) because two hits are needed on a single cell. Sporadic cases are unilateral. Children with the heritable variety are diagnosed earlier at a mean age of 12 months, because only a second hit is needed after birth and all retinal cells are at risk. These cases are usually bilateral with multiple tumors on both eyes. Clinically, heritable retinoblastoma behaves as an autosomal dominant trait with marked penetrance. Retinoblastoma is very rarely seen after the age of six. III. History In a child suspected to have retinoblastoma it is important to ask about the birth and maternal history. Factors such as preterm birth, birth weight, family history of metabolic disorders, childhood blindness and death are important. Occasionally, a complete history is enough to rule out retinoblastoma. The most common presenting symptom of retinoblastoma is leukocoria (white pupil) also called cat’s eye reflex or matang pusa, in Filipino. This is seen in two thirds of retinoblastoma patients with intraocular tumors. See figure 1.

Fig. 1 Leukocoria

If the tumor is located in the posterior part of the retina then the leukocoria may be constant. However, if it is located in the periphery, the cat’s eye reflex might be seen only on certain directions of gaze. Retinoblastoma may occasionally present as strabismus, inflammation or mimic infection. Sometimes tumor cells seed into the anterior chamber and settle inferiorly giving the appearance of pus (hypopion) .When the tumor grows it may cause glaucoma and, later, buphthalmos or enlargement of the eye. Advanced tumors that

180

RETINOBLASTOMA / 181

grow beyond the confines of the globe (extraocular extension) present with exophthalmos and an orbital mass (fig. 2). At this stage intracranial extension via the optic nerve and hematogenous metastasis is likely.

Fig.2 Retinoblastoma with extraocular extension

IV. Physical Examination A complete ocular and physical exam on a child suspected to have retinoblastoma is imperative. Unilateral visual loss usually goes unnoticed and must be tested in a child. Gross examination will usually show a white pupil. Great care must be taken to ascertain the cause of the leukocoria, as this may be caused by opacities in the cornea, lens or vitreous other than a tumor. A dilated funduscopy will show a yellow, white or pink mass. Indirect ophthalmoscopy gives a more panoramic and three dimensional view. See fig. 3.

Fig. 3 Intraocular retinoblastoma

A firm eye and an eye that is larger than the other are signs of an enlarging tumor. And an extraocular tumor, aside from gross picture might also result in extraocular muscle restriction. In these cases a systemic and neurologic evaluation of the child should be done to detect signs of metastasis.

181

RETINOBLASTOMA / 182

V. Differential diagnosis There are several conditions that may present with leukocoria other than retinoblastoma. A white pupil may be due to a corneal scar or an opacity in the lens such as the developmental cataract shown below. Cataracts may be congenital or may develop as the child grows. A careful examination is usually helpful in identifying the lens as the cause of the white reflex.

Fig.4 Congenital cataract presenting with leukocoria.

Pathologies posterior to the lens may also cause a cat’s eye reflex which may be more difficult to differentiate from retinoblastoma. A condition commonly mistaken for retinoblastoma is retinopathy of prematurity or ROP. In advanced ROP the retina may become detached, disorganized and form a mass like structure causing leukocaria. A history of preterm birth less than 28 weeks gestation and birth weight less than 1.5 kg will point us in the right direction. Persistent hyperplastic primary vitreous (PHPV) is another condition which may mimic retinoblastoma. The leukocoria is present at birth and the affected eye is usually smaller (microphthalmia). There is a fibrous plaque behind the lens connected through a stalk to the optic disc. This is an anomaly of embryogenesis which is otherwise benign Coats’ Disease involves telangiectatic changes in the retina of children. The blood vessels leak and cause subretinal exudation, retinal detachment and scarring. If the affected portion of the retina is large enough then it may cause leukocoria, although these patients are usually older than those affected with retinoblastoma. A distinct retinal mass would not be seen on funduscopy. Other more unusual diseases such as retinal dysplasia, parasitic endophthalmitis and tuberculosis may present as retinoblastoma. In all cases the need for a good history and thorough physical examination can never be overemphasized. VI. Ancillary Tests Although a complete ophthalmologic exam including indirect ophthalmoscopy leads to an accurate diagnosis of retinoblastoma in the great majority of patients, sometimes ancillary tests might be useful in the few equivocal cases. An ocular ultrasound (B scan) is readily available in most eye centers and can easily demonstrate the presence of a distinct mass in cases wherein there is doubt (see picture below) especially if there is an opacity in the cornea or lens occluding the view.

182

RETINOBLASTOMA / 183

Fig. 5 Ultrasound photo showing a solid mass at the center.

However, by far the most useful ancillary procedure in the diagnosis of retinoblastoma is still a CT scan. One particular characteristic of retinoblastomas is that calcification is seen in more than 90% of tumors and the CT scan easily demonstrates this (see picture below). On the other hand, all other childhood eye pathologies very rarely calcify before the age of seven. Therefore, a child less than six years old with an intraocular mass showing calcifications on CT scan is almost certainly suffering from retinoblastoma.

Fig. 5 CT scan showing intraocular tumor with calcification on the left.

A deadly characteristic of retinoblastoma is its propensity to invade the optic nerve, and in the advanced stage spread directly into the brain. The CT scan is also useful in documenting this and guides the physicians in treatment planning. VII. Management Once a patient is diagnosed with retinoblastoma the situation must be treated as urgent. Visual loss is something to be considered but the threat to life is of utmost importance because untreated retinoblastoma is almost uniformly fatal.

183

RETINOBLASTOMA / 184

The management of retinoblastoma is performed by a team primarily consisting of an ophthalmologist, a pediatrician and a radiologist. Small tumors such as that seen in Fig. 6 below may be treated conservatively.

Fig. 6 Small intraocular retinoblastoma.

These small tumors may be treated with lasers or cryotherapy depending on the size and location. Medium sized tumors may be treated with chemotherapy combined with the previously mentioned modalities. Radiation therapy is also sometimes used, either alone or in combination, to save an eye with retinoblastoma. Larger tumors occupying more than half of the eye and with no hope for vision are enucleated. It is imperative for enucleated eyes to undergo histopathologic examination to look for signs of extraocular spread. Tumors with extraocular extension have higher rates of metastasis and result in poorer prognosis. Extraocular extension is an indication for chemotherapy post operatively. Patients with large tumors involving the orbit have the worst prognosis. They are treated with chemotherapy and more extensive surgery. The newer chemotherapeutic drugs such as the platinum compounds are increasing the survival rates of these patients. Medical therapy is commonly used in combination with radiation treatment. Retinoblastoma patients must be closely followed up for signs of recurrence or development of new tumors. Heritable cases are especially at risk and have been reported to develop other malignancies in later life. Genetic counseling for affected families is also necessary. VIII. Summary Retinoblastoma is a life threatening eye disease in children. It is important to differentiate it from other benign conditions because delay in treatment may mean not only loss of vision but, more importantly, loss of life. A high index of suspicion is needed when confronted with a child showing leukocoria or other signs and symptoms suggestive of retinoblastoma. A complete history and physical examination will usually lead to an accurate diagnosis which is necessary to start the appropriate treatment.

184

RETINOBLASTOMA / 185

REFERENCES 1. Charl , Devron . Clinical Ocular Oncology, Churchill Livingsotne Inc. New York, NY, 1989. 2. Mc Lean, I.W.; Burnier, M.N.; Zimmerman, L.E., Jakobiec, F.A.Tumors of the Eye and Ocular Adnexa Armed Forces Institute of Pathology , Washington, DC, 1993. 3. Fine, B.S. and Yanoff, M.,Ocular Histology: A Text and Atlas, 2nd ed. Harper and Rowe, Hagerstown, MD, 1979. SELF-TEST 1. The most common presentation of retinoblastoma is: a. Blurring of vision b. Cat’s eye reflex c. Red eye d. Squint 2. A 6 month old child with bilateral leukocoria since age one month who was kept in an incubator for six weeks after delivery probably has: a. Heritable retinoblastoma b. Persistent Hyperplastic Primary Vitreous (PHPV) c. Retinopathy of Prematurity d. Coats’ disease 3. A one year old child is brought to you with a smaller right eye with leukocoria initially noticed one month after birth. The most probable diagnosis is: a. Retinoblastoma b. PHPV c. ROP d. Coats’ disease 4. A 3 year old child with leukocoria is presented to you. On examination the cornea is hazy and the anterior chamber is not very clear. What examination would be most helpful in definitive diagnosis of this patient? a. CBC and peripheral blood smear b. Ultrasound c. CT scan d. Bone scan 5. A four year old child with left sided leukocoria and exotropia is diagnosed with retinoblastoma, the most likely treatment modality is: a. Laser treatment b. Chemotherapy c. Enucleation d. External beam radiation. 6. The most pertinent histopathologic finding in an eye enucleated for retinoblastoma is: a. Mass occupying the entire vitreous cavity. b. Tumor seeding in the anterior chamber with glaucoma. c. Choroidal involvement. d. Invasion of the optic nerve beyond the margin of resection. 185

RETINOBLASTOMA / 186

7. A six month old child underwent enucleation of the right with intraocular retinoblastoma, the other eye is normal. On discharge you would advice the parents: a. That they should relax since the tumor was completely removed and no further follow up is needed. b. To watch closely the enucleated side for recurrence. c. To bring the patient back every few months to check if there are tumors developing on the remaining eye. d. To see the pediatric oncologist for post op chemotherapy. 8. A man with bilateral retinoblastoma marries and the couple decide to have children, they should: a. Not worry because the chance of having children with retinoblastoma is very small. b. Bring their children to the doctor once they see leukocoria or other signs of retinoblastoma. c. Take their children to be seen by an ophthalmologist the soonest time possible after birth. d. Change their minds and not have any children.

Answers 1. 2. 3. 4. 5. 6. 7. 8.

B C B c C D C C

186

OCULAR MANIFESTATIONS OF SYSTEMIC DISEASES / 187

OCULAR MANIFESTATIONS OF SYSTEMIC DISEASES Romulo N. Aguilar, MD, PhD Teresita R. Castillo, MD, MHPEd INTRODUCTION This self-instructional material is designed to help the student learn key concepts concerning eye manifestations of common systemic diseases. It aims to emphasize the critical role of Primary Care Physicians in preventing visual loss by appropriate recognition, treatment and referral to the ophthalmologist. Many systemic conditions can present with significant ocular manifestations. Certain ocular signs and symptoms may signal the presence of serious underlying systemic disorders. As such, data gathered from an eye examination can provide the clinician with clues that may serve as aid in the diagnosis and management of the underlying systemic disease. While some ocular findings may be non-specific, certain findings may be highly suggestive of the presence of one or more diseases. OBJECTIVES Upon completion of this unit of instruction, the student should be able 1. To recognize characteristic ocular features of systemic diseases, specifically those associated with the following conditions • Diabetes Mellitus • Hypertension • HIV / AIDS • Thyroid Disease • Tuberculosis 2. To determine when it is appropriate to refer a patient to an ophthalmologist for consultation or treatment. PREREQUISITE KNOWLEDGE AND PREPARATION Students should have a working knowledge of the basic anatomy of the eye and its adnexa. The student should also possess a working knowledge of common ocular signs and symptoms. It is advised that written materials regarding the above topics be completed first prior to working on this instructional material. INTENDED USERS This material was developed to provide the medical student with concepts and information on the various ocular manifestations of systemic diseases that is more commonly encountered by a primary physician. It is best that this material be supplemented by clinical exposure to provide the students with actual cases. CONTENT The eye provides clues to the diagnosis of many systemic diseases and many important complications of these diseases occur in the eye. Evaluation of the ocular fundus is particularly important in the evaluation in systemic disease since it is the only region in the body where one can directly visualize manifestations of macro and microvascular pathology.

187

OCULAR MANIFESTATIONS OF SYSTEMIC DISEASES / 188

An outline of the more significant of these diseases along with their ocular presentation is summarized in Table 1. A review of the important ocular pathology seen in some systemic diseases will subsequently be discussed. Table 1. Overview of Ocular Manifestations of Common Systemic Diseases System/category

Disease

Ocular manifestation

Endocrine

Diabetes mellitus Thyroid eye disease Pituitary lesions

Diabetic retinopathy Thyroid orbitopathy Field loss

Cardiovascular

Hypertension Retinal Emboli Hyperlipoproteinemia Marfan Syndrome Endocarditis

Hypertensive Retinopathy Retinal Vessel Occlusion Corneal Arcus/ Xanthelasma Lens Dislocation Roth Spots

Rheumatology

Rheumatoid Arthritis Sjogrens Syndrome Seronegative Spondeloarthropathies Collagen Vascular Disease Juvenile Rheumatoid Arthritis

Scleritis Keratitis sicca Uveitis Keratitis sicca Anterior Uveitis

Infections

Herpes zoster HIV, CMV Candidiasis Syphilis

Uveitis, glaucoma Retinitis Endophthalmitis Iritis, Optic Neuritis, Ophthalmoplegia

Malignancy

Lymphoma Leukemia Metastases

Infiltrate, uveitis Infiltrative retinitis Choroidal mass

Neurological

Multiple sclerosis Giant cell arteritis Myasthenia gravis

Optic neuritis, uveitis Ischemic optic neuropathy Diplopia, ptosis

Dermatological

Acne rosacea Atopy

Keratitis Keratitis, cataract

DIABETES MELLITUS

Diabetes (DM) is associated with a number of ocular complications. These include cataracts, EOM palsies (Cranial Nerves III, IV, VI), diabetic optic neuropathy, neurotrophic keratitis and diabetic retinopathy. Among these, diabetic retinopathy is the most common and may lead to permanent loss of vision if not attended to. According to the 1995 Second National Survey of Philippine Blindness, 1.47% Filipinos suffer from bilateral blindness from vascular diseases including DM Retinopathy and was the 6th leading cause of blindness among Filipinos. Studies have shown that at any one time, 25% of the total diabetic population will have diabetic retinopathy, and if followed through time, 90% will eventually develop the condition. The prevalence of diabetic retinopathy has been found to be related to the duration of the systemic condition (Table 2). Other risk factors which have been identified to be associated with the development of diabetic retinopathy include the following: • Hypertension • Renal status • Onset of puberty

188

OCULAR MANIFESTATIONS OF SYSTEMIC DISEASES / 189



Hb A1c level Table 2. Relation of Diabetic Retinopathy to the Duration of the Disease Duration 0 – 5 yrs

IDDM

NIDDM

0%

10 – 15 yrs

25 – 50%

23 – 43%

16 – 20 yrs

75 – 95%

60%

30+ yrs

100%

Although the likelihood of developing retinopathy varies between Type I and Type II diabetes, generally, the risk of retinopathy increases with the duration of the disease. Almost 100% of Type I patients will develop retinopathy after 15 years of diabetes. The more severe form of proliferative diabetic retinopathy does not appear at all until disease duration of at least 10 - 15 years. Elevated glucose is thought to damage retinal (and renal) capillaries in the following ways: (1) capillary basement membrane thickening (2) loss of capillary pericytes and (3) breakdown of the blood-retinal barrier. As a result of this process, the blood vessels can leak, they produce a special growth chemical (VEGF= vascular endothelial growth factor) responsible for triggering new blood vessel growth and the vessels may eventually close and block.

Figure 1. Pathogenesis of DM Retinopathy

189

OCULAR MANIFESTATIONS OF SYSTEMIC DISEASES / 190 Table presents a summary of the various stages of diabetic retinopathy and its hallmark characteristics. Table 3. Classification of Diabetic Retinopathy DISEASE STAGE

HALLMARK CHARACTERISTICS

Non-proliferative Diabetic Retinopathy Mild

Microaneurysms ohly

Moderate

Microaneurysms Hard exudates Macular edema* Intraretinal hemorrhages

Severe

Microaneurysms (multiple) Intraretinal hemorrhages (diffuse) IntraRetinal Microvascular Abnormalities (IRMAs) Venous beading Soft exudates

Proliferative Diabetic Retinopathy

Neovascularization of the disc (NVD) Neovascularization of the retina elsewhere (NVE) Fibrovascular scar Tractional Retinal Detachment

A

B

Figure 2. Microaneurysms are the earliest clinically visible changes of diabetic retinopathy. They are localized capillary dilatations which are usually saccular (round). They usually appear as small red dots in clusters (A) although they may also be isolated (B). Figure 3. Intraretinal Hemorrhages may be ‘dot’ or’ blot’ shaped (termed ‘dot/blot hemorrhages’) or flame shaped depending upon their depth within the retina. The capillary network in the posterior retina is found in two layers; a superficial one in the nerve fiber layer and a deeper on within the inner nuclear layer. Hemorrhage within the nerve fiber layer tends to be flame shaped, following the divergence of axons (1). In the inner layer, haemorrhage is aligned at right angles to the retinal surface and is consequently viewed end-on when using an ophthalmoscope; these hemorrhages appear dot (2) or blot shaped (3).

190

OCULAR MANIFESTATIONS OF SYSTEMIC DISEASES / 191

Figure 4. Cotton-wool spots are grayish-white patches of discoloration in the nerve fiber layer which have indistinct (fluffy) edges. They are the result of local ischemia which leads to disruption of axoplasmic flow.

Figure 5. Hard exudates are distinct yellow-white intra-retinal deposits which can vary from small specks to large patches. They may evolve into rings known as circinates and form large confluent plaques.

Figure 6. Venous beading is a sign of retinal ischemia.

Figure 7. (IRMA) Intraretinal microvascular abnormalities are areas of capillary dilatation and intraretinal new vessel formation They arise within ischemic retina and when they are present in numbers are a feature of preproliferative retinopathy.

191

OCULAR MANIFESTATIONS OF SYSTEMIC DISEASES / 192

A

B

Figure 8. Neovascularization or new vessel formation occurs as the retina becomes more ischemic. New blood vessels may arise from the optic disc (NVD)-A; or elsewhere in the periphery of the retina (NVE)-B. These new vessels are fragile and bleed easily, hence the importance of proliferative retinopathy.

1

1

2

3

2 3 3 1

Figure 9. Mild Non Proliferative Diabetic Retinopathy with microaneurysms.

Figure 10. Moderate Pre-proliferative Diabetic Retinopathy with typical findings of multiple cotton wool spots (1), hard exudates (2) and intra-retinal hemorrhages (3).

1 3

2 Figure 11. Severe Non-proliferative diabetic retinopathy with multiple microaneurysms (1), soft exudates (2) and diffuse intraretinal hemorrhages (3).

Figure 12. Proliferative Diabetic Retinopathy with new vessels at the disc and retinal periphery.

192

OCULAR MANIFESTATIONS OF SYSTEMIC DISEASES / 193

Figure 13. Vitreous hemorrhage can give rise to profound loss of vision if the macula is obscured. Only a small amount of bleeding is required since blood dissolved in the vitreous produces a haze effect which impairs vision (and ophthalmic assessment).

Figure 14. Subhyaloid hemorrhage refers to accumulation of vitreous blood in areas of localized detachment. Blood accumulates the retina and the vitreous and is often described as boat-shaped.

Figure 15. Retinal Fibrosis and fibrovascular proliferation occurs as a consequence of bleeding from the new vessels. This subsequently leads to traction on the retina and loss of vision.

The primary goal in the treatment of diabetic retinopathy is the prevention of visual loss. Diabetic patients without evidence of diabetic retinopathy will benefit from early detection of the condition. Patients with IDDM (Type 1) should be referred for ophthalmologic consult if the disease is of five years duration already. All patients diagnosed to have NIDDM (Type 2 Diabetes) should be referred to an ophthalmologist at the time of initial diagnosis. Diabetic patients who complain of any visual symptoms should be referred immediately to an eye specialist for proper evaluation. Pregnant women with history of diabetes should likewise be referred to an eye specialist for proper monitoring of her ocular status for the duration of her pregnancy. Foremost in the management of patients with any stage of diabetic retinopathy would be strict blood sugar control. Management of the eye condition would depend on the stage of the retinopathy. Patients classified to have mild to moderate NPDR should have regular fundus examination and fluorescein angiography. Grid or focal laser photocoagulation should be considered for patients with severe NPDR and/or clinically significant macular edema (CSME). Panretinal photocoagulation is the intervention of choice for patients with proliferative diabetic retinopathy. Complications such as vitreous hemorrhage and traction retinal detachment are managed by the performance of vitreoretinal surgery.

193

1

OCULAR MANIFESTATIONS OF SYSTEMIC DISEASES / 194

Figure 16. Fundus Photographs (top) with corresponding Fluorescein Angiogram pictures.

Figure 17. Grid Laser Treatment

Figure 18. Focal Laser Treatment

Figure 20. Laser Scars secondary to laser treatment.

Figure 19. Panretinal photocoagulation

194

OCULAR MANIFESTATIONS OF SYSTEMIC DISEASES / 195

HYPERTENSIVE RETINOPATHY Systemic arterial hypertension is one of the most common diseases worldwide. More than half of the population over 60 years has this condition. The changes leading to primary systemic arterial hypertension is multifactorial. In the elderly, an increase in basal smooth muscle tone occurs as a result of sympathetic activity, rennin-angiotensin overactivity, cell membrane changes and progressive architectural alterations in the vessel walls. Other factors which contribute to this are salt sensitivity, volume depetion and orthostasis. The overall incidence of hypertensive retinopathy is 15%. The ocular picture is directly related to status of retinal arteries and the rate of rise and degree of systemic blood pressure. Arterial sclerosis occurs in the normal aging population (involutional sclerosis) as well as in long standing hypertension. Retinal manifestations of hypertension include vascular constriction, leakage and arteriosclerosis. Vasoconstriction manifests with generalized or focal arteriolar narrowing. Leakage occurs due to abnormal vascular permeability. This manifests as flame-shaped hemorrhages, retinal edema and hard exudates. Optic disc edema occurs in cases of malignant hypertension. The presence of hard exudates in Henle’s layer of the fovea presents in a star-like configuration, the “macular star”. Arteriosclerosis occurs as a result of intimal layer hyalinization, medial layer hypertrophy and endothelial hyperplasia. Clinically, this presents as focal narrowing and straightening of the retinal arterial walls. Arteriovenous (AV) crossing changes occurs as the condition progresses.

Figure 21. Macular Star – hard exudates in the fovea.

Grading systems for Hypertensive Retinopathy have been presented by various authors. The Keith Wagener Barker Classification was published in 1939 which classifies the condition to four stages as follows: Table 4. Keith Wagener Barker Classification of Hypertensive Retinopathy STAGE

CHARACTERISTICS

I

mild to moderate arteriolar narrowing or sclerosis

II

moderate to marked arteriolar narrowing with focal or neneralized narrowing, exaggerated light reflex and AV crossing changes

III

the above PLUS cotton wool spots, hard exudates, retinal hemorrhages, extensive microvascular changes and retinal edema

IV

the above PLUS disc edema

195

OCULAR MANIFESTATIONS OF SYSTEMIC DISEASES / 196

A more recent grading system was proposed by Scheie. This classification system takes into consideration two aspects of the condition: changes due to hypertension and changes due to arteriosclerosis. Tables 5 and 6 present these. Table 4. Scheie Classification for Grading Hypertensive Retinopathy – Changes due to Hypertension GRADE

CHARACTERISTICS

I

slight, generalized arteriolar attenuation (AV ratio 1:2)

II

obvious arteriolar narrowing with focal areas of attenuation (AV ratio 1:3)

III

the above PLUS exudates and hemorrhages

IV

the above PLUS optic disc edema

B

A

1

1 2

1

1 1 D

Figure 22. Schiei Classification based on Hypertension: A – Grade I ; B – Grade II; C – Grade III; D – Grade IV showing swollen optic nerve, retinal arteriolar narrowing, nerve fiber layer infarcts (1) and blot hemorrhage (2).

C

196

OCULAR MANIFESTATIONS OF SYSTEMIC DISEASES / 197

Table 5. Scheie Classification for Grading Hypertensive Retinopathy – Changes due to Arteriolar Sclerosis GRADE

Arteriolar Reflex Changes

Blood Vessel Changes

I

broadening of the arteriolar light reflex

minimal AV crossing changes

II

obvious broadening of the arteriolar light reflex

moderate AV crossing changes

III

copper-wire arterioles

marked AV crossing changes

IV

silver-wire arterioles

severe AV crossing changes

B

A

C

Figure 23. Arteriolar Reflex Changes: A – normal; B – broad light reflex “copper wiring” C – “silver wire”

A

B

Figure 24. Blood Vessel Changes: A – normal; B – Tapering (green arrows) ; C – Banking (green arrow)

Other ocular problems associated with hypertension include the following: (1) Retinal vaso-occlusive diseases a. central retinal vein occlusion (CRVO) b. central retinal artery occlusion (CRAO) c. branch retinal vein occlusion (BRVO) d. branch retinal artery occlusion (BRAO) (2) subconjunctival hemorrhage (3) ptosis (4) EOM paresis (5) cortical blindness

197

C

OCULAR MANIFESTATIONS OF SYSTEMIC DISEASES / 198

A

B

Figure 25. Vaso-occlusive Diseases: A – CRVO; B - CRAO

The primary management of ocular complications associated with hypertension would be adequate blood pressure control. For complications such us vascular occlusions, close follow-up by an ophthalmologist is recommended. Laser treatment is performed whenever indicated.

HUMAN IMMUNE DEFICIENCY VIRUS/ACQUIRED IMMUNE DEFICIENCY SYNDROME

Infection with the Human immunodeficiency virus ( HIV) causes immune system suppression of patient which subsequently allows opportunistic infections and neoplastic conditions to affect the eye. Individuals at risk of acquiring this infection include the following: ƒ People receiving transfusions ƒ IV drug users ƒ Healthy sex partners of infected patients ƒ Babies born of infected mothers ƒ Hemophiliacs ƒ Prostitutes, sex workers ƒ Homosexuals and Bisexuals ƒ Health workers Ocular manifestations include the following: ƒ dry eye ƒ retinal microangiopathy often manifested as cotton-wool spots ƒ opportunistic infections commonly presenting as Cytomegalovirus (CMV) retinitis ƒ tumors like Kaposi’s sarcoma of the lids or conjunctiva ƒ neuro-ophthalmologic lesions Cottonwool spots are the most common finding in these patients. It is found in 100% of HIV infected patients. Cotton Wool Spots may be associated with retinal hemorrhages and microaneurysms. Patients are usually asymptomatic and these lesions may disappear spontaneously. It is proposed that this occurs as a result immune complex deposition and/or HIV infection of retinal vascular endothelium. Cytomegalovirus (CMV) retinitis affects 40% of patients with AIDS. Its presence signifies severe systemic involvement. This condition may however occur in other immunodeficiency states.

198

OCULAR MANIFESTATIONS OF SYSTEMIC DISEASES / 199

Figure 26. Cottonwool spot in a patient with HIV infection.

Figure 27. CMV retinitis in a patient with HIV infection

THYROID EYE DISEASE (TED)

This autoimmune eye condition is known by a variety of names such as Graves' Ophthalmopathy/Orbitopathy and Thyroid-associated Orbitopathy (TAO) and dysthyroid ophthalmopathy. While most patients with this condition will present with hyperthyroidism and the eye disease simultaneously, the ocular involvement may precede or lag after the endocrine manifestations. In some instances, the eye condition may even present in the absence of any evidence of thyroid dysfunction. Symptoms associated with thyroid eye disease often present between 20 to 45 years of age. It affects women nine times more often than men. Ocular involvement may be unilateral or bilateral. Symptoms may include excessive redness, tearing, itching, pressure, puffiness and pain. Unilateral or bilateral lid retraction is the most common sign of TED. As a result of lid retraction, the upper lid margin is at or above the superior limbus. This may be due to sympathetic overdrive affecting Mullers muscle, fibrosis of the muscles elevating the lid, or hypotropia. Other common signs of TED are lid lag on downgaze, proptosis, injection over the recti muscle insertions, esotropia and hypotropia. TED is the most common cause of unilateral or bilateral proptosis in adults. Proptosis or exophthalmos can be measured with an exophthalmometer. Proptosis is due to inflammation of the extraocular muscles and orbital fat, causing anterior protrusion of the globe (and sometimes optic nerve compression) in the relatively confined bony space of the orbit. Patients with TED can still develop significant optic nerve compression without marked proptosis. Although exophthalmometry is useful, proptosis can be determined by viewing the patient overhead. Digital palpation through the patient’s closed eyelids can be used to estimate orbital compliance. The most common clinical manifestations of TED are lid retraction (Dalrymple’s sign), lid lag (Von Graefe’s sign) and retraction together with exophthalmos referred to as “Thyroid Stare” (Kocher’s sign). In some instances the extraocular muscles may enlarge resulting in diplopia due to limited motility of the eye. Thyroid eye disease commonly affects the medial rectus and inferior rectus muscles accounting for the appearance of esotropia and hypotropia. Initially the muscles are swollen and there may be injection over the recti insertions. Later the muscles become fibrotic.

199

OCULAR MANIFESTATIONS OF SYSTEMIC DISEASES / 200

Table 3. Classification of thyroid orbitopathy Class

Description

Signs or symptoms

0

No signs or symptoms

1

Only signs

Upper lid retraction, stare, lid lag Proptosis

2

Soft tissue involvement

Epiphora, conjunctival and lid oedema Orbital fat extrusion Swollen muscles

3

Proptosis

Proptosis moderate or severe

4

Extraocular muscle involvement

Gaze restriction, diplopia

5

Corneal involvement

Staining with fluorescein Ulceration, infiltrate, perforation

6

Sight loss – optic nerve involved

Colour vision loss Visual acuity decrease Visual field loss Disc swelling or pallor

Figure 28. Only signs = Class 1

Figure 29. Soft tissue involvement = Class 2

Figure 31. Extraocular Muscle Involvement = Class 4

Figure 32. Corneal Involvement = Class 5

Figure 33. Sight Loss = Class 6

200

Figure 30. Proptosis = Class 3

OCULAR MANIFESTATIONS OF SYSTEMIC DISEASES / 201

Less serious complications of TED are exposure keratitis, tearing and foreign body sensation and lid and conjunctival edema / chemosis. More serious complications include diplopia, ophthalmoplegia and loss of vision. Loss of vision usually results from compression of the optic nerve by swollen tissues surrounding the eye. Urgent treatment is required otherwise visual loss can be permanent. Other complications include glaucoma and exposure of the front surface of the eye resulting from the inability to completely close the eyelids. Table 3 presents the Werner classification (NOSPECS classification) which is often used by clinicians to follow the course of the condition. There is no single laboratory examination that will confirm the presence of thyroid eye disease. As was mentioned earlier, the condition is not related to thyroid hormone levels and may present even when the patient is in a euthyroid state. Extraocular muscle involvement may be documented using radiologic examinations which will reveal extraocular muscle enlargement. It is common for thyroid eye disease to fluctuate within the first few years of the disease. Beyond this time, the disease usually attains a stable condition. The disease may however continue to progress even the patient’s thyroid status is controlled. Treatment of Congestive Phase include local therapy with tear substitutes and lubricants to help to protect the surface of the eye from drying. Head elevation particularly while sleeping reduces swelling around the eyes. Double vision can be troublesome if it affects straightforward and down-looking positions. Special lenses called prisms may be used to relieve this. In some instances, patients may also benefit from strabismus surgery. Steroids are used in selected cases. Since steroid use may cause a number of undesirable side effects with chronic use, they are only given as a temporary measure. Radiation is also utilized to reduce swelling of periocular tissue and subsequently decompressing the optic nerve. When vision is threatened, early lid or orbital decompression surgery may be necessary. Otherwise, surgery is usually reserved for stable, inactive or Cicatricial Phase of the disease with the following complications: abnormal staring appearance; severe protrusion of the eyes; disturbing double vision not relieved by prism glasses and drooping or sagging of tissues around the eyes.

TUBERCULOSIS

Ocular involvement in tuberculosis can be caused by either direct invasion of organism, or as a result of hypersensitivity reaction to tuberculoprotein. Ocular involvement is more common in patients with miliary tuberculosis, although, it may also be seen in patients with no evidence of pulmonary disease. The most common manifestation of TB in the eye is granulomatous uveitis (anterior and/or posterior). Other manifestations are: ƒ Phlyctenulosis ƒ Vitritis ƒ Retinal periphlebitis ƒ Choroidal tubercles ƒ Panuveitis

201

OCULAR MANIFESTATIONS OF SYSTEMIC DISEASES / 202

The inflammatory condition associated with TB is usually treated with corticosteroids. It is however, imperative that patients be treated with anti-Koch’s medication prior to institution of the anti-inflammatory regimen to avoid exacerbation of the systemic infection.

A

B

C

Figure 34. Tuberculous Granulomatous Uveitis: (A) – keratic precipitates (B) Bussacca nodules (C) – Koeppe nodules

A

B

C

Figure 35. Other Manifestations of Ocular TB: (A)-phlyctenulosis (B)-choroidal tubercle (C)-retinal periphlebitis

SUMMARY

The importance of being able to recognize ocular signs, symptoms and complications of many systemic diseases is a vital part of good medical practice. It is therefore necessary for the primary care physician to be able to perform a thorough eye examination, particularly that of the fundus. Early diagnosis of the conditions earlier discussed, in particular diabetic retinopathy is crucial in ultimate outcome of treatment. This instructional material is by no means complete. Only conditions that are more commonly encountered in local practice have been emphasized. Additional reading is recommended to supplement the information provided. REFERENCES 1. Kanski JJ: Clinical Ophthalmology: A Systematic Approach, ed 3. Oxford: Butterworth-Heinemann, 1994. 2. Federman JL, Gouras P, Schubert H, Madison Slusher M, Vrabec TR: Retina and Vitreous, in Podos SM, Yanoff M (eds): Textbook of Ophthalmology, Vol 9. London: Mosby, 1994. 3. Schachat AP, Murphy RP (eds): Medical Retina, in Ryan, SJ (ed): Retina, ed 2. St Louis: Mosby, 1994. 4. Yanuzzi LA, Guyer, DR, Green, WR: The Retina Atlas. St Louis: Mosby, 1995.

202

OCULAR MANIFESTATIONS OF SYSTEMIC DISEASES / 203

5. Tang RA, Coleman AL, Wilkins JK, Brown J, Newman SA, Skootsky S, Whitcup SM: Ocular Manifestations of Systemic Disease: A Slide-Script Program. San Francisco: American Academy of Ophthalmology, 1996. EVALUATION Picture 1. Identify the encircled lesions indicated by arrows. Give a diagnosis.

1

Picture 2. Using Scheie Classification, give the stage of hypertensive retinopathy (give staging for both arteriosclerosis and hypertensive changes)

2

Picture 3. In what disease condition is this associated? What are the abnormal findings?

Picture 4. . In what disease condition is this associated? What are the abnormal findings?

203

OCULAR MANIFESTATIONS OF SYSTEMIC DISEASES / 204

SELF-TEST 1. A 45-year-old male consulted a private MD for a bulging left eye. He was advised to undergo an MRI. Because of the expense involved, he decided to seek a second opinion from you. You will: A. order a CT scan instead B. go on with the MRI C. order a T3 & T4 determination D. d. order an HIV test 2. In the management of diabetic retinopathy, the following will apply: A. Type 1 diabetics of > 2 years duration should be referred to an ophthalmologist B. A 26 year old diabetic should be referred to an ophthalmologist at the time of diagnosis C. Panretinal photocoagulation should be considered in background retinopathy D. Type 2 diabetics of > 5 years duration should be referred to an ophthalmologist 3. A 68-year-old male has had a history of BPs ranging from 170-200/100-120. On ophthalmoscopy one would commonly expect to find: A. arteriolar constriction B. copper-wire venule C. dilated venule D. cotton wool spots 4. A 50-yr old female who has had NIDDM for the last 10 years consults for blurred vision. You would: A. look for vitreous hemorrhage B. order ultrasound examination to rule out a retinal detachment C. examine for corneal defects with a biomicroscope D. concentrate your examination on the macular area 5. Grave’s ophthalmopathy is the most common cause of proptosis in the adult. In the progression of this disease, the following may be encountered: A. “thyroid stare” resulting from the combination of lid lag and lid retraction B. a “frozen” eyeball C. sudden, painless loss of vision D. consistently abnormal thyroid hormone levels 6. Mild nonproliferative diabetic retinopathy is best managed with: A. panretinal photocoagulation and strict blood sugar control B. vitreoretinal surgery and endolaser treatment C. good blood sugar control and regular fundus fluorescein angiography D. pulse insulin therapy and regular eye exams 7. On routine ophthalmic examination, an asymptomatic 28 year old GRO was found to have a small, solitary whitish lesion with blurred margins on the left fundus. You will order for: A. T3 & T4 determination B. fasting blood sugar C. serum cholesterol D. HIV test

204

OCULAR MANIFESTATIONS OF SYSTEMIC DISEASES / 205

8. Non-proliferative diabetic retinopathy is characterized by the following EXCEPT: A. microaneurysms B. NVE and NVD C. dot hemorrhages D. hard exudates 9. Routine eye examination of a 57 yr. old hypertensive male patient revealed exudates and hemorrhages all over the fundus with distinct disc borders and an AV ratio was 1:3. The patient has: A. Hypertensive retinopathy grade I B. Hypertensive retinopathy grade II C. Hypertensive retinopathy grade III D. Hypertensive retinopathy grade IV 10. In order to prevent diabetic retinopathy from becoming the world’s leading cause of blindness, the following should be done: A. regular and proper eye examination for individuals at risk B. aggressive oral hypoglycemic therapy for type II diabetics C. aggressive insulin therapy for type I diabetics D. proper diet and exercise for the elderly population ANSWERS TO SELF-TEST 1. 2. 3. 4. 5. 6. 7. 8. 9. 10.

C B A D B C D B C A

205

EYELID MALPOSITIONS / 206

Eyelid Malpositions Franklin P. Kleiner, M.D. INTRODUCTION This self-instruction material is designed to help the student learn the basic concepts of eyelid malpositions, their classification, pathophysiology, diagnosis and management. The eyelids are important accessory structures that give support and complement the proper functioning of the eye. Every student of ophthalmology should be able to recognize eyelid malpositions, and know theoretically their pathophysiology and management. OBJECTIVES Upon completion of this unit of instruction, the student should be able to recognize the 4 different eyelid malpositions and know the basic concepts of diagnosis and management. Specifically they should be able to: 1. 2. 3. 4. 5. 6.

define the 4 lid malpositions: Ectropion, Entropion, Ptosis and Lid Retraction. classify of the different types of Ectropion, Entropion, Ptosis and Lid retraction discuss the pathophysiology of the different types of Ectropion, Entropion, Ptosis and Lid retraction. recognize the different eyelid malpositions perform diagnostic maneuvers to evaluate the 4 lid malpositions discuss the theoretical management of each of the 4 lid malpositions. PREREQUISITE KNOWLEDGE:

Students should have a working knowledge of the anatomy of the eye and adnexae, particularly eyelid anatomy. They should also know the basic history taking and physical examination. And basic eye examination. INTENDED USERS This material is developed for the use of medical students level IV, but should b useful to any student of ophthalmology. The detailed discussion on the management of the different eyelid disorders maybe more appropriate for the level VI and VIII students CONTENT The eyelids are important accessory structures that give support and complement the proper functioning of the eye. They provide protection, support and lubrication for the eye to enhance its proper functioning, particularly vision. Any abnormality or irregularity of the position of the eyelids can have drastic effects on the eye and its function. There are 4 basic eyelid malpositions: Ectropion, Entropion, Blepharoptosis or Ptosis, and Eyelid Retraction. These will be discussed one by one. ECTROPION: Definition: ECTROPION is the outward turning of the eyelid margin.

206

EYELID MALPOSITIONS / 207

Recognition: This can be easily recognized in patients due to the everted position of the lid margin, with a space or gap between the palpebral conjunctiva and the globe, in severe cases with visualization of the palpebral conjunctiva. If the ectropion is carried medially, one may even visualize the punctum, which is normally not visible, since its normal position is in apposition with the globe. Following is its classification or possible causes: 1. Senile Involutional Ectropion 2. Paralytic Ectropion 3. Cicatricial Ectropion 4. Congenital Ectropion 5. Mechanical Ectropion SENILE INVOLUTIONAL ECTROPION As we age, our tissues tend to stretch out and become lax. Patients who constantly rub their eyes can hasten this process. The eyelid tends to stretch out and lengthen, especially at the canthal tendons. In the eyelid, excessive eyelid laxity can result in ectropion. One can test for eyelid laxity by doing the snap back test. Pull the eyelids downward toward the orbital rim, then let go. Normally, the eyelids will snap back to their original position, even without the patient blinking. However, if there is laxity in the patient’s eyelids, they will not snap back, but will go slowly back up. Sometimes they go back to their original position. But if very lax, they may stay ectropic. But once the patient blinks, the eyelid will go back again to its original position. Another test is the eyelid distraction test wherein the eyelid is pulled away from the globe to see how far it can stretch out. Stretching of 6mm or more confirms the presence of laxity. Signs and symptoms are tearing due to the interrupted flow of tears from the lateral to the medial canthus, interruption of the tear meniscus and lacrimal pump, and at times even due to an ectropic punctum (see medial ectropion). In more severe cases, there can be reflex tearing due to irritation from dry eye or exposure keratitis, with superficial punctate keratitis and corneal opacification. Irritation and dryness of the conjunctiva is common.

Senile Involutional Ectropion

207

EYELID MALPOSITIONS / 208

Treatment: Senile involutional ectropion is best treated by a horizontal eyelid tightening procedure. The lateral tarsal strip operation is the most popular, which involves tightening of the eyelid horizontally by attaching the lateral most portion of the tarsus to the lateral orbital rim. the lateral tarsal strip operation. There are two advantages of this operation over other lid tightening or lid shortening procedures. One is that it corrects the laxity at the site of the pathology which is the lateral canthal tendon. Other lid shortening procedures such as a wedge resection or Kuhnt Szymanowski shorten the tarsus of the lid, leaving the laxity of the lateral canthal tendon uncorrected. Thus a phimosis or shortening of the length of the palpebral fissure may be induced. The second advantage is that the scar is tucked away in the lateral canthus. In a wedge resection, the scar may be more obvious, and there may even be lid notching.

Senile Involutional Ectropion after Tarsal Strip

MEDIAL ECTROPION In the normal eyelid, the punctum is not visible. This is because it is turned inward facing the lacrimal lake. If a patient’s punctum is visible in a normal resting position, this means that it is in an ectropic position. Surgical treatment. A medial spindle excision operation is most suitable for this condition. This procedure involves the excision of a medial spindle of papebral conjunctiva just below the punctum, with the placement of inverting sutures to restore the punctum to its normal position. PARALYTIC ECTROPION When the orbicularis oculi becomes paralyzed in affectations of the 7th nerve, there is loss of eyelid tone. In temporary cases such as Bell’s Palsy, this is reversible. But in prolonged cases, such as a stroke, or 7th nerve palsy or paresis, atrophy of the muscle can occur resulting in more permanent laxity of the eyelid. Conservative treatment: In temporary conditions, lubrication with artificial tears, gels, ointments, and in worse cases, goggles or moist chambers may be sufficient. Punctal plugs may be inserted as well. Patient’s eyelids may need to be tape at bed time. Surgical treatment: But if exposure will be present for a longer period of time, tarsorrhaphy may be done (suturing of the eyelids together to effect closure). Or a gold weight may be implanted in the upper lid, in conjunction with a

208

EYELID MALPOSITIONS / 209

tarsal strip and a small 3-4 mm lateral tarsorrhapy. Implantation of gold weights in the upper lid allows the eyelid to close due to the weight of gravity. CICATRICIAL ECTROPION In the normal eyelid, there is a balance between the anterior lamella and the posterior lamella. If there is shortening of the anterior lamella, cicatrical ectropion can occur. Some conditions that cause this are: scarring of the eyelid skin due to trauma, healed lacerations, thermal burns, chemical burns, infections or abscesses that healed, longstanding chronic infections of the lid, etc. Recognition of this condition is done by checking on the tightness of the anterior lamella, checking while the patient is in upgaze or with the mouth open. Management involves lengthening of the existing anterior lamella. For mild conditions, lengthening of the anterior lamella by scar excision and z-plasty or multiple z-plasties may be performed. In moderate to severe cases of cicatricial ectropion, skin grafts may be necessary. Best donor sites for skin grafts to the upper lid are from the opposite upper lid. Best donor site for skin grafts to the lower lids are from the post auricular area. Other areas are the preauricular area, supraclavicular area and medial arm. (in that order of preference).

Cicatricial Ectropion Preop

After skin graft

CONGENITAL ECTROPION – this is a form of cicatricial ectropion, usually due to shortage of anterior lamella since birth, usually in conjunction with some other congenital conditions such as blepharophimosis. MECHANICAL ECTROPION – This is a form of ectropion caused by a mass or lump in the eyelids, weighing down on the eyelid and causing it to become ectropic. Management is excision of the mass, if possible ENTROPION Entropion is the inward turning of the eyelid margin. Recognition: The eyelid margin is rotated inwards, with loss of visualization of the lid margin, and the lashes are turned in, rubbing against the globe. There is frequent irritation, redness and possibly discharge of the eye. Patient complains of constant irritation and foreign body sensation caused by rubbing of the lashes against the cornea. In severe cases, corneal abrasion and scarring may occur, causing decrease in visual acuity.

209

EYELID MALPOSITIONS / 210

Extreme complications are infection of the cornea, resulting in corneal ulcer with possible rupture of the eye. This is why entropion is considered a semi-emergency situation.

Following is the classification according to different causes: 1. 2. 3. 4.

Cicatricial Entropion Senile involutional Entropion Congenital Entropion Acute spastic Entropion

CICATRICIAL ENTROPION Cicatricial entropion is caused by scarring of the posterior lamella: conjunctiva and tarsus. This scarring causes a deformity of the tarsus, causing it to rotate inwards permanently. Frequent causes of scarring are chronic infection of the eyelids, trauma, in the anophthalmic socket - chronic irritation from artificial eye use, inflammatory conditions of the conjunctiva such as pemphigus, pemphigoid or Stevens Johnson Syndrome, systemic diseases such as leprosy.. This condition must also be differentiated from trichiasis, which is the inward turning of lashes, to rub on the cornea. This may or may not be in conjunction with a cicatricial entropion, which involves actual rotation of the lid margin inwards. Another condition known as distichiasis may be present, which is the growth of a second row of lashes, usually from the area of the meibomian gland orifices. SENILE INVOLUTIONAL ENTROPION Senile involutional entropion is caused by aging and laxity of tissues. There are 4 factors that come into play causing the inward turning of the eyelid margin: 1. lid laxity 2. preseptal orbicularis over ridng the pretarsal. 3. detachment or dehiscence of the lower lid retractors. 4. involutional enophthalmos. The detachment or dehiscence of the lower lid retractors contributes to the instability of the lower border of the tarsus, and when combined with lid laxity and overriding of the orbicularis muscle, this causes the eyelid to flip inwards. Involutional enophthalmos is not always present, but may play a role if present. One can differentiate the two by the following test: Press on the lower lid to return it to its original non entropic position. If on releasing it, it jumps into an entropic position right away, then the entropion is cicatricial in nature. If, however, after returning it to its normal non entropic resting position, the eyelid stays in that position, and only turns inward when the patient blinks or closes his eyes, then it is senile involutional entropion.

210

EYELID MALPOSITIONS / 211

One should also examine the lid margin under slit lamp for conjunctivalization of the lid margin. Its presence signify chronicity of the condition. One may even detect the mildest of entropions with this technique. The normal mucocutaneous junction along the lid margin is posterior to the meibomian glands. If the mucocutaneous junction looks like it has advanced to the level of the meibomian glands, or even more anteriorly, to the gray line, or the lash line, this implies that posterior portion of the lid margin is in apposition with the globe, making the posterior portion of the lid margin conjunctivalized. This is a telltale sign of chronic entropion. Management of Cicatricial Entropion and trichiasis If trichiasis is present in a short segmental area, lashes may be removed selectively by epilation (plucking of lashes), however the condition will return when the lashes grow back in about two weeks. Destruction of follicles of the lashes for trichiasis may also be done. Electrocautery or radiofrequency may be used for this purpose. Others have used cryotherapy to destroy the follicles by freezing. These may require repeated procedures to attain a satisfactory result. Cicatricial Entropion, being caused by a permanent deformity of the tarsus, is best managed by surgical correction. If the area of entropion is very short or segmental, a wedge resection of the eyelid involving that portion of lashes may be done. In mild to moderate cases of cicatricial entropion, a lid margin rotation procedure such as a tarsotomy may be done, to rotate the lid margin outwards. In severe cases of deformity of the posterior lamella or tarsus, a grafting procedure must be done to replace the severely deformed tarsus. A number of tissues may be used, including conjunctiva, buccal mucosa, or more firm tissue such as sclera, hard palate mucosa or ear cartilage. Management of Involutional Entropion Conservative management: One may use lubricants and ointments to prevent rubbing of the lashes against the cornea. Taping of the eyelid in an everted position may help relieve the patient, but these are only temporary measures which cannot definitively solve the problem. Entropion, being a mechanical problem of the lids, will usually require a surgical solution. Surgical management: There are numerous procedures for involutional entropion in many textbooks. We will only include a few more common procedures here. Quickert Sutures are full thickness everting sutures that can be placed along the entire length of the eyelid. This procedure can correct overriding of the orbicularis and dehiscence of the lower lid retractors. It however does not address lid laxity. It is a very simple procedure that can be done in the minor operating room. However there is a high recurrence rate. It is usually reserved for patients with medical problems or on blood thinning medication that cannot undergo a definitive surgical procedure. Wies procedure – This is a full thickness blepharotomy with everting sutures which adressses overriding of the orbicularis and dehiscence of the lower lid retractors, but it does not address lid laxity. Jones procedure – This involves identification, isolation and tightening of the lower lid retractors through a subciliary incision. The subciliary incision will prevent overriding of the orbicularis. It does not address lid laxity.

211

EYELID MALPOSITIONS / 212

Tarsal strip with retractor reinsertion – A tarsal strip operation is performed to remedy lid laxity. In addition, lid retractors are identified and reattached to tarsus through a subciliary incision. This addresses 3 factors: lid laxity, lid retractor dehishcence, and orbicularis overriding, and therefore is the preferred procedure for this condition. The success rate for this procedure is quite high, and recurrence rate low. CONGENITAL ENTROPION Congenital entropion is present in newborn children and may be present up to early childhood. It is more frequently found in the lower lid, but may be present in the upper lid as well. It is usually caused by a prominent or hypertrophic roll of orbicularis and skin or epicanthal fold which rotates the eyelashes inwards toward the eye. If lash touch to the eyes is mild, lubricants and periodic follow up will suffice. In due time as the face of the child matures, the nasal bridge grows higher with age, and the medial eyelid skin retracts and the entropion resolves spontaneously. In moderate to severe cases, or those that do not resolve in early childhood and become persistent, surgery involving removal of the excess skin and hypertrophic muscle must be done, concomitantly with everting eyelid sutures. In some cases, an epicanthal fold in the lower lid (epicanthus inversus) may be present. Remedy for this situation may also involve a Y-V medial canthoplasty. SPASTIC

ENTROPION

This type of entropion may occur due to constant squeezing of the eyelids and spasm of the orbicularis such as in irritative conditions of the eye with constant foreign body sensation, causing the patient to squeeze his eyelids frequently, or in conditions such as blepharospasm. Relief of the spasm by relieving the irritative condition will usually resolve this type of entropion. However in chronic cases such as blepharospasm, botulinum toxin may be used. PTOSIS (BLEPHAROPTOSIS) Definition – Blepharoptosis is drooping of the eyelids below the normal position. This is commonly referred to as just plain Ptosis (drooping). The normal position of the upper eyelid is 2 millimeters below the superior limbus. If the eyelid position is any lower than this, ptosis is present. A short review of the anatomy and physiology of the eyelid retractors and protractors and other pertinent eyelid structures: The levator muscle originates from the annulus of Zinn, slightly above the superior rectus muscle. It is in close association with the superior rectus muscle almost throughout its length. It is held up and supported by the Whitnall’s ligament, and within the eyelid its muscle fibers teriminate into a broad tendinous sheet, the levator aponeurosis, which inserts into the superior portion of the anterior surface of the tarsus. Both levator muscle and superior rectus muscle are innervated by the superior branch of the 3rd cranial nerve, and its function is to open the eyelid. Also, closely associated with the levator muscle is the Mullers muscle. This is a lid retractor innervated by sympathetic nerves. Its function is also to open the eyelid. This muscle originates from the underside of the belly of the levator muscle and inserts on the superior tarsal border. It lies between the levator muscle above it, and the palpebral conjunctiva below it. It can be recognized by the vertical orientation of its muscle

212

EYELID MALPOSITIONS / 213

fibers, and the superior palpebral arcade of blood vessels which travels on top of it, along the superior tarsal border. The protractor of the eyelid is the orbicularis muscle or orbicularis oculi muscle, which is responsible for eyelid closure. It is the antagonist of the levator muscle. It is a circular, sphincter muscle which lies just beneath the skin of the eyelids. Origin is from the medical canthal tendon. It has three parts, the pretarsal, preseptal and preorbital portions. The pretarsal forms the innermost circle, and overlies the tarsus of the upper and lower lids. The preseptal forms the middle circle and overlies the septum. The outermost circle is the preiorbital portion, which overlies the orbital rim and bone. The orbicularis muscle is innervated by the 7th cranial nerve, and is responsible for eyelid closure. The tarsal and preseptal portions are responsible for involuntary blinking and reflex closure. The periorbital portion is responsible for forceful and voluntary closure. Classification of Ptosis I. According to Etiology 1. Neurogenic – caused by a paralysis or paresis of the third cranial nerve. This may be isolated to the levator, or in the case of complete 3rd nerve palsy, extraocular muscle involvement will be manifested, with the eye usually taking a downward and outward position. If both levator muscle and superior rectus muscle are affected by a paralysis or paresis of the superior branch of the 3rd cranial nerve, we call this double elevator palsy. 2. Myogenic –caused by atrophy or weakening of the muscle. When the muscle does not develop properly or muscle fibers die, they become replaced with fibrous or fatty tissue. This is commonly seen in congenital ptosis. 3. Aponeurotic- caused by a detachment of the aponeurosis from the tarsus. Levator function may be good or normal. This comprises a vast majority of acquired ptosis. 4. Traumatic – caused by trauma 5. Mechanical – caused by the weighing down of the upper lid by a mass or growth 6. Pseudoptosis – hypotropia . When a hypotropia is present, the eyelid may follow the eye that is turned downward, mimicking a ptosis. Other cases of pseudoptosis may occur in the presence of dermatochalasis, where overhanging skin from the upper eyelid may simulate a ptosis.Enophthalmos may also simulate a ptosis. 7. Myasthenia gravis – gradual weakening of muscles due to malfunction of the neuromuscular junction, involving fatigue in the production and sensitivity of the neurotransmitter, acetylcholine. This is a medical condition that may involve weakness of other muscles, such as extraocular muscles manifesting as diplopia, or muscles of the extremities. It is characterized by a variable type of ptosis. Eyelid height may be normal on waking in the morning but may begin to droop as the day progresses, with full blown ptosis and fatigue in the afternoons. Confirmation is by tensilon test, which restores function to the ptotic lid. Medical management with mesthinon is the indicated treatment. Surgical intervention may be contemplated, but usually only as a last resort, after medical treatment has been tried. II. According to Time of Onset 1. Congenital –This is a ptosis that is present at birth. More commonly this type of ptosis is associated with a weak muscle and no lid crease. As the child gets older, the nonfunctional or atrophic muscles become replaced with fibrous or fatty tissue. The eyelid does not open well due to lack of muscle fibers, and it does not close well due to the rigidity of the fibrous tissue. This can be easily identified in downgaze, when the patient’s ptotic eyelid is paradoxically higher than the opposite eyelid. 2. Acquired – This is a ptosis that is acquired after birth. This type of ptosis is more commonly associated with a strong levator muscle whose aponeurosis has detached from its tarsal attachment. This is commonly manifested by a comparatively higher lid crease than the opposite side. This can be identified in downgaze. When asking the patient to look downwards, the ptotic eyelid is usually lower than the opposite side, due to the levator detachment from the tarsus.

213

EYELID MALPOSITIONS / 214

Congenital Ptosis

Acquired Ptosis

III. According to severity or degree 1. Mild – 2 mm, MRD+1 2. Moderate – 3 mm MRD 0 3. Severe – 4 mm MRD -1 Measuring the severity or degree of ptosis: Palpebral fissure technique: One should have ready a penlight and a mm ruler. With the patient looking straight ahead, measure the palpebral fissure height of each side, record and compare. Normal palpebral fissure height is 10 mm. In cases of unilateral ptosis, a difference of 2 mm between the 2 eyelids means a 2 mm ptosis, a difference of 3 mm means a 3 mm ptosis. A difference of 4mm means a 4 mm ptosis. In bilateral ptosis cases, comparing the two eyelids is not practical. In these cases, one can judge the level of ptosis by observing the level of the eyelid. Since the normal position of the eyelid is 2 mm below the limbus, a 2 mm ptosis would be 4mm below the limbus, a 3 mm ptosis would be 5 mm below the limbus, and a 4mm ptosis 6 mm below the limbus. This technique is not very useful in patients where the lower eyelid is retracted downwards, or asymmetric on either side. This may result in misinterpretations.

Measuring Eyelid Height

214

EYELID MALPOSITIONS / 215

MRD or marginal reflex distance method: The marginal reflex distance is defined as the distance between the lid margin and the central corneal reflex. This is taken by shining a light onto both corneas, and measuring the distance between the upper lid margin and the corneal light reflex. The measurement is expressed in +mm if the eyelid margin is above the reflex or –mm if the lid margin is below the light reflex. If one would assume that the normal vertical corneal diameter is 10mm, then the corneal light reflex would be in the center, thus 5 mm from the superior limbus. A normal eyelid situated 1-2 mm below the superior limbus would have an MRD of +3 to +4. An eyelid with a 2 mm or mild ptosis would have an MRD of +1 or 1 mm above the reflex. An eyelid with a 3 mm or moderate ptosis would have an MRD of 0 or with the lid margin right at the level of the reflex. An eyelid with a 4 mm or severe ptosis would have an MRD of -1, or 1 mm below the level of the light reflex. The MRD technique may be subject to misinterpretation in cases of hypertropia or hypotropia, and extra caution should be exerted in interpretation of these cases. IV. According to strength of levator function 1. Poor 0-4 mm 2. Fair 5-7 mm 3. Good 8-10 mm 4. Very Good 11-12 mm 5. Excellent 13-15 mm Technique of measuring levator function: One should have ready a mm ruler.In measuring the levator function, the patient should be seated in front of the examiner looking straight ahead. The examiner lays one hand on the patients forehead, using his thumb to neutralize the motion of the eyebrow. Patient is instructed to move only the eyes and not the head. Examiner then asks the patient to look down as far as he can. The position of the eyelid is noted and coincided with the 0 of the ruler. Patient is then asked to look up as high as he can. The maximum excursion of the eyelid is noted and measured. Examination can be repeated several times to determine accuracy of the measurement. Levator function is then similarly measured on the opposite eyelid. One may use the above table to interpret levator function.

Measuring Levator Function

215

EYELID MALPOSITIONS / 216

A complete Eye Examination for ptosis should start with the basic eye examination, including gross exam, visual acuity, tonometry, EOM, and funduscopy.In addition to the routine basic eye examination, one should check the following: 1. Test for degree of ptosis – measure palpebral fissures (previously described) -Marginal Reflex Distance (previously described) 2. Test for levator function (previously described) Other tests: 1. Bells phenomenon – The Bells phenomenon is the reflex action of the eyes to turn upwards when the eyes are closed as in sleeping, or during actual sleeping. This protects the eyelids in the sleeping position and should be present in all patients where ptosis surgery is contemplated. If not present, one must think twice or thrice before proceeding with a ptosis procedure that will raise the eyelid, and cause exposure. This will surely be aggravated if performed in a patient with poor bells phenomenon. This can be taken by asking the patient to close his eyes as if sleeping, then gently lift the upper lid with your fingers and note the position of the eyeball. Most patients will have the eye in an upward position. This is the bells phenomenon.

2. Orbicularis tone – this needs to be checked in a patient where the eyelids will be raised, to check if the patient has enough muscle tone to cause adequate closure of the eyelids. This is done by asking the patient to squeeze his eyelids forcibly. Now using the thumb and forefinger, try to forcibly open the patient’s eyes. One should feel the forceful closure of the eyelid against the fingers, and one can gauge the strength of the orbicularis muscle. 3. Tear production – Again in a procedure that will raise the eyelids, we must have adequate tear production to ensure proper lubrication of the corneal surface. One would not want to expose a patient with dry eye to a ptosis operation which would cause further exposure and drying of the cornea. This can be done by performing the Schirmer’s test. Put the folded end of the Schirmer’s strip in each lower eyelid, with the strip hangin out, and observe for 5 minutes. The strip with the corner cut off is traditionally placed in the Right eye. The eyes of the patient may remain open, or closed, whichever is more comfortable. After 5 minutes, one must check the amount of tears absorbed by the Schirmer’s strip, and measure using a mm ruler. Dry eye is suspected if the result is less than 2mm of tears in repeated examinations. Another test would be the fluorescein dye test. Instill a drop of fluorescein in each eye, or using a fluoresein strip, place the tip in each inferior fornix until some dye has come off onto the conjunctiva. Examine the cornea under the slit lamp with blue light. One may see if there is any superficial punctate keratitis present due to dry eye. 4. Corneal sensation – In a patient undergoing ptosis surgery, corneal sensation is important, because this will serve as the alarm system that will warn the individual that his cornea is in danger of exposure or drying. A

216

EYELID MALPOSITIONS / 217

patient with poor corneal sensation will be predisposed to exaggerated drying and exposure of the cornea, if he should undergo ptosis surgery. This is performed with the patient seated and instructed to look upwards. One may use the twisted tip of a piece of tissue paper, or a cotton wisp to touch the cornea on each side. Ask the patient to grade on a scale of 1 to 10 how much of the cotton wisp did he feel against his eye. SURGERY FOR PTOSIS Repair of the ptosis is accomplished through a number of surgical procedures: 1. Fasanella Servat procedure – this is a tarsomullerectomy approached from the conjunctival side, where the superior border of tarsus and mullers muscle are clamped, excise and sutured. This technique is best used for a narrow spectrum of ptosis patients that fit the following criteria: 1. Mild ptosis – 2 mm ptosis only 2. very good or excellent function ptosis 3. formed lid crease. 2. Levator Resection –This is the most common procedure used for ptosis repair. The levator muscle is identified, isolated and reattached to tarsus in a higher position. This is usually approached through an external lid crease incision. Operation can be performed under local anesthesia without sedation, allowing for adjustment of height and contour intraoperatively. It is best used for the majority of fair to excellent function ptosis of any degree. 3. Fascia Lata Sling – This procedure involves harvesting long thin strips of fascia lata from the thigh, attaching it to tarsus and fashioning a sling attached to the frontalis muscle. The eyelid is suspended by the sling to the desired height, and can be raised by raising the eyebrow. Some amount of lagophthalmos results, and the eyelid generally does not close completely at night. Patient requires lubrication at bedtime to prevent exposure. Procedure is usually done under general anesthesia, and can be performed on one or both eyelids. This procedure is best for patients with poor function ptosis 5 years or older (due to the length of the leg)

Fascia Lata Sling

Levator Resection

Preop

Pre op

Post op

Post OP

4. Suture sling, or similar material – Prolene sutures or Supramid sutures may be used to suspend the eyelid similar to fascia lata. This procedure is best performed in young children or infants or adults who refuse to undergo fascia lata harvesting. Other materials used are goretex and silicone. The procedure is meant to be a temporary one, due to possible reaction to the material, and would require a fascia lata sling in the future.

217

EYELID MALPOSITIONS / 218

LID RETRACTION Definition: Lid retraction is the abnormal displacement of the eyelid toward the orbital rim, resulting in exposure of the sclera above or below the limbus. Upper lid retraction may result in retraction of the eyelid above the superior limbus, while lower lid retraction results in retraction of the eyelid below the inferior limbus. Sclera which is visible below the limbus is termed as scleral show. Etiology: 1. Thyroid Ophthalmopathy – The most common cause of lid retraction is thyroid ophthalmopathy or Graves Ophthalmopathy. In the acute stage, lid retraction may be due to inflammatory changes in the eyelid and eyelid muscles, particularly the eyelid retractors. Stimulation of the Mullers muscle by circulating catecholamines may cause it to contract resulting in lid retraction. Enlargement and hypertrophy of the levator muscle may cause further lid retraction. In the chronic stage, fibrosis and scarring of the muscles and septum may set in, causing permanency of the lid retraction. Lid retraction is frequently accompanied by proptosis, and this may further contribute to the appearance of a retracted lid, however these 2 conditions should be differentiated from each other. A lateral flare may frequently accompany the lid retraction of thyroid ophthalmopathy. 2. Anterior Lamellar shortening – This may be caused by cicatricial changes of the skin, such as in trauma or burns, or excessive removal of skin during blepharoplasty. This can frequently result in lid retraction or scleral show. 3. Post muscle surgery – Patients having undergone recession of the rectus muscles may occasionally develop a lid retraction. 4. Idiopathic- In some cases, no known cause can be identified.

Lid Retraction in Thyroid Ophthalmopathy

Idiopathic Lid Retraction

Measurement of lid retraction – One must have ready a mm ruler. With the patient looking straight forward, the position of the lid may be measured above or below the limbus.

218

EYELID MALPOSITIONS / 219

Treatment of Lid retraction: 1. Conservative treatment: All cases of lid retraction must be worked up for etiology. Thyroid ophthalmopathy must be ruled out by requesting for the pertinent laboratory examinations: T3, T4,TsH. An orbital CT scan, axial and coronal views will help to determine the presence of thyroid induced hypertrophy of muscles. In mild cases of lid retraction, the eye may be at risk of exposure. Topical lubricants may be given. Taping of the lids may be done at night. 2. Surgical Management: In more severe cases, tarsorrhapy may be indicated to protect the globe and prevent exposure. Definitive surgical treatment of the lid retraction should only be undertaken if the patient is euthyroid for at least 6 months. Upper lid retraction may be treated by recession of the levator mullers muscle complex. This can be approached from an external lid crease incision, or from a transconjunctival approach. In severe cases, a spacer graft may be placed to maximize recession of the levator / mullers muscle complex. Spacer grafts that have been more commonly used are sclera and fascia. For lower lid retraction, spacer grafts are commonly used. Sclera, fascia and hard palate mucosa, and ear and nasal cartilage have been used. For retraction caused by anterior lamellar defects or cicatrisation, z plasties, and skin flaps and skin grafts are used to lengthen the anterior lamella. See section on cicatricial ectropion. REFERENCES 1. Kersten RC (editor) et al Basic and Clinical Science Course 2003-2004 Section 7 Orbit Eyelids and Lacrimal System, American Academy of Ophthalmology, San Francisco, USA, 2003 2. Beard C, Ptosis , 3rd edition, CV Mosby, St. Louis, 1981 3. Hatt M, Ophthalmic Plastic and Reconstructive Surgery , Georg Thieme Verlag, Stutttgart, 1986 4. Collin, JRO, A Manual of Systematic Eyelid Surgery, Churchiull Livingstone, 1983 5. McCord Jr. CD, Chapter 5 Surgery of the Eyelids, Duane’s Clinical Ophthalmology Volume 5, TD Duane, ed, Harper and Row, 1984 6. Kersten RC, Kleiner, FP, Kulwin DR Tarsotomy for the treatment of cicatricial entropion with trichiasis, Arch Ophthalmol 1992: 110:714-717. SELF TEST 1. The following are the factors that contribute to involutional entropion EXCEPT: A. Lid laxity B. Detachment of lower lid retractors C. Overriding of pretarsal over preseptal orbicularis D. Enophthalmos 2. Cicatricial entropion is more commonly caused by: A. Scarring of the posterior lamella B. Scarring of the anterior lamella C. Scarring of the lid margin D. Acute blepharoconjunctivitis

219

EYELID MALPOSITIONS / 220

3. Entropion is a semi emergency because: A. It may cause irritation of the eye B. Lashes may cause corneal abrasion C. This may eventually lead to rupture of the globe D. All of the above 4. Senile involutional ectropion is commonly caused by: A. Eyelid inflammation B. canthal tendon laxity C. tarsal shortening D. mass on the eyelid 5. Cicatricial ectropion can be treated by: A. skin grafts B. skin flaps C. Z plasty D. all of the above True or False 6. One should always check for presence of Bells Phenomenon prior to doing ptosis surgery. 7. A patient with poor function ptosis is a good candidate for Fasanella Servat. 8. The reason one must neutralize the eyebrow when checking for levator function is so that it does not affect raising of the eyelid. 9. The marginal reflex distance is an efficient way to test for the levator function. 10. The most common cause of lid retraction is thyroid ophthalmopathy. ANSWERS 1. 2. 3. 4. 5.

C A D B D

6. 7. 8. 9. 10.

T F T F T

220

OCULAR TRAUMA AND EMERGENCIES /221

OCULAR TRAUMA AND EMERGENCIES Ma. Margarita L. Luna, MD Marissa N. Valbuena MD, MHPEd Paolo Ma. Pagkatipunan MD, MHA INTRODUCTION Ocular trauma is a common cause of unilateral blindness in children and young adults. Domestic accidents, violent assaults, fire-cracker injuries, work and sports related injuries and vehicular accidents are among the common circumstances in which trauma occurs. Central retinal artery occlusion and acute angle closure glaucoma are urgent non-traumatic eye conditions which needs prompt diagnosis and management. OBJECTIVES At the completion of this study material, the student should be able to 1. given a trauma patient, be able to extract a relevant medical history, and be able to perform the necessary ocular examination. 2. to recognize the condition which needs prompt referral to the ophthalmologist. 3. discuss the principles of management of the common ocular emergencies. RECOMMENDED PREPARATION Before going to this material, the student must have previous knowledge of anatomy and physiology of the eye and the skills of history taking and ocular examination. INTENDED USERS This SIM is intended to provide an overview to LU IV students. LU V to VII students who have exposure to actual patients during their duty in the eye ward and emergency room will benefit from this module. CONTENT I. EVALUATION OF THE EYE TRAUMA PATIENT A. History 1. Details of the event: place and time, interval from injury (minutes, hours, days), accidental or intentional, work-related 2. Past ocular and medical history 3. Prior treatment 4. Accompanying symptoms: ƒ pain, ƒ loss of vision ƒ diplopia ƒ irritation ƒ foreign body sensation ƒ other organ-system involvement 5. Specific injury: mechanism or type of injury (blunt vs. penetrating), nature of injury ƒ Chemical injury: acid or base ƒ Foreign body injury: metal, organic material, glass ƒ Blunt trauma 221

OCULAR TRAUMA AND EMERGENCIES /222

ƒ ƒ

Thermal burn Animal bites

B. Ocular Examination IMMEDIATE MANAGEMENT OF OCULAR TRAUMA IF THERE IS OBVIOUS RUPTURE OF THE GLOBE, AVOID FURTHER MANIPULATION OF THE GLOBE. Unnecessary manipulation by a non-ophthalmologist while attempting to do a complete ocular examination may cause further damage to the eye. All topical anesthetics, dyes and other medications placed in an injured eye must be sterile. An eye shield should be taped over the eye and systemic broad-spectrum antibiotics started. 1. Approach to the Patient 2. Gross External Examination ƒ performed prior to visual acuity testing if with history of chemical injury; ƒ do copious washing with clean water ƒ Face and lids – observe for lacerations, contusion-hematoma ƒ Conjunctiva – observe for subconjunctival hemorrhage, lacerations 3. Visual acuity 4. Pupils ƒ check for relative afferent pupillary defect ƒ peaked pupil ƒ fixed, dilated ƒ unreactive pupil 5. Extraocular motility ƒ diplopia ƒ inability to look opposite the orbital fracture :suspect trapped muscles 6. Visual fields ƒ confrontation test

222

OCULAR TRAUMA AND EMERGENCIES /223

ƒ

Amsler grid (for macular edema)

7. Anterior segment examination a. Conjunctiva – subconjunctival hemorrhage, lacerations, congestion b. Cornea ƒ Lacerations ƒ Edema

ƒ ƒ

Abrasions Ulcerations

c. Anterior chamber ƒ Hyphema – blood in the anterior chamber ƒ shallow anterior chamber

Note how the slit beams of the cornea and the iris are almost touching each other. The slit beams should be far apart if the anterior chamber is deep.

d. Iris – iridodonesis, iridodialysis e. Lens ƒ Phacodonesis ƒ lens rupture

223

OCULAR TRAUMA AND EMERGENCIES /224

ƒ

f.

cataract

ƒ dislocation Choroid and Sclera – perforation / rupture, avulsion

8. Intraocular pressure – defer finger/palpation tonometry if with penetrating or perforating injury 9. Vitreous and retina Do a dilated funduscopic examination ƒ hemorrhage

ƒ retinal detachment, macular edema, foreign body 10. Optic nerve ƒ congestion, edema, hemorrhage, avulsion Special studies – ERG (Electroretinogram) and VEP (Visual Evoked Potential) II. MOST URGENT OCULAR CONDITONS 1. Chemical burns 2. Central retinal artery occlusion 3. Acute angle closure glaucoma CHEMICAL BURNS - most urgent - usually work-related or with assault, household cleaning agents A. Types of Chemical Burns 1. Alkali – lye (NaOH); caustic potash (KOH); fresh lime [Ca(OH)2] found in plaster, cement, mortar, whitewash; ammonia (NH4) found in household cleaners, fertilizers, magnesium hydroxide (sparklers), and refrigerant

224

OCULAR TRAUMA AND EMERGENCIES /225

ƒ ƒ ƒ ƒ

more extensive corneal damage due to protein denaturation rapid penetration, often less than one minute damage is related more to the degree of alkalinity (pH) rather than tothe actual cation permanent injury is determined by the nature and concentration of the chemical as well as by the time lapsed before irrigation

2. Acid – battery acid, industrial cleaner (H2SO4), laboratory glacial acetic acid or HCl, fruit and vegetable preservatives, bleach, refrigerant (H2SO3), industrial solvents, mineral refining agents, gas alkylation agents, silicone production agents, glass etching agents (HFl) ƒ ƒ ƒ ƒ

less damaging to the cornea because less progressive and less penetrating cause maximum damage within the first few minutes to hours precipitate tissue proteins that rapidly set up barriers against deep penetration by the chemical damage localized to the area of contact* *with the exception of burns from hydrofluoric acid or from acids containing heavy metals, which tend to penetrate the cornea and anterior chamber resulting in intraocular scarring and membrane formation

Anterior segment burns due to tear gas and mace should be managed as alkali burns. Ocular injury from sparklers and flares containing magnesium hydroxide should also be managed as chemical rather than as thermal burns.

B. Treatment 1. Immediate Treatment a. Topical anesthetic immediately instilled b. Lids are held apart, with retractors if necessary c.Immediate lavage with at least 2000 mL of normal saline 0.9% over a minimum period of 1 hour d. Direct pressure on the globe during lavage should be avoided if laceration of the globe is suspected e.Apply topical anesthetic every 20 minutes f. Conjunctival fornices and palpebral conjunctiva should be swept with sterile cotton-tipped applicators to remove any foreign matter g. Eversion of the eyelids should be done to remove any retained particulate matter such as lime or cement

225

OCULAR TRAUMA AND EMERGENCIES /226

h. Irrigation should be continued until pH paper reveals that the conjunctiva’s pH is close to normal (pH between 7.3 and 7.7). Patient should be checked every 5 minutes to make sure that the pH is not changing again in the direction of alkalinity or acidity. 2. Medications a. Mydriatic-cycloplegics such as Atropine 1% should be instilled to dilate the pupil and prevent posterior synechiae and reduce painful iridociliary spasm b. Antibiotic drops such as Ciprofloxacin, Ofloxacin, Tobramycin, or Polymyxin-Bacitracin c.Carbonic anhydrase inhibitors for increased IOP 3. Analgesics a. Paracetamol 500 mg one tablet per orem every 3 to 4 hours b. Meperidine 50 to 100 mg intramuscularly 4. Emergency complete physical examination a. By an otolaryngologist, because toxic chemicals are aspirated or swallowed during the initial injury b. By an internist, to rule out concomitant burns of the respiratory or upper gastrointestinal tract c. Acute obstruction of the airway due to laryngeal edema may occur 5. Patching of the burned eye once stable a. Admit patient b. If burn is mild, continue medications and follow-up on an outpatient basis CENTRAL RETINAL ARTERY OCCLUSION (CRAO) A. History ƒ pertinent medical history (vaso-occlusive diseases) ƒ may be preceded by transient ischemic attacks of visual blurring or blackout in embolic or inflammatory vasculitic disease ƒ most common cause is embolization, with emboli arising from fatty material of atheromas, calcium deposits of diseased heart valves, septic and non-septic fibrin, and platelet thrombi ƒ atheromas, calcium deposits of diseased heart valves, septic and non-septic fibrin, and platelet thrombi B. Ocular findings 1. Visual acuity 2. Painless loss of vision to 20/400 (unless the patient retains central vision via a cilioretinal artery supplying the papillomacular nerve fibers) 3. Vision with no light perception suggests choroidal ischemia due to ophthalmic artery occlusion in addition to CRAO 4. Funduscopy ƒ Retina at the posterior pole becomes milky white and swollen ƒ “Cherry-red spot” at the fovea is seen if cilioretinal artery is intac ƒ Hemorrhage is minimal

226

OCULAR TRAUMA AND EMERGENCIES /227

Cherry red spot at the macula

Pale , edematous retina.

C. Treatment – should be carried out immediately for anyone presenting within 24 hours of visual loss due to CRAO 1. Correction of specific precipitating event – orbital decompression for 2. acute retrobulbar hemorrhage or ocular hypotension for acute glaucoma 3. Non-specific methods to increase blood flow and dislodge emboli a. Digital massage, or ocular massage using a three-mirror contact lens for approximately 10 seconds, to obtain central retinal artery pulsation or cessation of flow, followed by 5 seconds of release b. The aim is to mechanically collapse the arterial lumen and cause prompt changes in arterial flow. c.Carbonic anhydrase inhibitors – acetazolamide d. Sublingual isosorbide dinitrate 10 mg to dilate peripheral blood vessels and decrease resistance e.Intravenous methylprednisolone for possible arteritis f. 95% oxygen-5% carbon dioxide mixture to dilate retinal vessels g. Paracentesis of aqueous humor to decrease IOP acutely ACUTE ANGLE CLOSURE GLAUCOMA A. History a. Sudden onst of severe blurring of vison followed by excruciating pain, halos, nausea and vomiting B. Ocular findings ƒ Red eye (ciliary injection) ƒ Hazy or steamy cornea ƒ Moderately dilated and unreactive pupil ƒ Increased intraocular pressure (IOP) ƒ Shallow anterior chamber

Ciliary injection, hazy cornea, middilated pupil

227

OCULAR TRAUMA AND EMERGENCIES /228

C. Treatment 1. Reducce IOP ƒ Oral acetazolamide ƒ IV and oral hyperosmotic agents (mannitol and glycerine respectively) ƒ Topical beta blockers ƒ Pilocarpine 2-4% I drop every15 minutes for 1 hour 2. Laser peripheral iridotomy 3. Prophylactic laser iridotomy of the fellow eye

LACERATIONS OF THE EYELIDS 1. Determine if there are associated ocular injury. If there are, REFER to an ophthalmologist 2. Consider what special structures w/in the area of laceration may be injured. Ex canaliculi, levator muscle, lacrimal gland. 3. Is there Significant tissue loss ? 4. Lacerations not involving the lid margins 5-0 chromic, dexon or vicryl 6-0 silk for skin 5. Lacerations involving lid margins, REFER to an ophthalmologist Cosmetic & functional success depends on exact re-approximation 6. Antibiotics, anti-tetanus, cold compresses

CORNEAL ABRASION Causes 1. Fingernail or any object to the eye 2. Contact lens over wear 3. UV burn from welding Abraded portion of the ccornea stains with green fluorescein dye

228

OCULAR TRAUMA AND EMERGENCIES /229

Signs & Symptoms 1. Pain 2. Photophobia 3. Tearing 4. Lid swelling 5. Blurring of vision 6. Epithelial defect Management 1. Removal of occult foreign body 2. Antibiotic drops 3. Cycloplegic drops 4. Eye patch CORNEAL FOREIGN BOBIES Characteristic of FOREIGN BODY 1. Metallic - rust ring 2. Vegetable - increased risk of infectious keratitis Examination 1. How deep did the FB penetrate? 2. Are there other FB present, extra or intra- ocular? 3. Is infection present? Management 1. Superficial FB - irrigation, use of moist cotton swab 2. Deeper FB - G25 needle under slit lamp, remove rust ring 3. Antibiotic drops

Metallic foreign body on the corneal limbus at 5 o’clock position

Will there be a scar after the removal of the foreign body ? Yes, if Bowman’s and stroma injured. CONTUSION OF THE EYEBALL Hyphema – blood in the anterior chamber

229

OCULAR TRAUMA AND EMERGENCIES /230

Commotio retinae – retinal edema

Retinal detachment

230

OCULAR TRAUMA AND EMERGENCIES /231

231

OCULAR TRAUMA AND EMERGENCIES /232

ORBITAL FRACTURE Usually occurs with facial trauma and may be associated with intraocular injury. When orbital entrance receives a blow, compressive forces can fracture the thin medial and inferior walls with prolapse and possible entrapment of soft tissues. .Enopthalmos may develop. Diplopia can be due to direct neuromuscular damage, swelling of the orbital content or entrapment of the inferior rectus and inferior oblique within the fracture.

Management 1. CT scan, axial and coronal views 2. Antibioltics and anti-inflammatory drugs 3. Surgical repair SUMMARY All physicians, especially those working in the emergency rooms should be familiar with the treatment of the injured eye and its adnexae. He should be skilled in rendering immediate treatment and should prevent

232

OCULAR TRAUMA AND EMERGENCIES /233

further injuries to the patient. He should likewise be familiar with the common non-traumatic emergencies like central retinal artery occlusion and acute angle closure glaucoma. Most cases require referral to the ophthalmologist for definitive treatment. REFERENCES 1. Kanski , J.J. Clinical Ophthalmology- A Systemic Approach. Butterworth H. Einemann, 5th Edition 2003 2. Pavan-Longston, D. (Ed), Manual of Ocular Diagnosis and Therapy. Lippincott Williams and Wilkins, 2002 3. Parver, L.M. and Pieramice D.J. (Ed) Issues in Ocular Trauma. Ophthalmology Clinics of North America. 8 (4) December 1995, 589-708 4. Riordan-Eva, P, Whitcher, J.P.. Vaughn and Ashbury’s General Ophthalmology , 16th Edition, New York: Lange Medical Books/ McGraw Hill, 2004 SELF-TEST 1. Signs of acute angle closure glaucoma include the following, EXCEPT A. Ciliary injection B. Irregular miotic pupil C. Hazy cornea due to bedewing D. Ocular pain E. Blurring of vision 2. Which of the following red eye conditions would require early recognition due to their potentially vision threatening nature A. Internal hordeolum B. Viral conjunctivitis C. Chemical burn D. Blepharitis E. Nasolacrimal duct obstruction 3. A patient presents with a one day history of severe eye pain and marked blurring of vision. Upon doing your ophthalmologic examination, you note that the patient has a mid-dilated pupil with a firm eyeball. Based on this, your primary consideration would be A. Corneal abrasion B. Contact lens overwear C. Bacterial conjunctivitis D. Acute angle closure glaucoma 4. The following conditions require IMMEDIATE intervention or treatment: A. endophthalmitis and orbital rim fracture B. canalicular transaction and intraocular foreign body C. chemical burn and central retinal artery occlusion D. hyphema and central retinal vein occlusion E. scleral laceration and commotion retinae 5. This step in the ocular examination may be deferred if globe rupture is suspected: A. visual acuity B. gross examination C. extraocular muscle movement test D. palpation tonometry E. direct funduscopy 233

OCULAR TRAUMA AND EMERGENCIES /234

6. A patient who figured in a vehicular accident was referred from another hospital with a diagnosis of traumatic optic neuropathy. What examination will help confirm this diagnosis? A. Hirschberg light reflex test B. swinging flashlight test C. flashlight test using a slit beam D. extraocular muscle movement test E. lid elevation test 7. What clinical finding is not seen in a patient with orbital floor fracture? A. ecchymosis B. diplopia C. hypesthesia D. limited extraocular muscle movements E. exophthalmos 8. A patient’s eye was hit by a tennis ball. Which of the following tests will not be useful in confirming your diagnosis of commotio retinae? A. Hirschberg light reflex test B. Amsler grid test C. confrontation test D. direct funduscopy 9. Retinal artery occlusion is NOT commonly associated with: A. emboli from atheromas B. retrobulbar hemorrhage C. hypertension D. thyroid exophthalmos E. temporal arteritis 10. The following measure is part of the long-term management of chemical burns: A. copious irrigation with NSS B. sweeping conjunctival fornices with cotton-tipped applicators C. determination of conjunctival pH D. mydriatic-cycloplegic eyedrops E. artificial lubricants Answers to Self –Test 1. 2. 3. 4. 5. 6. 7. 8. 9.

B C D C D B E D C

10. E

234

OCULAR PHARMACOLOGY / 235

OCULAR PHARMACOLOGY Rosie Reyes-Noche, M.D. INTRODUCTION In clinical practice, an ophthalmologist listens to the patients reason for consulting then examines the patient thoroughly. He should be able to recognize the common ocular signs and symptoms and correlate these with the corresponding anatomic and physiologic changes in the eye and related structures. .A working diagnosis is formulated and finally decides on the management. Ocular Pharmacology deals with the basic properties of drugs, their action, their fate in the human body and their known side-effects. It refers to drugs utilized for the diagnosis and treatment of ocular diseases. OBJECTIVES At 1. 2. 3.

the end of this unit of instruction, the medical student should be able to Discuss the various ways in which drugs act on the body, with specific emphasis on drugs used in the eye. Discuss the diagnostic and therapeutic uses of drugs on the eye. Discuss the implications of systemically administered drugs to ocular function. PRE-REQUISITE KNOWLEDGE & PREPARATION

The student should have an overview of the normal anatomy and physiology of the eye. To evaluate the patient with eye problem, he should be able to do a thorough history taking and basic eye examination. He should be aware of common eye conditions causing the following : Vision changes eye redness, eye pain, tearing, eye discharge, eye deviation, eye displacement.. He should be able to decide whether the eye condition is urgent and considered as an ocular emergency and make the necessary referrals. CONTENT I. Routes of administration II.Ocular Diagnostic Drugs (uses) Fluorescein dye Anesthetics Mydriatics Cholinergic Blocking agents Adrenergic stimulating drugs III. Topical Ocular Therapeutic Drugs ( Indications/Mechanism of Action/Side Effects) Decongestants Anti-inflammatory Agents Antibiotics IOP lowering Agents: Beta-Adrenergic Blockers Cholinergic Stimulating Drugs Adrenergic Stimulating Drugs Prostaglandin Analogs IV. Systemic Medications Used for Ophthalmic Conditions: Glaucoma Medications Systemic Anti-inflammatory Agents

235

OCULAR PHARMACOLOGY / 236

Systemic Antibiotics V. Ocular Side Effects Of Systemic Drugs PART I. ROUTES OF DRUG ADMINISTRATION A. topical B. systemic C. subconjunctival injections - retrobulbar injections A. Topical : Placed directly into the eye. There are 4 forms of topical eye medications 1. Suspensions 2. Solutions 3. Ointments 4. Gels B. Systemic : Delivery of the drug to the affected area by way of the bloodstream 1. Oral (P.O.) : by mouth 2. Intramuscular (I.M.) : injection in the muscle 3. Intravenous (I.V.) : injection into the vein 4. Subcutaneous (S.C.): injection under the skin C. Subconjunctival Injection Made under the conjunctiva to gain access to the deep structures of the eye by absorption into the blood stream by way of the episcleral and conjunctival blood vessels. D. Retrobulbar Injection Made directly through the skin of the lower lid with the point of the needle emerging behind the eyeball PART II. OCULAR DIAGNOSTIC DRUGS A. FLUORESCEIN DYE Water soluble non-toxic dye, yellow at neutral pH, fluoresces with cobalt blue stimulation Uses: 1. Diagnosis of corneal abrasion 2. Tear layer evaluation in dry eye or Contact lens fitting 3. Applanation tonometry for glaucoma 4. Diagnosis of aqueous leak post surgery 5. Injected intravenously for angiography of retina B. TOPICAL ANESTHETICS Uses: 1. Removal of foreign body/sutures 2. Applanation tonometry They are mildly toxic to the cornea and when used in excess can retard healing. Examples 1. Proparacaine HCl 0.5% 2. Tetracaine HCl 0.5% 3. Lidocaine 1.0% 4. Oxybupavacaine 0.4%

236

OCULAR PHARMACOLOGY / 237

C. MYDRIATICS AND CYCLOPLEGICS These are drugs which produces pupillary dilatation and are used to allow adequate examination of the fundus and to reduce the incidence of posterior synechiae formation in uveitis. They are also used pre-operatively in cataract surgery. The stronger mydriatics also paralyzes the ciliary body and are known as cycloplegics.. They are used to paralyze accommodation in cycloplegic refraction and are used in iritis, cyclitis and diffuse uveitis 1. Sympathomimetic : Phenylephrine 2.5% 2. Parasympatholytics: A. Atropine 1%, B. Cyclopentolate 1% C. Tropicamide 0.5% D. CHOLINERGIC BLOCKING AGENTS Causes pupillary constriction or miosis 1. Direct acting cholinergic blocking agents (Parasympathomimetic) : Pilocarpine Hcl eye drops 1%,2% and 4%. Stimulates the ciliary muscle and subsequently increases outflow. Used primarily in acute angle closure glaucoma 2. Anti-cholinesterase Isofluorophate( DFP) Echothiopate iodide (Phospholine iodide) Domecanium bromide (Humorsol) Physostigmine salicylate (Eserine) Neostigmine bromide E. Adrenergic Stimulating Drugs Its use in ophthalmology is for vasoconstriction and ocular pressure reduction.Used in glaucoma patients after laser trabeculoplasty Examples: 1. Epinephrine eye drops 2. Dipivefrine(Dipivalyl epinephrine) 3. Apraclonidine eye drops . PART III. Topical Ocular Therapeutic Drugs A. DECONGESTANT Indications:For short term relief of eye redness and eye irritation. Mechanism of action: Vasoconstrictors Side-effects: Angle closure in narrow angles and keratitis sicca (dry eye syndrome) Examples : 1. Naphazoline HCl 0.005%-0.01% 2. Tetrahydrozoline HCl 0.04%(Visine/EyeMo) 3. Antazoline HCl 0.05% B. ANTI-INFLAMMATORY AGENTS 1. Non-steroidal Anti-inflammatory Agents (NSAID)

237

OCULAR PHARMACOLOGY / 238

Indications: 1. Used for prevention of intra-operative miosis 2. Post-operative and traumatic inflammation control. 3. Vernal keratoconjunctivitis 4. Uveitis / Scleritis Mechanism of action : Anti-prostaglandin. In the eye, prostaglandins increase vascular permeability, produce miosis and cause the breakdown of the blood-aqueous barrier. Side effects: 1. Transient stinging and burning on instillation 2. Ocular irritation 3. Allergic reactions Examples : 1. Diclofenac 0.1% (Voltaren/Naclof) 2. Indomethacin 1.0% (Indocin) 3. Suprofen 1% (Profenal) 4. Flurbiprofen 0.03% (Ocufen) 5. Ketorolac 0.5% (Toradol) 2. Corticosteroids Indications : post-operative inflammations, anterior uveitis, severe allergic reactions Mechanism of Action: Suppress inflammatory response by inhibiting edema, cellular infiltration, capillary dilatation, fibroplatic proliferation, deposition of collagen, scar formation. Side-Effects 1. steroid induced glaucoma 2. cataracts 3. activation of infections and in some cases corneal perforation. Examples: 1. Prednisone/ Prednisolone 2. Dexamethsone 3. Fluorometholone C. ANTIBIOTICS Indications : For external eye infections Mechanism of action: Bactericidal or bacteriostatic by inhibition of protein/mucopeptide/ cell wall synthesis Side-Effects:local conjunctival irritation, stinging, burning, allergic reactions, sensitization Examples : 1. Aminoglycosides, 2. Penicillins 3. Cephalosporins, 4. Bacitracin 5. Chloramphenicol, 6. Erythromycin, 7. Quinolones, 8. Sulfonamides, 9. Tetracyclines

238

OCULAR PHARMACOLOGY / 239

D. IOP LOWERING AGENTS 1. Beta adrenergic Blocking Agents Indications: Open angle glaucoma Mechanism of Action: Reduce IOP by suppressing aqueous production Side Effects:Transient ocular burning, stinging, itching, discomfort, dryness, dizziness , headache. Examples: 1. Timolol Maleate 0.25 –0.50% 2. Betaxolol Hcl 0.25- 0.50% 3. Levobunolol Hcl 0.50% 4. Metipranolol Hcl 0.3% 2. Cholinergic Stimulating Drugs Indication : for acute angle closure glaucoma Mech.of Action: Relief of pupillary block by constricting the pupil and tautening the iris IOP is reduced when the root of the iris pulls away from the trabecular meshwork and angle opens. Contracts ciliary muscles leading to increase aqueous outflow Side-effects 1. Increase permeability of blood-aqueous barrier leading to greater inflammation 2. Accommodation spasm resulting to pain and blurred vision 3. Constriction of visual fields Example : Pilocarpine 3. Adrenergic Stimulating Drugs Indications : for vasoconstriction and reduction of intra-ocular pressure Mechanism of Action : Increase aqueous outflow, decrease aqueous secretion Side-Effects : Burning,pain,allergy Examples : 1. Epinephrine 2. Dipivalyl epinephrine 3. Apraclonidine 4. Prostaglandin Analogs Indications :Primary open angle glaucoma, ocular hypertension Mechanism of Action : Decrease IOP by increasing uveo-scleral outflow Side-effects : Increase iris pigmentation, lengthening of eyelashes, and hyperemia Examples :: 1. Latanoporst 50 mcg/ml(Xalatan ) 2. Bimatoprost 0.03% ( Lumigan ) 3. Travoprost 0.004% ( Travatan) PART IV. Systemic Drugs s Used For Ophthalmic Conditions A. GLAUCOMA MEDICATIONS 1. Carbonic Anhydrase inhibitors Indications : acute angle closure glaucoma Mechanism of Action: Decrease aqueous production Side-Effects: GIT discomfort, paresthesia, mental confusion, transient myopia, anemia, agranulocytosis Example : Acetazolamide(Diamox ) 250 mg./tab

239

OCULAR PHARMACOLOGY / 240

2. Osmotic Agents Indications: Acute angle closure glaucoma Mech.of Action : Increase serum osmolarity to reduce intra-ocular(vitreous) Side-Effects=caution in CHF,avoid glycerol in diabetics Examples 1. Glycerol 50% oral solution 2. Mannitol 20% IV solution

water content

B. SYSTEMIC ANTI-INFLAMMATORY AGENTS 1. NSAID: Indications : ocular inflammation where steroids are contraindicated. Mechanism of action : prevent synthesis and release of PGE 1,and PGE 2 Side-effect ; :Gastric irritation, hyperacidity Examples : 1. Aspirin 2. Indomethacin 2. Corticosteroids Indication : posterior uveitis, optic neuritis, sympathetic ophthalmia Mechanism of Action : anti-inflammatory Side-Effects: Suppress resistance to invasion of micro-organism, systemic (moon facies, buffalo hump,acne) C SYSTEMIC ANTIBIOTICS Indications : Severe intra-ocular infections , bacterial endophthalmitis, infections of the adnexae and orbit Mechanism of action : Bactericidal or bacteriostatic Side-Effects : Renal toxicity, hepatic toxicity , allergic reactions, nausea, vomiting or diarrhea Examples : 1. Aminoglycosides 2. Penicillin 3. Sulfonamides 4. Cephalosporins 5. Tetracycline 6. Chloramphenicol 7. Erythromycin 8. Clindamycin PART V. OCULAR SIDE-EFFECTS OF SYSTEMIC DRUGS A. IRREVERSIBLE SIDE EFFECTS SIDE EFFECTS 1. Optic neuropathy

EXAMPLES OF DRUGS Disulfiram (for alcoholism) Hexachlorophene(antiseptic) Vincristin/Busulfan (anti-cancer)

2. Retinal degeneration 3. Conjunctival fibrosis 4. Posterior subcapsular cataract

Chloroquine/ hydrochloroquine (for arthritis) Practolol (for hypertension) Glucocorticoids (for inflammation)

240

OCULAR PHARMACOLOGY / 241

5. Pigment epitheliopathy 6. Retinal degeneration

Quinine (for muscle spasms) Chlorpromazine (for psychiatric disorders)

B. REVERSIBLE SIDE EFFECTS SIDE EFFECTS 1. Blepharoconjunctivitis

EXAMPLES OF DRUGS Retinoic acid( for acne)

2. Pseudotumor cerebri

Tetracycline/Vit A (for acne) Hexachlorophene(antiseptic) Minocycline ( anti-infective) Glucocorticoids

3. Corneal epithelial deposits and retinal deposits:

Chloroquine/ hydroxychloroquine Tamoxifen/Tilorone ( for cancer) Amidarone ( for cardiac arythmia)

4. Lens deposit 5. Myasthenia 6. Altered colored perception and visual hallucinations

D-penicillamine D-penicillamine (for arthritis) Digitalis/Digoxin/Lanatoside C (for cardiac arythmias)

7. Nystagmus

Diazepam/Phenobarbital/Phenytoin (anti-convulsant)

8. Myopia

Carbonic anhydrase inhibitors (for glaucoma) Sulfonamides (anti-infective) Bromocriptine (for Parkinsonism)

9. Optic neuropathy

Chloramphenicol (for infection) Ethambutol Diethylcarbamazine Suramin

10. Mydriasis and Cycloplegia

anti-histamines anti-Parkinson anti-muscarins ( for peptic ulcer)

11. Ocular hypertension

Glucocorticoids

12. Lens opacification 13. Oculogyric crisis

Methoxsalen ( for psoriasis) MAO inhibitors( for psychiatric disorders)

14. Corneal and Lens deposits

Thioridazine ( for psychiatric disorders)

15. Retinal degeneration

Chlorpromazine ( for psychiatric disorders)

241

OCULAR PHARMACOLOGY / 242

SUMMARY This course presents an overview of the clinical pharmacology of the ocular diagnostic and therapeutic agents. Specific topics include clinical pharmacokinetics of the eye, use of autonomic agents for the production of miosis, mydriasis and cycloplegia and the use of local anesthetics and dyes. The pharmacology and toxicology of the ocular anti-allergic, anti-inflammatory and anti-infective agents as well as drugs used in the treatment of glaucoma is presented. The principles of drug administration,evaluation of therapeutic response, and ocular & systemic adverse reactions is also included. REFERENCES 1. Bartlett and Jaanus .Clinical Ocular Pharmacology 2nd ed. 2. Vale and Cox . Drugs and the Eye , 3rd ed. 3. O’Connor Davies,Hopkins & Pearson . The Action and Uses of Ophthalmic Drug, 4th ed. SELF-TEST 1. Eye medications are usually given as: A. topical eye drops B. peri-ocular injections C. systemic oral D. topical eye ointments 2. Acute catarrhal conjunctivitis is treated with; A. antibiotics B. anti-inflammatory C. anti-viral D. anti-fungal 3. Mydriatics and cycloplegics are drugs that A. dilates the pupil B. paralyzes accommodation C. used in uveal inflammation D. all of the above 4. The drug most likely to increase intra-ocular pressure A. Corticosteroids B. ketamine C. Trichloroethylene D. alcohol 5. One of the following drug may cause optic neuritis: A. Ephedrine B. Ethambutol C. Vit A D. Proparacaine

242

OCULAR PHARMACOLOGY / 243

6. The following drug reduce(s) aqueous production: A. Metipranolol B. Betaxolol C. Pilocarpine D. All of the above 7. Corticosteroids: A. slows epithelialization B. stabilizes lysosomal membrane C. delays wound healing D. all of the above 8. In a case of bacterial endophthalmitis, the drug of choice is A. Oral antibiotic B. topical antibiotic C. oral anti-inflammatory D. topical anti-inflammatory 9 Local injections: A. will deliver drugs that penetrate the cornea poorly B. may provide repository for drugs for days or weeks C. may be used to deliver antibiotics intra-ocularly in severe endophthalmitis D. all of the above 10 Fluorescein dye is used in ophthalmology for the following: A. diagnosis of corneal abrasion B. injected IV for retinal angiography C. applanation tonometry for glaucoma D. all of the above

Answers To Self Test 1. A 2. A 3. D 4. A 5. B 6. D 7. D 8. A 9. D 10. D

243

View more...

Comments

Copyright ©2017 KUPDF Inc.
SUPPORT KUPDF